Download as pdf or txt
Download as pdf or txt
You are on page 1of 148

NAME OF THE OWNER: ……………………………………………………………………………

DEGREE PROGRAM: …………………………………………………………………………………

CONTACTS: ………………………………………………………………………………………

DESIGNED AND PRINTED BY: NUNDU EMMANUEL JR

Tel: 0762813988, 0715699968


MUHIMBILI UNIVERSITY OF HEALTH AND ALLIED SCIENCES
SCHOOL OF MEDICINE

DEPARTMENT OF ANATOMY
MODULE 1: GROSS ANATOMY CONTINOUS EXAMINATION
NOVEMEBER 18TH, 2013 TIME: 1HOUR

INSTRUCTIONS:
1. This paper consists of 50 multiple choice questions; answer all of them.
2. For each question select the most correct answer and write its corresponding letter alongside its
question number in the answer table provided below
3. There is no penalty for any wrong answer; but guesswork is discouraged
_____________________________________________________________________________________

YOUR REG NO.: ………………………………………………………………………………………. .


DEGREE PROGRAM: ………………………………………………………………………………….

WRITE YOUR ANSWERS IN THIS TABLE


QN. ANSWER QN. ANSWER
1. 26.
2. 27.
3. 28.
4. 29.
5. 30.
6. 31.
7. 32.
8. 33.
9. 34.
10. 35.
11. 36.
12. 37.
13. 38.
14. 39.
15. 40.
16. 41.
17. 42.
18. 43.
19. 44.
20. 45.
21. 46.
22. 47.
23. 48.
24. 49.
25. 50.

RE-WRITE YOUR REG. NO ………………………………………………


1. Which early scientist is credited as the extending to the posterior midline. What
father of modern medicine? is this structure?
A. William Harvey A. Coracoid process
B. Hippocrates B. Acromion
C. Alexander Fleming C. Inferior scapular angle
D. Andreas Vesalius D. C7 transverse process
E. Galen E. Scapular spine
2. Pick the “odd man” out; 8. During the inhalation, the diaphragm:
A. Transverse A. Remains static
B. Median B. Moves upward and forward
C. Frontal C. Moves downward and backward
D. Sagittal D. Relaxes and shrinks
3. Which movement is employed at the E. Recoils
forearm when loosening a bolt with the 9. The accumulation of blood in the pleural
right upper limb? cavity is called:
A. Circumduction A. Hemopneumothorax
B. Anticlockwise rotation B. Tension pneumothorax
C. Supination C. Haemothorax
D. Clockwise rotation D. Empyema thoracis
E. Pronation E. Hydrothorax
4. A plane that divides the body into 10. The anterior descending artery supplies
superior and inferior parts is : the:
A. Coronal A. Right ventricle
B. Sagittal B. Right atrium
C. Anteroposterior C. Left atrium
D. Transverse D. Apex and the Left ventricles
E. Contralateral E. Apex and the Right ventricles
5. An example of a primary cartilaginous 11. All of the blood returning from coronary
joint is: circulation will eventually collect in
A. Manubriosternal which vessel before pouring into the
B. Xiphisternal right atrium?
C. Intervertebral A. Coronary sinus
D. Costovertebral B. Great cardiac vein
E. Symphsis C. Oblique vein of LV
6. The esophagus is a muscular tube about D. Venae cordis minimae
how many cm long? E. Middle cardiac vein
A. 10 12. The heart chamber which is closely
B. 15 related to the esophagus on its posterior
C. 20 aspect is the:
D. 30 A. Right ventricle
E. 25 B. Left ventricle
7. During the physical examination of the C. Right atrium
back, a doctor palpated an obliquely D. Left atrium
disposed bony elevation in the shoulder E. Right and Left ventricles
CASE SCENARIO: During a physical E. 5th intercostal space mid-
examination of a child with pneumonia, a clavicular line
pediatrician palpated a pointed bony structure in 18. A patient with congestive heart failure
the superior midline chest area. He explained to (CHF) was noted to have an overtly
the students that it is an important landmark in swollen and pulsating vein in the
auscultation. Answer Qn 13-15 anterior triangle of the neck below the
mandible. What blood vessel is this
13. Anterolaterally, the point is at the level likely to be?
of: A. Common carotid artery
A. 1st rib B. Internal carotid artery
B. 2nd rib C. Superior vena cava
C. 3rd rib D. Internal jugular vein
D. 4th rib E. Brachiocephalic vein
E. 5th rib 19. The distinguishing features of a typical
14. Posterioly, it is at the level of: cervical vertebra include all of the
A. C7 following EXCEPT:
B. T1 A. Triangular vertebral foramen
C. T2 B. Small vertebral body
D. T3 C. Long transverse process
E. T4 D. Bifid spinous process
15. Internally, which of the following E. Horizontal articular surface
structures is not located at the horizontal 20. Which of the following thoracic
plane of the point? dermatomes is found in the upper limb?
A. Arch of aorta A. T1
B. Plane of the Diaphragm B. T3
C. Division of superior and inferior C. T5
mediastinum D. T9
D. Superior vena cava bifarcation E. T11
E. Bifarcation of the Trachea 21. The most superficial muscle of the neck
16. The right subclavian artery branches is;
from the: A. Platysma
A. Arch of Aorta B. Temporalis
B. Brachiocephalic trunk C. Sternocleidomastoid
C. Ascending aorta D. Subclavius
D. Descending aorta E. Omohyoid
E. Common carotid artery 22. The underwater seal drainage (uwsd) is
17. The apex beat is located at: done at the 7th intercostal space:
A. 6th intercostal space mid-axillary A. Mid-clavicular line above the
line rib below
B. 7th intercostal space mid- B. Mid-axillary line below the rib
clavicular line above
C. 5th intercostal space mid-axillary C. Mid-axillary line above the rib
line below
D. 6th intercostal space mid-
clavicular line
D. Mid-clavicular line below the 27. The shoulder joint is commonly
rib above dislocated:
E. Mid-clavicular right at the A. Inferiorly
center of space B. Anteriorly
23. The right coronary artery takes its origin C. Posteriorly
from the: D. Superiorly
A. Right atrium E. Laterally
B. Right ventricle
C. Right aortic sinus 28. The pointed part at the tip of the breast
D. Right aortic artery is called:
E. Descending aorta A. Breast proper
24. A young boy slept with his arm pressing B. Areola
against the edge of the bed. The C. Nipple
following morning the child complained D. Upper quadrant
of the numbness in the skin on the E. Lower quadrant
medial hand extending to medial two
fingers. Which nerve was most likely 29. Which of the following muscle of the
affected? upper limb assist in adduction of the
A. Ulnar shoulder joint?
B. Radial A. Pectoralis major
C. Median B. Infraspinatus
D. Superficial radial C. Supraspinatus
E. Musculocutaneous D. Subscapularis
E. Teres minor
25. A nurse wanted to insert cannula for
blood transfusion via the dorsal venous 30. Which of the following vessels of the
network of the hand. At the base of upper limb is commonly used for blood
which finger are the veins least variable pressure determination?
and hence most easily located? A. Axillary artery
A. Little finger B. Brachial artery
B. Ring finger C. Radial artery
C. Middle finger D. Subclavian artery
D. Index finger E. Ulnar artery
E. Thumb
CASE SCENARIO: Following a road
26. A phlebotomist wanted to draw blood accident a motorcyclist was rushed in at
from a superficial vein in the lateral arm the emergency department. On
area. What vessel is that? examination there was oozing of blood
A. Axillary from the lateral forehead and a CT scan
B. Basilic showed a fracture of a site where four
C. Cephalic bones articulated and an injury to an
D. Median cubital intracranial artery deep to the point.
E. Radial From this scenario answer Qns. 31-33
31. From the scenario, what is the point of A. Maxilla
the injury? B. Nasal
A. Pterion C. Zygomatic
B. Zygoma D. Hyoid
C. Vertex E. Frontal
D. Frontal pole
E. Anterior fontanelle 37. The heart, lungs and most of the
abdominal viscera receive their
32. From the scenario, which of the parasympathetic supply from which
following bones was unlikely affected? nerve(s)?
A. Frontal A. Oculomotor
B. Parietal B. Facial
C. Zygomatic C. Glossopharyngeal
D. Temporal D. Vagus
E. Sphenoid E. Spinal S2-S4

33. Which intracranial artery was injured? 38. A young mother was worried when she
A. Internal carotid noted a palsating area in the anterior
B. Middle meningeal head of her child. The physician
C. Ophthalmic informed her that it was a normal
D. Anterior cerebral fontanelle and should be closed by the
E. Middle cerebral age of:
A. 12 months
34. The skeleton of the neck is formed by B. 18 months
how many cervical vertebrae? C. 3 months
A. 5 D. 9 months
B. 6 E. 15 months
C. 7
D. 8 39. While administering an anesthetic, a
E. 9 dentist injected the drug in the medial
aspect of the mandible to block the
35. A severely ill child presented with fever, nerve as it enters foramen. What nerve
difficult breathing and general body is that?
weakness. She had a distended and A. Mandibular
pulsating vessel just anterior to the ears B. Inferior alveolar
sending branches into face and above C. Maxillary
the ears. What artery is this? D. Lingual
A. Facial E. Facial
B. Middle meningeal 40. From Qn.39 above, what is the foramen
C. Superficial temporal in question?
D. Mandibular A. Mandibular
E. External carotid B. Mental
36. The bones of the face include the C. Alveolar
following EXCEPT: D. Lingual
E. Palatine E. Ellipsoid
41. Arrange the brain meninges orderly 47. Choose the most correct statement:
from within to without: A. Nerves are synonymous to
A. Arachnoid,pia,dura neurons
B. Pia,arachnoid,dura B. Neurons are made nerves
C. Dura,pia,arachnoid C. Nerves are made of neurons
D. Pia,dura,arachnoid D. Microglia support neurons
E. Dura,arachnoid,pia E. Microglia support nerves

42. The foramen rotundum transmits: 48. The ulnar nerve arises from which of the
A. Mandibular nerve following part of brachial plexus?
B. Middle meningeal artery A. Medial cord
C. Maxillary nerve B. Lateral cord
D. Internal carotid artery C. Posterior cord
E. Hypoglossal nerve D. Superior branch
E. Inferior branch
43. The foramen ovale transmits: 49. Numbness to the skin on the lateral half
A. Mandibular nerve of the palm of the hand would be caused
B. Middle meningeal artery by a lesion to which nerve?
C. Maxillary nerve A. Ulnar
D. Internal carotid artery B. Radial
E. Hypoglossal nerve C. Median
D. Superficial radial
44. The massive muscle of the upper thorax E. Anterior interosseous
anteriorly is the: 50. The motor neurons:
A. Trapezius A. Convey impulses to the CNS
B. Pectoris B. Convey impulses to the effector
C. Subscapularis organ
D. Serratus anterior C. Are exclusively located in the
E. Infraspinatus CNS
45. The extensive muscle of the upper D. Leads to numbness when
thorax posteriorly is the: injured
A. Trapezius E. Has their dendrites in peripheral
B. Pectoris organ
C. Subscapularis
D. Serratus anterior
E. Infraspinatus
46. The proximal radioulnar joint is the
prototype example of which typeof
joint?
A. Saddle END
B. Hinge
C. Ball and Socket
D. Pivotal
MUHIMBILI UNIVERSITY OF HEALTH AND ALLIED SCIENCE
SCHOOL OF MEDICINE
DEPARTMENT OF ANATOMY

FIRST CONTINOUS EXAMINATION FOR THE MD & DDS DEGREE PROGRAMS


DECEMBER 2010 TIME: 2hrs
_____________________________________________________________________________________
INSTRUCTIONS:
1. This paper consists of 75 multiple choice questions; answer all of them.
2. For each question choose the most correct answer and write its corresponding letter alongside its question
number on the ANSWER SHEET provided.
3. Do not use the question paper for answering this examination.
4. NOTE: ¼ will be deducted for every wrong answer
________________________________________________________________________

1. Any plane which is parallel to the 5. Which of the following muscle does
median plane is: NOT rotate the arm medially?
A. Coronal A. Subscapularis
B. Frontal B. Supraspinatus
C. Sagittal C. Latissimus dorsi
D. Medial D. Pectoralis major
E. None of the above E. Anterior fibers of the deltoid

2. Adduction refers to a movement: 6. Which of the following wrist bones is


A. Towards the median plane found in the proximal row?
B. Away from the median plane A. Trapezium
C. Around the median plane B. Trapezoid
D. That increase the angle of a C. Capitate
joint D. Hamate
E. All of the above E. Scaphoid

3. A typical vertebra contains the 7. The muscle that forms the bulk of the
following EXCEPT: anterior axillary fold is the:
A. Body A. Latissimus dorsi
B. Transverse foramen B. Pectoralis major
C. Vertebral foramen C. Subscapularis
D. Spinous process D. Teres minor
E. Transverse process E. Teres major

4. The long head of the biceps brachii 8. The axillary artery is divided into three
muscle arises from the: parts by these structures EXCEPT:
A. Infraglenoid tubercle A. Teres major
B. Acromion process B. Pectoralis major
C. Coracoid process C. Pectoralis minor
D. Clavicle D. First rib
E. Supraglenoid tubercle E. None of the above
10. Muscular attachment to the medial C. Accessory nerve
border of the scapular includes the D. Thoracodorsal nerve
following EXCEPT: E. Suprascapular nerve
A. Trapezius
B. Rhomboideus major 16. All of the following muscles are
C. Rhomboideus minor attached to the clavicle EXCEPT:
D. Levator scapulae A. Pectoralis minor
E. Serratus anterior B. Pectoralis major
11. Which of the following does not stem C. Deltoid
from the subclavian artery? D. Sternocleidomastoid
A. Internal thoracic artery E. None of the above
B. Thoracoacromial artery 17. The functions of the biceps brachii
C. Vertebral artery muscles:
D. Thyrocervical trunk A. Flexion of the shoulder
E. None of the above B. Flexion of the elbow
C. Supination of the elbow
12. Veins that converge to form the axillary D. A and B
vein are: E. A, B and C
A. Brachial vein 18. The median nerve innervates all these
B. Brachial venae comitantes muscles EXCEPT:
C. Basilica vein A. Pronator teres
D. A and B B. Palmaris longus
E. B and C C. Palmaris brevis
D. Flexor carpi radialis
13. Which of the following trunks of the E. Flexor digitorum superficialis
brachial plexus is closely associated
with the first rib? 19. The medial wall of the axilla is formed
A. Inferior by which of the following muscles?
B. Middle A. Teres major
C. Superior B. Serratus anterior
D. A,B and C C. Pectoralis major
E. None of the above D. Subscapularis
E. Pectoralis minor
14. The rotator cuff muscles comprises of
all of the following muscles EXCEPT: 20. The ulnar nerve arises from which of the
A. Supraspinatus following parts of the brachial plexus?
B. Infraspinatus A. Medial cord
C. Subscapularis B. Lateral cord
D. Teres major C. Posterior cord
E. Teres minor D. Superior branch
E. None of the above
15. The trapezius muscle is innervated by:
A. Dorsoscapular nerve
B. Long thoracic nerve
21. Numbness to the skin to the lateral half D. Rhomboid
of the palm of the hand would be caused E. Deltoid
by a lesion to which nerve? 27. Veins of the upper limb include:
A. Ulnar A. Basilic
B. Radial B. Cephalic
C. Median C. Median cubital
D. Superficial radial D. A and B
E. Anterior interosseous E. A, B and C
28. The nerve most likely to be injured in
22. The following dermatomes are found in fractures of the medial epicondyle is the:
the upper limb EXCEPT: A. Radial
A. C4 B. Axillary
B. C5 C. Ulnar
C. C6 D. Median
D. C8 E. Musculocutaneous
E. T1 29. Which artery supplies the posterior
compartment of the arm?
23. A branch of axillary artery is the: A. Axillary
A. Costocervical B. Brachial
B. Vertebral C. Ulnar
C. Dorsal scapular D. Profunda brachii
D. Suprascapular E. Radial
E. Subscapular 30. Muscles of hypothenar eminence are
innervated by the:
24. The triceps brachii muscle is inserted at A. Median nerve
the: B. Ulnar nerve
A. Radial tuberosity C. Radial nerve
B. Coracoids process of the D. Median and radial nerve
scapula E. Median and ulnar nerve
C. Olecranon process of the ulna 31. Which of the following is not a TRUE
D. A and B rib?
E. A and C A. 1st rib
B. 3rd rib
25. Brachialis muscle is inserted at the: C. 5th rib
A. Radial D. 7th rib
B. Musculocutaneous E. 9th rib
C. Ulnar
D. A and B 32. Which of the following depicts the best
E. A, B and C example of an atypical rib?
26. Which of the following muscles is not A. First rib
named based on their shapes? B. Third rib
A. Pyramidalis C. Fifith rib
B. Trapezius D. Seventh rib
C. Infraspinatus E. Ninth rib
33. The heart is located in which anatomical 39. Increased resistance to pulmonary blood
division of the mediastinum? flow in the lungs would cause a direct
A. Anterior strain on which chamber of the heart?
B. Middle A. Right ventricle
C. Posterior B. Left atrium
D. Lateral C. Right atrium
E. Superior D. Left ventricle
34. The aortic arch is located in which E. None of the above
subdivision of the mediastinum? 40. Which of the following accompanies the
A. Superior anterior interventricular artery?
B. Middle A. Middle cardiac vein
C. Posterior B. Coronary sinus
D. Anterior C. Small cardiac vein
E. None of the above D. Great cardiac vein
35. The left bronchial arteries frequently E. Anterior cardiac vein
take their origin from the:
A. Internal thoracic artery 41. The vagus nerve accompanies which of
B. Descending aorta the following through the diaphragm?
C. Aortic arch A. Sympathetic vein
D. Subclavian artery B. Inferior vena cava
E. None of the above C. Phrenic vein
36. All of the following are parts of the D. Aorta
parietal pleura EXCEPT: E. Esophagus
A. Cervical
B. Costal 42. During the surgical repair of the patent
C. Pulmonary ductus arteriosus, which nerve is likely
D. Diaphragmatic to be injured and hence needs extra
E. Mediastinal care?
37. How many lobes does the left lung A. Right recurrent laryngeal nerve
contains? B. Left phrenic nerve
A. 1 C. Left recurrent laryngeal nerve
B. 2 D. Left vagus
C. 3 E. Right phrenic nerve
D. 4
E. 5 43. The sinuatrial node is located in the:
38. Which of the following opening of the A. Right atrial wall
diaphragm is/are the most anterior? B. Left atrial wall
A. Caval opening C. Right ventricular wall
B. Esophageal hiatus D. Left ventricular wall
C. Aortic hiatus E. Interventricular septum
D. A and C
E. A,B and C 44. The azygous vein drains into the:
A. Superior vena cava
B. Inferor vena cava
C. Right atrium B. Moves upward and forward
D. Left brachiocephalic vein C. Moves downward and backward
E. Right brachiocephalic vein D. Relaxes and shrinks
45. Thoracic duct usually drains into the: E. Recoils
A. Left internal jugular vein 51. A collection of serous fluid inside the
B. Left subclavian vein pleural cavity is clinically termed as:
C. Junction of the left internal A. Ascites
jugular and subclavian veins B. Pneumothorax
D. Superior vena cava C. Pleurisy
E. Junction of the right internal D. Hydrothorax
jugular and subclavian veins E. Hydrocele
46. The following branches arise from the 52. The heart is supplied by the:
aortic arch EXCEPT: A. Coronary artery
A. Brachiocephalic trunk B. Aorta
B. Left common carotid artery C. Left pulmonary artery
C. Left subclavian artery D. Right pulmonary artery
D. Right subclavian artery E. Internal thoracic artery
E. None of the above 53. The base of the heart is mainly formed
by the:
47. If a child inhales a foreign body, it is A. Left atrium and partly right
more likely to be found at the: atrium
A. Left main bronchus B. Right atrium and right ventricle
B. Right main bronchus C. Left ventricle
C. At the carina D. Left ventricle and left atrium
D. Oesophagus E. Left ventricle and right atrium
E. none of the above 54. Exocytosis and endocytosis are
48. The root of the lungs contains the accomplished by the :
following EXCEPT: A. Nucleus
A. Bronchus B. Mitochondria
B. Vagus nerve C. Cell membrane
C. Pulmonary ligament D. Endoplasmic reticulum
D. Pulmonary artery E. All of the above
E. Pulmonary vein
55. Which of the following doesnot consist
49. The trachea bifurcates into right and left of intermediate cytoskeleton?
primary bronchi at the level of the: A. Vimentin
A. Plane of the sternal angle B. Cytokeratin
B. Suprasternal (or jugular) notch C. Actin
C. First rib D. Desmin
D. Seventh cervical vertebra E. None of the above
E. Xiphoid process
56. What part of the cell synthesizes
50. During inhalation, the diaphragm: protein?
A. Remains static A. Ribosomes
B. Mitochondria B. Chondrocytes
C. Lysosomes C. Osteoclasts
D. Endoplasmic reticulum D. Osteoblasts
E. All of the above E. None of the above
57. Histamine receptors are examples of: 63. Bone grows in length at the:
A. Ion channels A. Epiphysis
B. G-protein coupled receptors B. Metaphysis
C. Intracellular proteins C. Epiphysial plate
D. Receptors tyrosine kinase D. Periosteum
E. None of the above E. Diaphysis
58. The intracellular receptors include the 64. The absorptive surface of the gut is lined
following EXCEPT: by:
A. Glucocorticoid receptors A. Simple cuboidal epithelium
B. Progesterone receptors B. Simple columnar epithelium
C. Androgen receptors C. Stratified columnar epithelium
D. Estrogen receptors D. Stratified squamous epithelium
E. None of the above E. All of the above
59. Membrane bound organelles include the 65. The epidermis is formed by the:
following: A. Simple columnar epithelium
A. Mitochondria B. Simple squamous epithelium
B. Ribosomes C. Stratified squamous epithelium
C. Cytoskeleton D. Transitional epithelium
D. Nucleolus E. All of the above
E. Centrioles 66. The glial cells include all of the
60. The main site for adenosine following EXCEPT:
5’triphosphate energy molecule A. Microglia
production is the: B. Astrocytes
A. Mitochondrion C. Epindymal cells
B. Nucleus D. Oligodendrocytes
C. Microsomes E. None of the above
D. Lysosomes
E. Peroxisomes 67. The thyroid gland:
A. has two lobes joined by an
61. Which of the following is not found in a isthmus
bacterial cell? B. lies immediately anterior to the
A. DNA esophagus
B. Nuclear membrane C. lies immediately posterior to the
C. Cell membrane trachea
D. Cytoplasm D. is invested in superficial fascia
E. None of the above of the neck
E. all of the above
62. The following cells are found in the
bone EXCEPT: 68. The veins of the thyroid gland include:
A. Osteocytes A. Superior thyroid
B. Middle thyroid A. Semispinalis capitis
C. Inferior thyroid B. Splenius capitis
D. A and B C. Levator scapulae
E. A, B and C D. Trapezius
69. Transmitting the spinal cord and E. None of the above
maninges, the foramen magnum is an 75. The foramen spinosum is located in
opening in the: which bone?
A. Temporal bone A. Sphenoid
B. Parietal bone B. Occipital
C. Frontal bone C. Frontal
D. Occipital bone D. Temporal
E. Pterygoid bone E. Ethmoid

70. Intracellular calcium storage organelles


include:
A. Peroxisomes
B. Mitochondria
C. Endoplasmic reticulum ---SUCCESS---
D. A and B
E. B and C
71. The carotid sheath encloses the
following structures EXCEPT:
A. common and internal carotid
artery
B. vagus nerve
C. internal jugular vein
D. interior jugular vein
E. none of the above
72. The cranial nerve VI leaves the skull
through the:
A. foramen rotundum
B. jugular foramen
C. inferior orbital fissure
D. superior orbital fissure
E. foramen ovale
73. The nervous system includes:
A. Brain
B. Spinal cord
C. Peripheral nerves
D. A, B and C
E. A and B only
74. Which of the following muscles doesnot
form the floor of the posterior triangle of
the neck?
MUHIMBILI UNIVERSITY OF HEALTH AND ALLIED SCIENCES
SCHOOL OF MEDICINE

DEPARTMENT OF ANATOMY

GROSS ANATOMY MODULE FINAL EXAMINATION FOR THE DEGREES OF:

1. DOCTOR OF MEDICINE (MD)


2. BACHELOR OF MEDICAL LABORATORY SCIENCES (BMLS)
3. BACHELOR OF RADIOTHERAPY TECHNOLOGY (BSCRTT)
4. BACHELOR OF NURSING (BSCN)
5. BACHELOR OF PHARMACY (BPHARM)
6. BACHELOR OF DENTAL SURGERY (DDS)

DATE: MONDAY, 12TH NOVEMBER 2012

TIME: 1HR 15 MIN (75 MIN)

INSTRUCTIONS

1. This paper consists of 50 multiple choice questions; answer all of them


2. For each question choose the most correct answer and write its corresponding
letter alongside its question number on a different ANSWER SHEET
provided.
3. Do not use the question paper for answering this examination
4. No penalty for any wrong answer
1. During a physical examination of the back, a along the frontal plane. What movement was
doctor palpated obliquely disposed bony impaired?
elevation in the shoulder extending to the A. Circumduction
posterior midline. What is this structure? B. Flexion
A. Coracoid process C. Extension
B. Acromion D. Abduction
C. Inferior scapular angle E. Adduction
D. C7 transverse process 7. The term vertebra prominens refers to which
E. Scapular spine cervical vertebra?
A. Atlas
2. From Qn.1 above, the doctor extends the B. Axis
examination fingers downward about finger- C. C3
breadth lateral to the midline to find a pointed D. C5
and mobile bony landmark. What is this E. C7
structure? 8. The distinguishing features of a typical cervical
A. Coracoid process vertebra include the following EXCEPT:
B. Acromion A. Triangular vertebra foramen
C. Inferior scapular angle B. Small vertebral body
D. C7 transverse process C. Long transverse process
E. Scapular spine D. Bifid spinous process
E. Horizontal articular surface
3. Any plane which is parallel to the median plane 9. At the age of 8 years, a boy developed signs of
is: dwarfism (short stature). A consulting physician
A. Coronal explained to the parent that the boy’s bones have
B. Medial stopped elongating which necessitated hormonal
C. Frontal therapy. From your anatomy knowledge, which
D. Sagittal part of the bone was most likely affected?
E. Transverse A. Periosteum
B. Metaphysis
4. The ancient anatomist commonly credited as the C. Epiphysis
father of modern anatomy is: D. Diaphysis
A. Vesalius E. Endosteum
B. Galen
C. Malphigi 10. A woman developed pain in her back region. A
D. Harvey CT scan showed a spinal nerve compression as it
E. Hippocrates exits from the intevertebral foramen between
5. In fowler’s position the patient is: two adjacent vertebrae with large bean shaped
A. Lying flat and facing down bodies. Which part of the vertebral column was
B. Lying flat and facing up affected?
C. Sitting and inclining forward A. Cervical
D. Sitting and inclining backward B. Thoracic
E. Lying on the side and tilted forward C. Lumbar
6. Following the motor traffic accident, a driver D. Sacral
was unable to move his arm away from the trunk E. Coccygeal
11. In anatomical terminology the term profundus D. Superficial radial
refers to what type of structures? E. Anterior interosseous
A. Deep 17. Which of the following muscles is not named
B. Superficial based on their shapes?
C. External A. Pyramidalis
D. Internal B. Trapezius
E. Outer C. Infraspinatus
D. Rhomboideus
12. The following dermatomes are found in the E. Deltoid
upper limb EXCEPT:
A. C4 18. A middle aged man experienced sharp pains in
B. C5 his left chest. A cardiologist attending him
C. C6 ordered an echocardiography (ultrasound picture
D. C8 of the heart). One of the heart valves with two
E. T1 cusps was found to be defective. What valve was
that?
13. The motor neurons: A. Right atrioventricular
A. Convey impulses to the CNS B. Aortic valve
B. Convey impulses to the effector organ C. Left atrioventricular
C. Are exclusively located in the CNS D. Pulmonary valve
D. Leads to numbness when injured E. Cardiac valve
E. Has their dendrites in peripheral organ
19. In her bedside learning a novice nursing student
14. Choose the most correct statement: was required to ascula tate (detect using
A. Nerves are synonymous to neurons stethoscope) the mitral valve beat. In which
B. Neurons are made nerves intercostals space and distance from the midline
C. Nerves are made of neurons is the beat normally detected?
D. Microglia support neurons A. 2nd , 3cm
E. Microglia support nerves B. 3rd ,5cm
C. 4th , 7cm
15. The ulnar nerve arises from which of the D. 5th , 9cm
following part of brachial plexus? E. 6th ,11cm
A. Medial cord
B. Lateral cord 20. From Qn.19 above, suppose the student as
C. Posterior cord required to detect the tricuspid valve sound. In
D. Superior branch which intercostals space and distance from the
E. Inferior branch midline would she detect the sound?
A. 2nd , 1cm
16. Numbness to the skin on the lateral half of the B. 3rd ,3cm
palm of the hand would be caused by a lesion to C. 4th , 5cm
which nerve? D. 5th , 1cm
A. Ulnar E. 6th , 3cm
B. Radial
C. Median
21. From Qns. 19& 20 above, what is the most B. Costal groove inferiorly
common clinical landmark for locating these C. Costal angle posteriorly
intercostals spaces? D. Intercostal muscular attachment
A. Clavicle E. Costochondral junction
B. Sterna angle 27. Which of the following is not a typical rib:
C. Jugular notch A. First
D. First rib B. Third
E. Base of the heart C. Fifth
D. Seventh
22. The pigmented part around the tip of the breast E. Ninth
is called: 28. Which of the following is not a true rib?
A. Breast proper A. First
B. Areola B. Third
C. Nipple C. Fifth
D. Upper quadrant D. Seventh
E. Lower quadrant E. Ninth
29. A young mother reported to the pediatrician
23. On a chest X-ray of a young boy presenting with complaining of her six monthly old baby’s
signs of lower respiratory tract infections, a fluid pulsating area/structure of the skull just above
filled-space was noted just above the right lobe the face. The pediatrician assured her that the
of the liver. What anatomical space was this pulsation was simply normal. What are the name
likely to be? and normal closure time (in months) of this
A. Superior mediastinum structure?
B. Posterior cardiac sinus A. Mastoid fontanelle, 12
C. Costodiaphragmatic recess B. Anterior fontanelle, 18
D. Inferior mediastinum C. Sphenoid fontanelle, 30
D. Posterior fontanelle, 36
24. The arterial supply to the heart is from the: E. Occipital fontanelle, 48
A. Coronary artery
B. Aorta 30. From Qn. 29 above, which bones are involved in
C. Left pulmonary artery the formation of the structure?
D. Right pulmonary artery A. Occipital and Temporal
E. Common carotid artery B. Temporal and frontal
C. Frontal and Parietal
25. The base of the heart is mainly formed by the: D. Parietal and temporal
A. Left atrium and partly right atrium E. Temporal and Sphenoid
B. Right atrium and right ventricle 31. Which of the following vessels can be felt as
C. Left ventricle pulsations just above the zygomatic arch and in
D. Left ventricle and left atrium front of the ear?
E. Left ventricle and right atrium A. Facial
B. Internal jugular vein
26. The following are the features of a typical rib C. Superficial temporal artery
EXCEPT: D. Retromandibular vein
A. Two articular faces E. Maxillary vein
32. The flat bones of the skull are joined by: D. Pia,dura,arachnoid
A. Diathroses E. Dura,arachnoid,pia
B. Saddle joints
C. Suture joints 38. Which of the following doesnot constitute the
D. Pivot joints muscles of mastication?
E. Hinge joints A. Masseter
B. Buccinator
33. All of the following paranasal sinuses drain into C. Temporalis
the middle nasal meatus, EXCEPT the: D. Medial pterygoid
A. Frontal E. Lateral pterygoid
B. Maxillary
C. Sphenoid 39. A young girl developed painful swollen masses
D. Anterior ethmoid in the pharyngeal wall and was feverish. The
E. Middle ethmoid doctor said that these were tonsils. Which of the
34. The following are the bones of the skull following doesnot constitute the tonsils?
EXCEPT: A. Lingual
A. Facial B. Tubal
B. Temporal C. Laryngeal
C. Parietal D. Palatine
D. Occipital E. Pharyngeal
E. Frontal 40. The thyroid gland can be examined in which of
the following triangles of the neck?
35. While administering an anesthetic, a dentist A. Submental
injected the drug in the medial aspect of the B. Glandular
mandible to block the nerve as it enters foramen. C. Carotid
What nerve is that? D. Submandibular
A. Mandibular E. Muscular
B. Inferior alveolar
C. Maxillary 41. The coronoid process belongs to which bone in
D. Lingual the head?
E. Facial A. Maxilla
B. Mandible
36. From Qn.35 above, what is the foramen in C. Sphenoid
question? D. Occipital
A. Mandibular E. Temporal
B. Mental
C. Alveolar 42. While drawing intravenous blood from a two
D. Lingual years old girl tried to use a large vein on the
E. Palatine outer aspect of the arm. What vein is that?
37. Arrange the brain meninges orderly from outside A. Median cubital
inwards: B. Basilic
A. Arachnoid,pia,dura C. Cephalic
B. Pia,arachnoid,dura D. Median antecubital
C. Dura,pia,arachnoid E. Axillary
43. From Q.42 above, the vein was not entirely D. Radial
detectable and hence a senior technologist E. Cephalic
recommended the use of a large vein in the neck.
What vein is that? 48. The upper limb is innervated by the branches of:
A. Internal jugular vein A. Lumber plexus
B. External jugular B. Brachial plexus
C. Common jugular C. Sacral plexus
D. Common carotid D. Cervical plexus
E. Internal carotid E. Thoracolumbar plexus

44. While vaccinating against tuberculosis a nurse 49. How many bones does the carpal joint contain?
injected the BCG vaccine in the skin over a flesh A. 4
muscle in the lateral aspect of the arm B. 8
proximally. What muscles is that? C. 6
A. Deltoid D. 5
B. Trapezius E. 10
C. Biceps brachii
D. Triceps brachii 50. The laxity of the capsule and disproportionate
E. Rhomboid articular surfaces renders the shoulder joint
unstable and therefore often liable to
45. While playing football, a boy fell on his elbow dislocations. Which of the following positions of
joint and seriously injured it. A radiograph the shoulder joint dislocation is commonest?
showed a fractured pointed proximal part of the A. Superior
ulna. What structure is this? B. Inferior
A. Olecranon C. Lateral
B. Styloid process D. Medial
C. Medial epicondyle E. Posterior
D. Lateral epicondyle
E. Ulnar head

46. A severely ill woman was rushed in the


emergency room and during triaging a nurse
student was instructed to take her pulse from an
artery in the forearm. Which artery did nurse
student use?
A. Brachial ---- END---
B. Axillary
C. Ulnar
D. Radial
E. Cephalic
47. From Q.46 above, the artery is a branch of;
A. Brachial
B. Axillary
C. Ulnar
COURSE: ………………………

UNIVERSITY OF DAR ES SALAAM


MUHIMBILI COLLEGE OF HEALTH SCIENCES
SCHOOL OF MEDICINE

DEPARTMENT OF ANATOMY/HISTOLOGY
ANATOMY TEST 1

Date: 16.10.2006 Time: 2.00 – 4.00


_____________________________________________________________________________________

SECTION I: MULTIPLE CHOICE QUESTIONS


Choose one most appropriate answer and CIRCLE the preceding letter (1 mark each)
NOTE: Quarter a mark will be deducted for each wrong answer
_____________________________________________________________________________________

1. In anatomical position the palms of the hands A. Cancellous bones


faces: B. Compact bone
A. Anteriorly C. Only bones that ossify in membranes
B. Posteriorly D. A and B
C. In the median plane E. A,B and C
D. Backwards 5. Examples of secondary cartilaginous joint are:
E. The gluteal region A. Symphysis pubis
2. The following statement (s) is /are TRUE: B. Intervertebral joints
A. the coronal plane is any vertical plane C. Epiphyseal cartilage
that passes through the body at right D. A and B
angles to the median plane E. A,B and C
B. the sagittal plane is any vertical plane 6. Examples of communicating intercellular
passing through the body parallel to junctions include:
the median plane A. Gap junctions
C. The term ventral means nearer to the B. Chemical synapse
median plane C. Electrical synapses
D. A and B D. A and B
E. A, B and C E. A,B and C
3. Find the CORRECT statement (s): 7. The thymus is identified by the presence of:
A. Sensory ganglia are associated with A. Hassals’ corpuscles
all spinal nerves B. Large number of macrophages
B. Autonomic ganglia contains pre and C. Numerous lymphocyte
post-ganglionic fibers D. Presence of sinusoids
C. The sensory roots of the spinal nerves E. Lymphatic nodules
are examples of pseudounipolar 8. The tonsils include:
neurons A. Palatine
D. A and B B. Lingual
E. A, B and C C. Tubal
4. The following types of bone are associated D. Pharyngeal
with bone marrow E. All of the above
9. The following cells show phagocytosis: 14. The cell organelle(s) that have a high
A. Macrophages rate of oxygen utilization are/is:
B. Neutrophils A. Peroxisomes
C. Monocytes B. Microtubules
D. A and C C. Centrioles
E. A,B and C D. Lysosomes
E. A and C
10. The connexins are found in the 15. Transport vesicles are associated with
following cell junctions the following cell organelles:
A. Tight junctions A. Rough endoplasmic reticulum
B. Desmosomes B. Golgi apparatus
C. Gap junctions C. Peroxisomes
D. Septate junctions D. A and B
E. Spot desmosomes E. A,B and C
16. The trachea is lined by the following
11. Regarding the bone find the CORRECT cells:
statement(s): A. Ciliated pseudo-stratified cells
A. It is surrounded by a fibrous B. Mucus secreting goblet cells
tissue known as the periosteum C. Stratified squamous cells
B. Calcium salts are deposited in D. A and B
the intercellular matrix E. A, B and C
C. Red bone marrow contains 17. Which of the following is NOT a variety
erthyroblasts, myeloblasts and of synovial joint?
megakaryoblasts A. Saddle
D. A and B B. Ellipsoid
E. A,B and C C. Pivot
D. Syndesmosis
12. The following cells are found in the E. Condyloid
bone EXCEPT: 18. Regarding the lungs surfactant is
A. Osteocytes secreted by the following cells:
B. Chondrocytes A. type I alveolar pneumocytes
C. Osteoclasts B. type II alveolar pneumocytes
D. Osteoblasts C. alveolar macrophages
E. Osteoprogenitor cels D. A and C
E. None of the above
13. Which of the following is NOT a long 19. The following parts of the digestive tract
bone? contains the mucus secreting
A. Radius submucosal glands
B. Ulna A. Stomach
C. Scaphoid B. Oesophagus
D. Metatarsal C. Duodenum
E. Middle phalanx D. A and C
E. B and C
20. Kupffer cells are found in the following 25. The parafollicular cells are found in the :
organ: A. Thyroid gland
A. Kidney B. Parathyroid gland
B. Associated with the liver C. Thymus
sinusoids D. A and B
C. In the subcortical spaces of the E. A,B and C
lymph nodes
D. Thymus 26. Regarding the thyroid follicles the
E. B and C following statement(s) is/are TRUE:
A. the cells are examples of
21. Regarding the urinary system: stratified epithelium
A. The nephron is the functional B. the luminal surfaces of cells are
units of the kidney in contact with the colloid
B. The renal corpuscles are C. their shape varies according to
supplied by afferent arterioles the activity of the gland
C. Filtration takes place at the D. A and B
glomerulus E. A,B and C
D. A, B and C
E. A and C 27. The white pulp of the spleen is formed
by the:
22. Simple squamous epithelium lines the: A. Periarterial lymphatic sheath
A. Bowman’s capsule B. Splenic nodules
B. Stomach C. Venous sinuses
C. Body cavities D. A and B
D. A and B E. A,B and C
E. A and C
28. Regarding the spleen the following
23. Most parts of the esophagus are lined by statement(s) is/are TRUE:
the: A. It is surrounded by a capsule
A. stratified squamous epithelium that contains smooth muscle
B. simple squamous epithelium cells
C. pseudostratified epithelium B. The central artery is actually
D. simple cuboidal epithelium eccentric in position
E. none of the above C. The stroma contains reticular
and collagen fibers
24. The presence of the following lymphoid D. A and B
nodule may indicate infective process: E. A,B and C
A. Secondary lymphoid nodule
B. Primary lymphoid nodule 29. The urinary bladder is lined by the:
C. Both primary and secondary A. Stratified squamous cells
lymphoid nodules B. Transitional cells
D. A and B C. Simple columnar cells
E. A,B and C D. Pseudo-stratified columnar cells
E. Stratified columnar cells
30. The ureters receive urine from the: D. Tertiary bronchi supply the
A. Renal pelvis bronchopulmonary segments
B. Major calyces E. All of the above
C. Minor calyces 35. Regarding the lungs the following
D. Renal sinus statement(s) is/are TRUE:
E. Both the major and minor A. Air exchange is possible at the
calyces bronchiole segments
B. The bronchi do not have goblet
31. The trachea: cells in their walls
A. is continuous above with the C. The smooth muscles in the walls
cricoid cartilage of the bronchioles are supplied
B. is lined by respiratory by the autonomic nerves
epithelium D. A and B
C. has C-shaped cartilages E. A,B and C
D. is related posteriorly to the 36. The hormones that are released by the
oesophagus neurohypophysis are:
E. all of the above A. Oxytocin
B. ADH
32. Regarding the heart: C. Prolactin
A. Its muscle wall is made up of D. A and B
the myocardial muscles E. A,B and C
B. The muscles of the heart are 37. The Rathkes pouch give rise to the
supplied by the autonomic following part of the pituitary gland:
nerves A. Adenohypophysis
C. The muscles of the heart are of B. Neurohypophysis
striated type C. Pars nevosa
D. A and B D. Infundibulum
E. A,B and C E. A and B
38. Aldosterone is secreted by the cells of
33. The membrane that encloses the heart is the:
called: A. Zona glomerulosa
A. Endocardium B. Zona fasciculata
B. Mediastinum C. Zona reticularis
C. Peritoneum D. Both zona reticularis and the
D. Pericardium medulla
E. None of the above E. None of the above
34. Regarding the respiratory system find 39. Regarding the suprarenal glands the
the CORRECT statement(s) : capsule is situated closer to the
A. Primary bronchi are continuous following part:
above with the larynx A. Zona glomerulosa
B. Tertiary bronchi have also C- B. Medulla
shaped cartilage C. Zona reticularis
C. In each lung there are three D. Zona fasciculata
secondary bronchi E. None of the above
40. Regarding the digestive tract the tunica 45. The cell organelle that play a major role
mucosa has the following components: in intracellular transport is:
A. Lining epithelium A. Microtubules
B. Lamina propria B. Microfilaments
C. Muscularis mucosa C. Intermediate filaments
D. A and B only D. Desmin
E. A,B and C E. None of the above
46. The cell cortex is formed by the:
41. Regarding the digestive tract the A. Actin filaments
Auerbach’s autonomic plexuses are B. Intermediate filaments
located in the: C. Microtubules
A. tunica mucosa D. A,B and C
B. tunica submucosa E. B and C
C. tunica muscularis 47. The muscular walls of the oesophagus
D. A and B contains:
E. B and C A. Striated muscles in the upper
third
42. Find the CORRECT statement(s): B. Smooth muscles cells in the
A. The stroma of the thymus distal part that enters the
doesnot have reticular fibers stomach
B. The red pulp of the spleen C. A mixture of smooth and
contains many red blood cells skeletal muscles in the lower
compared to the white pulp third
C. The growth of the thymus D. A and B
reaches maximum at puberty E. A,B and C
D. A and B 48. The anal canal is lined by the:
E. A,B and C A. Stratified squamous cells
B. Simple columnar cells
43. The intercalated discs are associated C. Simple squamous cells
with: D. A and B
A. Smooth muscles cells E. A,B and C
B. Skeletal muscles 49. Peyer’s patches are characteristic to the:
C. Cardiac muscles cells A. Ileum
D. All types of striated muscles B. Vermiform appendix
cells C. Colon
E. A and B D. Jejunum
44. Multi- nucleation is a feature of the E. Duodenum
following muscle cell(s): 50. The cell organelle that contains the
A. Skeletal circular DNA is the:
B. Striated A. Mitochondria
C. Smooth B. Endoplasmic reticulum
D. Cardiac C. Golgi apparatus
E. A and C D. Peroxisomes
E. Nucleus
51. Define the following terms (6 Marks)
a) Nuclear envelope
b) Endocytosis
c) Brunner’s glands
52. Write short notes on the following:
i. Histology of the tongue (15 marks)

ii. Juxtaglomerular apparatus (9 marks)

iii. Plasma membrane (10 marks)

53. Regarding the abdominal hernias, the following statement(s) is/are TRUE:
A. Indirect inguinal hernia enters the inguinal canal through the superficial inguinal ring
B. Direct inguinal hernia lies lateral to the inferior epigastric artery
C. indirect inguinal hernia lies medial to the pubic tubercle
D. A and C
E. None of the above
54. Find the TRUE statement:
A. Lymphatic drainage of the testes is likely to go to the superficial inguinal nodes
B. Hydrocele refers to the accumulation of fluid in tunica vaginalis
C. Traumatic injury to the testes can cause destruction of the seminiferous tubules
D. A and C
E. B and C

55. The portal vein is formed by the union of the following veins:
A. Superior mesenteric and splenic
B. Inferior mesenteric and splenic
C. Is a continuation of splenic vein
D. Superior mesenteric and inferior mesenteric
E. None of the above

56. The following is/are retroperitoneal organs:


A. Spleen
B. Jejunum
C. Suprarenal glands
D. Stomach
E. None of the above
57. Regarding the Meckel’s diverticulum find the CORRECT statement(s):
A. If present is associated with ileum
B. If inflamed can mimic acute appendicitis
C. Occurs in 2% of subjects
D. A and C
E. A,B and C
58. The commonest site of location of the vermiform appendix is :
A. Retrocaecal
B. Paracaecal
C. Paraileal
D. Pelvic
E. Ileal
59. Find the CORRECT statement(s):
A. The proximal 2/3 of the transverse colon is supplied by the left colic artery
B. Descending colon is innervated by the vagus nerve
C. Ascending colon is supplied by the middle colic artery
D. A and B
E. None of the above
MUHIMBILI UNIVERSITY OF HEALTH AND ALLIED SCIENCES
SCHOOL OF MEDICINE
DEPARTMENT OF ANATOMY AND HISTOLOGY
ANATOMY TEST NO. 1 FIRST YEAR MD & DDS 2009/2010
NAME: ………………………………… DEGREE PROGRAM: …………………………
DATE: 23RD NOVEMBER 2009 TIME: 2HRS

INSTRUCTIONS:
1. This paper consists of 100 questions in two sections; answer all of them. Each worth one mark
2. For each question select the most correct answer and write its corresponding letter on a special
Answer sheet provided.
3. In section A, quarter (1/4) a mark will be deducted for every wrong answer
_____________________________________________________________________________________

1. Any plane is parallel to the median 5. Mast cells secrete these allergic
plane is: mediators EXCEPT:
A. Coronal A. Histamine
B. Frontal B. Inteleukin-1
C. Sagittal C. Heparin
D. Transverse D. SRS-A
E. None of the above E. ECF-A
2. Examples of immovable joints include
the following EXCEPT: 6. Connective tissue fibres include the
A. Sutures following EXCEPT:
B. Gomphoses A. collagen fibers
C. Ellipsoid joints B. keratin fibers
D. B and C C. elastic fibers
E. None of the above D. reticular fibers
3. The following factor(s) is/are important E. none of the above
in joint stability:
A. Ligaments 7. Simple squamous epithelia are typically
B. Muscle tone found in these locations EXCEPT:
C. Shape of articulating surface A. Bowman’s capsule
D. A and B B. Endothelium
E. A,B and C C. Epidermis
4. Regarding muscles generally: D. Alveoli
A. There are skeletal, smooth and E. Mesothelium
cardiac muscles
B. The insertion is mobile than the 8. Transitional epithelium is found in the:
origin A. Uterus
C. They can be named based on B. Vagina
their shape C. Urinary bladder
D. A and B D. Trachea
E. A,B and C E. None of the above
9. Regarding the muscular arteries; choose the A. Body
most correct option(s): B. Transverse foramen
A. They are also known as conducting C. Vertebral foramen
arteries D. Spinous process
B. Tunica media is made of smooth E. Transverse process
muscle
C. Internal elastic lamina is absent 15. Which of the following is the correct
D. External elastic lamina is prominent arrangement of the brachial plexus after they
E. All of the above arise as C5-T1 anterior roots?
A. Trunks,branches, divisions and cords
10. The following cells are associated with the B. Divisions,trunks, cords and branches
bone tissue: C. Trunks,cords,branches and divisions
A. Osteocytes D. Trunks, divisions, cords and branches
B. Histiocytes E. Divisions, branches, cords and
C. Chondrocytes branches
D. Oligodendrocytes
E. All of the above 16. The latissimus dorsi muscle is innervated by:
A. Long thoracic nerve
11. Examples of second messengers include the B. Thoracodorsal nerve
following EXCEPT the: C. Dorsoscapular nerve
A. Calmodulin D. Axillary nerve
B. Adenylate cyclase E. Lateral pectoral nerve
C. Cyclic AMP
D. Calcium ion 17. Regarding the ribs:
E. None of the above A. The 1st to 10th are true ribs
B. The 8th – 10th are false ribs
12. Which of the following receptors are NOT C. The 2nd rib is both a true and a typical
coupled to a G-protein? rib
A. Purinergic P2Y D. The last five ribs are floating ribs
B. Adrenergic receptors E. B and C
C. Muscarinic receptors
D. Androgen receptors 18. The posterior mediastinum:
E. Dopaminergic receptors A. Is bound by the vertebral bodies
posteriorly
13. Bones that make the distal row of the carpal B. Is bound by the heart anteriorly
joint include the following: C. Contains the ascending and descending
A. Triquetrum aorta
B. Trapezium D. A and B
C. Trapezoid E. B and C
D. Hammate
E. None of the above 19. Which of the following is true regarding the
thoracic duct?
14. A typical vertebra contains the following A. Receives lymph from all parts of the
EXCEPT: body
B. Drains into the left brachiocephalic 24. The arterial blood pressure is commonly taken
vein from:
C. Ascends to the right of the ascending A. Subclavian artery
aorta B. Brachial artery
D. A and B C. Axillary artery
E. B and C D. Radial artery
E. Aorta
20. When compared to its left counterpart, the
right main bronchus is comparatively: 25. The following nerves originate from the
A. Wider posterior cord of the brachial plexus EXCEPT
B. Shorter the:
C. More vertical A. Axillary
D. A and C B. Radial
E. A,B and C C. Suprascapular
D. Upper subscapular
21. Which is the correct order as blood enters and E. Lower subscapular
flows through the heart? (RV= Right ventricle,
RA= Right Atrium, LV= Left Ventricle, LA= 26. All of the following muscles are attached to the
Left atrium): medial border of the scapular EXCEPT the:
A. RA,LA,RV,LV A. Trapezius
B. LA,RA,LV,RV B. Rhomboid major
C. LA,RA,RV,LV C. Rhomboid minor
D. RA,RV,LA,LV D. Serratus anterior
E. LA,LV,RA,RV E. A and D

22. All of the following structures are found in the 27. Regarding the axilla:
RA EXCEPT: A. It is located between the arm and the
A. Crista terminalis thoracic wall
B. Septomarginal trabeculae B. The axillary artery is an important
C. Musculi pectinati content of the axilla
D. Bossa Ovalis C. Coracobrachialis and serratus lateralis
E. None of the above forms its anterior wall
D. A and B
23. Regarding the pericardium, choose FALSE E. A,B and C
statement(s):
A. It is also called epicardium 28. These are bones of the axial skeleton
B. The fibrous part is the most exterior EXCEPT:
C. The pericardial space lies deeper to the A. Scapula
parietal serous pericardium B. Skull
D. Serous pericardium secretes pericardial C. Vertebrae
fluid D. Ribs
E. None of the above E. Sternum
29. Which of the following muscles are NOT C. It is a component of anterior axillary
innervated by the musculo-cutaneous nerve? wall
A. Coracobrachialis D. It is attached to the under surface of
B. Brachioradialis the clavicle
C. Brachialis E. None of the above
D. Biceps brachii
E. None of the above 34. The following are branches of the axillary
artery EXCEPT:
30. Injury to the long thoracic nerve will result A. The anterior circumflex humeral artery
into: B. The internal thoracic artery
A. Waiter’s tip C. The lateral thoracic artery
B. Winging of the shoulder D. The subscapular artery
C. Paralysis of the latissimus dordi E. None of the above
muscle
D. A and B 35. The axillary artery:
E. B and C A. Is a continuation of the subclavian
artery at the lower border of the
31. The right lung presents impressions for all of clavicle
the following structures EXCEPT the: B. Is a continuation of the subclavian
A. Superior vena cava artery at the lateral border of the first
B. Aorta rib
C. Brachiocephalic vein C. Is divided into three parts by the teres
D. Trachea major muscle
E. Right common carotid artery D. Continues as the brachial artery at the
lower border of the teres minor muscle
32. Regarding the brachial artery find the FALSE E. All of the above
statement (s):
A. It gives a nutrient artery to the 36. Regarding the muscles of the arm:
humerus A. The long head of the triceps originates
B. It gives the superior ulnar collateral from the infraglenoid tubercle
artery to the arterial anastomosis B. The long head of the biceps brachii
around the elbow joint originates from the coracoid process
C. It gives the interosseous recurrent C. The biceps brachii muscle is a strong
artery to the anastomosis at the elbow supinator of the forearm
joint D. A and B
D. It gives off the profunda artery in the E. A and C
upper arm
E. None of the above 37. Triceps brachii muscle is supplied by the
following:
33. Regarding the clavipectoral fascia the A. Axillary nerve
following statements are TRUE except: B. Musculocutaneous and radial nerves
A. It encloses the pectoralis major muscle C. Radial nerves
B. Below the pectoralis minor it forms the D. Dorsal scapular nerve
suspensory ligament of the axilla E. Long thoracic nerve
38. Regarding the carpal tunnels: C. The pectoralis minor is supplied both
A. It’s bound by the extensor retinaculum by medial and lateral pectoral nerves
dorsally D. A and B
B. Compression in the tunnel my result E. A,B and C
into failure in the oppositional
movement of the thumb 43. Regarding the muscles of the back:
C. Compression in the tunnel my result to A. The trapezius is supplied by the
loss of sensation to the palmer surface accessory nerve
of the lateral one and half fingers B. The trapezius is important in elevating
D. A and B the arm above 900
E. B and C C. The middle fibers of the trapezius pulls
scapular medially
39. The axillary nerve supplies the following: D. The levator scapulae muscle elevates
A. Teres minor the scapula
B. Deltoid E. All of the above
C. Trapezius
D. A and B 44. Biceps brachii muscle is supplied by:
E. A,B and C A. Radial nerve
B. Musculocutaneous nerve
40. The median nerve supplies the following C. Axillary nerve
muscles EXCEPT: D. Upper and lower subscapular nerve
A. Pronator teres E. B and C
B. Flexor digitorum superficialis
C. Flexor carpi radialis 45. Rotator cuff muscles includes of the following:
D. Flexor carpi ulnaris A. Subscapularis
E. C and D B. Supraspinatus
C. Infraspinatus
41. Regarding the muscles of the extensor D. Teres minor
compartment of the forearm: E. Teres major
A. The extensor pollicis brevis extends the
metacarpophalangeal joint of the thumb 46. Which of the following muscles does not take
B. Extensor carpi radialis brevis extends the their origin from the lateral epicondyle of the
wrist joint humerus?
C. They are supplied by the radial nerve and A. Supinator
its branches B. Extensor digitorum
D. A and B C. Extensor pollicis longus
E. A,B and C D. Extensor carpi ulnaris
E. Anconeus
42. Regarding the pectoral muscles:
A. The pectoralis major is supplied by the 47. Regarding the palm:
lateral pectoral nerve A. The radial nerve innervates the skin of
B. The pectoralis major adducts and the dorsum lateral one and a half
flexes the arm and rotates it medially fingers
B. The ulnar nerve innervates the skin of A. Anatomical end artery anastomose
the dorsum medial one and a half with an artery supplying adjacent areas
fingers B. A functional end artery communicates
C. The skin of the dorsum of the palm is with an insufficient anastomosing
innervated by the median nerve branch
D. The palmar interosseous muscles are C. Most veins have valves on their
innervated by the median nerve luminal walls
E. All of the above D. A and B
E. B and C
48. The following are possible movements of the
thumb: 52. The following describes a body in anatomical
A. Opposition moves the thumb towards position EXCEPT:
the index finger A. Heels together
B. Abduction moves the thumb away B. Palms facing anteriorly
from the index finger C. Elbow joints extended
C. Flexion moves the thumb towards the D. Fingers slightly flexed
palm E. None of the above
D. A,B and C
E. B and C 53. Regarding the cephalic vein:
A. It begins at the medial aspect of the
49. Regarding the “Anatomical Snuffbox” the dorsal venous arch
following statements are TRUE: B. It has no valves
A. The medial wall is formed by the C. It joins the brachial vein
tendon of extensor pollicis longus D. It pierces the clavipectoral fascia
B. The lateral wall is occupied by the E. It receives the basilic vein
tendons of extensor pollicis brevis and
abductor pollicis longus 54. Regarding the prokaryotic cells the following
C. Pulsation of the radial artery can be statements are true EXCEPT:
felt here A. They have no nuclei
D. A and C B. They are devoid of the plasma
E. A,B and C membrane
C. They have very few membrane bound
50. Find an INCORRECT statement: organelles
A. The spinal cord is part of central D. Examples are bacteria
nervous system E. None of the above
B. All spinal nerves contains both sensory
and motor fibers 55. Membranes bound organelles include the
C. Myelination of the peripheral nerves is following:
done by Schwann cells A. Mitochondria
D. The pia matter lie deeper to the B. Ribosomes
arachnoid C. Cytoskeleton
E. None of the above D. Nucleolus
E. Centrioles
51. Find the CORRECT statement(s):
56. The lysosomes: 61. Numbness to the skin on the lateral half of the
A. Contains different kinds of hydrolytic palm of the hand would be caused by a lesion
enzymes for intracellular digestion to which nerve?
B. Are prominent in phagocytic cells such A. Ulnar
as macrophages B. Radial
C. Forms from smooth endoplasmic C. Median
reticulum D. Superficial radial
D. A and B E. Anterior interosseous
E. A,B and C
62. The nerve most likely to be injured in fractures
57. Intracellular calcium storage organelles of the medial epicondyle is the:
include: A. Radial
A. Peroxisome B. Axillary
B. Mitochondria C. Ulnar
C. Endoplasmic reticulum D. Median
D. A and B E. Musculocutaneous
E. B and C
63. All of the following are parts of the parietal
58. Examples of endocytosis include: pleura EXCEPT:
A. Exocytosis A. Cervical
B. Phagocytosis B. Costal
C. Pinocytosis C. Pulmonary
D. A and B D. Diaphragmatic
E. B and C E. Mediastinal
59. Regarding the smooth endoplasmic reticulum,
the following statements are true EXCEPT: 64. The trachea bifurcates into right and left
A. Is a site for protein synthesis primary bronchi at the level of:
B. They become prominent in liver cells A. Plane of sternal angle
that perform detoxification of B. Suprasternal (or jugular) notch
substances such as alcohol C. First rib
C. Are involved in the production of D. Seventh cervical vertebra
platelets from megakaryocytes E. Xiphoid process
D. May be associated with the nuclear
membrane 65. Increased resistance to pulmonary blood flow
E. Are prominent in steroid secreting in the lungs would cause a direct stain on
cells such as cells of the adrenal cortex which chamber of the heart?
A. Right ventricle
60. Which of the following muscles does NOT B. Left atrium
rotate the arm medially? C. Right atrium
A. Subscapularis D. Left ventricle
B. Supraspinatus E. None of the above
C. Latissimus dorsi
D. Pectoralis major
E. Anterior fibers of the deltoid
66. During surgical repair of a patent ductus A. it is small and circular
arteriosus, the surgeon must be careful not to B. has no histones
injure the: C. is found in the matrix
A. Right recurrent laryngeal nerve D. it is double stranded
B. Left phrenic nerve E. none of the above
C. Left recurrent laryngeal nerve
D. Vagus nerve 72. The cytoskeleton include:
E. Right phrenic nerve A. Actin filaments
B. Microtubules
67. The root of the lung contain the following C. Neurofilaments
EXCEPT: D. A and B
A. Bronchus E. A,B and C
B. Vagus nerve
C. Pulmonary ligament 73. Microtubules constitutes the following
D. Pulmonary artery structures EXCEPT:
E. Pulmonary vein A. Centrioles
B. Cilia
68. The left bronchial arteries frequently take C. Flagella
origin from the: D. Microvilli
A. Internal thoracic artery E. None of the above
B. Descending aorta
C. Aortic arch 74. Examples of proteins that forms the
D. Subclavian artery intermediate filaments include the following
E. None of the above EXCEPT:
A. Nuclear lamins
69. Bone can be classified based on their: B. Actin
A. Development C. Vimentin
B. Shape D. Desmin
C. Region E. None of the above
D. A and B
E. A,B and C 75. Regarding the nucleolus, find the CORRECT
statement(s):
70. Regarding the Golgi apparatus. Choose a A. Become prominent in protein secreting
FALSE statement (s): cells
A. Contains the trans and cis part B. Is associated with chromosomes that
B. Tha trans part receives vesicles from code for rRNA
the ER C. Produces chromatin
C. Is an intracellular site for synthesis of D. A and B
carbohydrate E. A,B and C
D. B and C
E. None of the above 76. Which of the following statement(s) is/are
FALSE?
71. The following are true regarding the A. Perimysium sorrounds the whole
mitochondrial DNA EXCEPT: muscle
B. Epimysium sorrounds the whole D. Microglia
muscle E. None of the above
C. An antagonist muscle counteracts the
agonist 82. The aortic valves are:
D. A motor unit comprises of a neuron A. Anterior and posterior
and the muscle fiber it supplies B. Anterior, posterior and septal
E. None of the above C. Left, right and anterior
D. Left, right and posterior
77. The prime mover in flexion of the elbow is: E. None of the above
A. Biceps brachii
B. Brachioradialis 83. White muscle fibers:
C. Brachialis A. Contracts faster
D. Triceps brachii B. Are metabolically aerobic
E. None of the above C. Easily fatigue
D. A and C
78. The main branches of the left coronary artery E. A,B and C
are:
A. Anterior interventricular artery 84. Muscle functions include:
B. Posterior interventricular artery A. Movement
C. Circumflex artery B. Posture maintainance
D. A and B C. Joint stabilization
E. A and C D. Heat production
E. All of the above
79. The coronary sinus:
A. Is the continuation of the great cardiac 85. The internal thoracic artery:
vein A. Originates from the axillary artery
B. Empties into the right atrium B. Descends anterior of the posterior
C. Also receives blood from the anterior thoracic walls
cardiac vein C. Divides into the superior epigastric and
D. A and B musculophrenic arteries
E. A,B and C D. A and B
E. A and C
80. Normally the right lung has how many lobes?
A. 1 86. The left recurrent laryngeal nerve:
B. 2 A. Stems from the left vagus
C. 3 B. Forms part of the pericardial plexus
D. 4 C. Hooks around the pulmonary trunk
E. 5 D. Also innervates the left root of the lung
E. All of the above
81. The cell of the CNS includes all of the
following EXCEPT: 87. The azygous vein:
A. Neurons A. Enters thorax through the aortic hiatus
B. Nerves B. Is related to the ascending aorta on its
C. Astrocytes left side
C. Drains the anterior and posterior
thoracic walls
D. Empties directly into the right atrium
E. All of the above

88. The external intercostals muscles:


A. Originate from the superior costal
margin
B. Origin from the inferior costal margins
C. Has its fibers directed downward and
laterally
D. A and B
E. B and C

89. Which of the following passes through tha


scapular notch?
A. Subscapular nerve
B. Subscapular artery
C. Suprascapular nerve
D. Suprascapular artery
E. Long thoracic nerve

90. Electrical signals of the heart moves in the


following order:
A. AV node, Purkinje fibers, SA node,
Bundle of His
B. AV node, SA node , Bundle of His
Purkinje fibers
C. AV node, Bundle of His, SA node
,Purkinje fibers
D. SA node, AV node, Bundle of His
,Purkinje fibers
E. SA node, Purkinje fibers, AV node,
Bundle His
SECTION II: MATCHING

For questions 91-100, choose a letter from Column B that matches the question numbers in Column A. One letter from
Column B may match one only number in column A and viceversa

COLUMN B

COLUMN A A. Median nerve


B. Ulnar nerve
91. …… Teres minor C. Long thoracic nerve
92. …… Rhomboid major D. Thoracodorsal nerve
93. …… Serratus anterior E. Dorsal scapular nerve
94. ……3rd & 4th lumbricals F. Medial and lateral pectoral nerve
95. ……Opponens pollicis G. Axillary nerve
96. ……Peroxisomes H. Lower subscapular nerve
97. …… Glial fibrillary acidic protein I. Cell secretion
98. …… Transcription J. RNA polymerase
99. …… Exocytosis K. Lysosomes
100. ……Tay sachs disease L. Actin filaments
M. DNA polymerase
N. Nucleic acid proteins
O. H2O2 metabolism
P. Intermediate filaments

-------SUCCESS-------
MUHIMBILI UNIVERSITY OF HEALTH AND ALLIED SCIENCES
SCHOOL OF MEDICINE
DEPARTMENT OF ANATOMY AND HISTOLOGY
MD 1 CAT 3 ANATOMY EXAMINATION
TIME: 1HR 30 MIN DATE: FEBRUARY 16, 2010

GENERAL INSTRUCTIONS:
This paper consists of 75 MCQ questions; answer all them. Choose the most correct answer
and write its corresponding letter alongside its question number on the ANSWER SHEET
provided.

1. The anterior abdominal wall is 5. Which of the following structure is


supplied by these arteries EXCEPT: retroperitoneal?
A. Superior epigastric arteries A. Descending colon
B. Inferior epigastric arteries B. Transverse colon
C. Posterior intercostals arteries C. Spleen
D. Lumbar arteries D. Ileum
E. None of the above E. Jejunum

2. Which of the following anterior 6. The broad ligament of the uterus is a


abdominal wall muscles is the double layer of peritoneum which
deepest to the others? encloses all of the following
A. Rectus abdominis EXCEPT:
B. Pyramidalis A. Ureter
C. Transversus abdominis B. Ovarian ligament
D. Internal oblique C. Uterine tube
E. External oblique D. Round ligament
E. Uterine artery
3. The muscles of the posterior
abdominal wall include the following 7. The internal iliac artery give rise to
EXCEPT: all of the following branches
A. Psoas major EXCEPT the:
B. Psoas minor A. Superior gluteal
C. Semispinalis thoracis B. Ovarian
D. Quadratus lumborum C. Superior vesicle
E. None of the above D. Internal pudendal nerve
E. Middle rectal
4. The pelvic diaphragm is formed by
the following: 8. The venous drainage of the liver is
A. Levator ani muscle through:
B. Coccygeous muscle A. Portal vein
C. Pelvic fascia B. Superior mesenteric vein
D. A and B C. Hepatic vein
E. A,B and C D. Inferior vena cava
E. Gastrosplenic veins
9. The anterior relation of the left C. The external oblique
kidney includes the following aponeurosis make its
EXCEPT: superficial inguinal ring
A. Spleen D. A and B
B. Pancreas E. A,B and C
C. Stomach 15. The abdomen is clinically divided
D. Duodenum into:
E. Splenic vessels A. Nine regions
10. The ischiorectal fossa contains the: B. Six regions
A. Pudendal nerve C. Four quadrants
B. Internal pudendal artery D. A and C
C. Internal pudendal vein E. B and C
D. A and C 16. The portal vein is formed by the:
E. A,B and C A. Right gastroepiploic and
11. The bisection of the kidney may superior mesenteric veins
show the following: B. Superior mesenteric and
A. Renal capsule splenic veins
B. Renal cortex C. Inferior and superior
C. Renal medulla mesenteric veins
D. Renal calyx D. Hepatic and splenic veins
E. All of the above E. Superior and inferior
12. The main distinctive features of the pancreaticoduodenal veins
large intestine are: 17. The following are the branches of
A. Haustra coli the superior mesenteric artery
B. Appendices epiploicae EXCEPT the:
C. Taenia coli A. Middle and colic artery
D. A and C B. Superior left colic artery
E. A,B and C C. Right colic aretery
13. The epiploic foramen opens into the D. Ileocolic artery
lesser omental sac, its boundaries E. inferior pancreatic duodenal
are formed by all of these EXCEPT: artery
A. Free border of the greater 18. Parts of the stomach includes:
omentum A. Cardia and fundus
B. Inferior vena cava B. Body
C. Caudate lobe of the liver C. Antrum
D. Duodenum D. Pylorus
E. None of the above E. All of the above
14. The inguinal canal: 19. The stomach is supplied by the
A. Transmit the spermatic cord following arteries EXCEPT:
in the male A. Left gastric artery
B. Is associated with the indirect B. Right gastric artery
inguinal hernia C. Left gastroepiploic artery
D. Right gastroepiploic artery
E. None of the above
20. The esophageal opening in the D. Sub-cecal
diaphragm lies at the vertebral level E. Sub-colic
of: 26. The common bile duct and major
A. T8 pancreatic duct join to form the:
B. T10 A. Ampulla of vater
C. T11 B. Common hepatic duct
D. T12 C. Cystic duct
E. L1 D. Cistern chyli
21. The uncinate process of the E. Hepatic duct
pancreas lies: 27. The prostate gland lies:
A. Within the lienorenal A. Within the urogenital
ligament diaphragm
B. In front of the portal vein B. At the neck of the bladder
C. Behind the superior above the pelvic diaphragm
mesenteric artery C. At the neck of the bladder
D. B and C below the pelvic diaphragm
E. A,B and C D. In the superficial perineal
22. The gall bladder normally gets its space
blood supply from the: E. In the retrovesical space
A. Celiac artery 28. The lateral wall of the ischiorectal
B. Splenic artery fossa is the:
C. Gastroduodenal artery A. Levator ani muscle
D. Branch of right of hepatic B. External anal sphincter
artery C. Internal anal sphincter
E. Right gastric artery D. Obturator internus muscle
23. The greater omentum is attached to E. Obturator externus muscle
the: 29. Ligaments of the liver includes:
A. Liver and stomach A. Falsiform
B. Stomach and jejunum B. Ligamentum teres
C. Stomach and colon C. Coronary ligament
D. Liver and colon D. Ligament venosum
E. Jejunum and colon E. All of the above
24. The distal transverse colon gets its 30. Regarding the spleen the following
blood supply from: are true EXCEPT:
A. Celiac trunk A. It is located in the left
B. Superior mesenteric artery hypochondrium
C. Inferior mesenteric artery B. It has visceral and parietal
D. B and C surfaces
E. None of the above C. Splenic vessels
25. The location of vermiform appendix enters/leaves it at the hillum
is commonly: D. Splenectomy is compatible
A. Retro-ileal with life
B. Retro-cecal E. None of the above
C. Sub-ileal 31. The diencephalons is a part of the:
A. Hindbrain D. Touch
B. Midbrain E. Proprioception
C. Forebrain 38. A lesion to the postcentral gyrus of
D. Cerebral hemisphere one cerebral hemisphere is likely to
E. None of the above lead to:
32. The nervous system includes: A. Ipsilateral paralysis
A. Brain B. Contralateral anaesthesia
B. Spinal cord C. Aphasia
C. Peripheral nerves D. Ipsilateral anaesthesia
D. A,B and C E. Contalateral paralysis
E. A and B only 39. Regarding the cerebrospinal fluid:
33. The medulla oblongata is externally A. It’s formed by the arachnoid
made of: granulations
A. The pons B. It enters the 3rd ventricle
B. The pyramids through the cerebral
C. The olives aqueduct
D. A and B C. It enters the subdural space
E. B and C through foramina Magendie
34. Distally the spinal cord terminates at and Luschka
the level of: D. A,B and C
A. T12 – L1 E. None of the above
B. L1-L2 40. The superior colliculi are associated
C. L2-L3 with:
D. L3-L4 A. Superior quadrigeminal
E. L4-L5 brachium
35. Which of the following doesnot B. Lateral geniculate body
consist part of the midbrain? C. Medial geniculate body
A. Tegmentum D. A and B
B. Substantia gelatinosa E. A and C
C. Substantia nigra 41. Hypoglossal nucleus is located in
D. Red nucleus the:
E. Corpora quadrigemina A. Pons
36. The basal nuclei include all of the B. Midbrain
following EXCEPT: C. Medulla oblongata
A. Putamen D. Upper part of the spinal cord
B. Caudate E. Diencephalon
C. Thalamus 42. Deep cerebellar nuclei include:
D. Globus pallidus A. Edinger-Westphal nucleus
E. None of the above B. Dentate nucleus
37. The medial geniculate body is a C. Globus pallidus
relay centre for: D. Copus striatum
A. Hearing E. All of the above
B. Vision 43. Which of the following nerves are
C. Smell purely motor in function?
A. Trigeminal A. Thalamus
B. Abducent B. Hippocampus
C. Facial C. Amygdala
D. Glossopharyngeal D. Dentate gyrus
E. Vagus E. None of the above
44. The cranial parasympathetics are 50. The vagus nerve has its origin in the
conveyed through all of these nerves following nuclei:
EXCEPT: A. Nucleus ambiguus
A. Oculomotor B. Nucleus solitaries
B. Facial C. Dorsal vagal nucleus
C. Glossopharyngeal D. A and C
D. Vagus E. B and C
E. None of the above 51. In the spinal cord, ascending
45. The vagus exits the cranium pyramidal fibers:
through: A. Carry sensory information
A. Foramen ovale B. Are mostly in the dorsal
B. Foramen rotundum column
C. Jugular foramen C. Their 1st order neurons
D. Vagal foramen synapse in the dorsal horn
E. Foramen magnum D. Their 2nd order neurons
46. The main lobes of the cerebellum decussate in the pyramids
are: E. All of the above
A. Anterior lobe 52. Paralysis of the muscles of facial
B. Posterior lobe expression could be due to the
C. Flocculonodular lobe lesions to the:
D. A and B only A. CN III
E. A,B and C B. CN V
47. The biggest cerebellar nucleus is C. CN VII
the: D. CN IX
A. Dentate E. CN XI
B. Globose 53. The cerebellum receives fibers from
C. Emboliform the:
D. Fastigial A. Pontine nuclei
E. All of the above B. Vestibular nuclei
48. Parkinson disease is mostly C. Spinal cord
associated with the lesions of: D. Inferior olivary nuclei
A. Thalamus E. All of the above
B. Hypothalamus 54. The broca’s motor area of speech is
C. Red nucleus located in the dominant hemisphere
D. Substantia nigra in the:
E. Insula A. Island of Reil
49. The main constituents of the limbic B. Frontal lobe
system include the following C. Temporal lobe
EXCEPT: D. Occipital lobe
E. Parietal lobe E. All of the above
55. The main lobes of the cerebral 61. Regarding the gonads:
hemispheres are: A. They are also called
A. Frontal lobe spaermatozoa
B. Temporal lobe B. Male and female gonads
C. Occipital lobe fuse to produce zygote
D. Parietal lobe C. They are also called germ
E. All of the above cells
56. The Edinger-westphal nucleus in D. A and B
midbrain give rise to: E. None of the above
A. Olfactory nerve 62. Spermatogenesis takes place in the:
B. Optic nerve A. Efferent ductules
C. Oculomotor nerve B. Rete testis
D. Trochlear nerve C. Epididymis
E. Trigrminal nerve D. Seminiferous tubules
57. The fasciculi gracilis and cuneatus E. None of the above
convey sensory modalities and are 63. Which of the following statements is
found in the: TRUE?
A. Dorsal spinal column A. Semen is a mixture of
B. Ventral spinal column spermatozoa and accessory
C. Lateral spinal column glands secretions
D. Dorsal horn B. Each primary spermatocyte
E. Ventral horn is diploid
58. The following are parts of the C. Secondary spermatocytes
primitive heart EXCEPT: have the haploid number of
A. Primitive ventricle chromosomes
B. Bulbus cordis D. A and B
C. Aortic arches E. A,B and C
D. Tranchus arteriosus 64. Regarding oogenesis:
E. None of the above A. Oogonia are diploid
59. The following are important shunts B. It takes place in the ovary
during embryonic development: C. Results in the production of
A. Foramen ovale the ovum
B. Ductus arteriousus D. A and B
C. Ductus venosus E. A,B and C
D. A and B 65. Regarding ovulation:
E. A,B and C A. It is the release of an ovum
60. The commonly encountered defects into the uterine tube
in the Tetralogy of Fallot are: B. It takes place at about the
A. Pulmonary stenosis 14th day of the ovarian cycle
B. Ventricular septal defect C. Doesnot occur in pregnancy
C. Overriding aorta D. A and B
(Dextroposition) E. A,B and C
D. Right ventricular hypertrophy
66. Usually fertilization takes place in: B. The maternal surface
A. Lower part of the uterus presents cotyledons
B. Upper part of the uterus C. The umbilical cord is
C. Uterine tube attached to the fetal surface
D. Ovary D. A and B
E. Vagina E. A,B and C
67. What is the effect of fertilization: 72. Regarding twinning, fraternal twin
A. Restoration of the diploid are also referred to as:
number of chromosomes A. Dizygotic
B. Determination of sex B. Two-egg twins
C. Initiation of cleavage C. Biovular
D. A and B D. A and B
E. A,B and C E. A,B and C
68. Regarding the second week of 73. Regarding twinning, identical twins
embryological development which of are also reffered to as:
the following statement is FALSE? A. Monozygotic
A. The embryo is cylindrical in B. Two-egg twins
shape C. Biovular
B. The embryo has two laminae D. A and B
(layers) E. A,B and C
C. The embryo has two cavities 74. Regarding the branchial apparatus
D. The embryo has an amniotic which of the following is NOT
cavity TRUE?
E. The embryo has a yolk sac A. Formation of a great part of
69. Regarding the third week of the head and neck
embryological development which of B. Arches develop during the
the following statement is FALSE? second week
A. The embryo is cylindrical in C. It is also called the
shape pharyngeal apparatus
B. The embryo has two laminae D. Presents five viable branchial
(layers) arches
C. The embryo has two cavities E. Its mesoderm gives rise to
D. The embryo has an amniotic skeletal elements
cavity
E. The embryo has a yolk sac 75. The following constitute the
70. The following are fetal membranes branchial apparatus EXCEPT the:
EXCEPT: A. Branchial arches
A. Chorion B. Branchial pouches
B. Yolk sac C. Forehead
C. Allantois D. Branchial clefts
D. Umbilical cord E. Branchial membrane
E. Amnion
71. Regarding the placenta:
A. Its fetal surface is smooth
MUHIMBILI UNIVERSITY OF HEALTH AND ALLIED SCIENCES
SCHOOL OF MEDICINE

DEPARTMENT OF ANATOMY
FIRST CONTINOUS EXAMINATION FOR THE DEGREE BMLS, BRTT,
BSCN AND BPHARM (TIME: 1 1/2HRS)
CAND. NO:……………………………. NAME:……………………………..

GENERAL INSTRUCTIONS: This paper consists of THREE sections A, B and C; Answer all
the questions in three sections

SECTION A: MULTIPLE CHOICE


1. For each question choose the most correct answer and CIRCLE its corresponding letter
2. Each question carries 1 point

1. Any plane which is parallel to the 5. The heart is located in which


median plane: subdivision of the mediastinum?
A. Coronal A. Anterior
B. Frontal B. Middle
C. Sagittal C. Posterior
D. Medial D. Lateral
E. None of the above E. Superior

2. Pick the ‘odd man’ out: 6. The aortic arch is located in which
A. Medial subdivision of the mediastinum:
B. Superior A. Superior
C. Ventral B. Middle
D. Sagittal C. Posterior
E. Anterior D. Anterior
E. None of the above
3. A typical vertebra contains the
following EXCEPT: 7. The left bronchial artery frequently
A. Body take their origin from the:
B. Transverse foramen A. Internal thoracic artery
C. Vertebral foramen B. Descending aorta
D. Spinous process C. Aortic arch
E. Transverse process D. Subclavian artery
E. None of the above
4. Which of the following is not a TRUE
rib? 8. All of the following are parts of the
A. 1st rib parietal pleura EXCEPT:
B. 3rd rib A. Cervical
C. 5th rib B. Costal
D. 7th rib C. Pulmonary
E. 9th rib D. Diaphragmatic
E. Mediastinal
9. How many lobes does the left lung A. Brachiocephalic trunk
contain? B. Left common carotid artery
A. 1 C. Left subclavian artery
B. 2 D. Right subclavian artery
C. 3 E. None of the above
D. 4
E. 5 15. If a child inhales a foreign body, it is
more likely to be found at the:
10. How many lobes does the right lung A. Left main bronchus
contain? B. Right main bronchus
A. 1 C. At the carina
B. 2 D. Oesophagus
C. 3 E. None of the above
D. 4 16. The root of the lung contains the
E. 5 following EXCEPT:
A. Bronchus
11. These are bones of the axial skeleton B. Vagus nerve
EXCEPT: C. Pulmonary ligament
A. Scapula D. Pulmonary artery
B. Skull E. Pulmonary vein
C. Vertebrae
D. Ribs 17. During the inhalation, the diaphragm:
E. Sternum A. Remains static
B. Moves upward and forward
12. Increased resistance to pulmonary C. Moves downward and
blood flow in the lungs would cause a backward
direct strain on which chamber of the D. Relaxes and shrinks
heart? E. Recoils
A. Right ventricle
B. Left atrium 18. The heart is supplied by the:
C. Right atrium A. Coronary artery
D. Left ventricle B. Aorta
E. None of the above C. Left pulmonary artery
D. Right pulmonary artery
13. Which of the following accompanies the E. Internal thoracic artery
anterior inter-ventricular artery?
A. Middle cardiac vein 19. The base of the heart is mainly formed
B. Coronary sinus by the:
C. Small cardiac vein A. Left atrium and partly right
D. Great cardiac vein atrium
E. Anterior cardiac vein B. Right atrium and right ventricle
C. Left ventricle
14. The following branches arise from the D. Left ventricle and left atrium
aortic arch EXCEPT: E. Left ventricle and right atrium
20. Exocytosis and endocytosis are C. Epindymal cells
accomplished by the: D. Oligodendrocytes
A. Nucleus E. None of the above
B. Mitochondria
C. Cell membrane 26. Simple squamous epithelia are typically
D. Endoplasmic reticulum found in these locations EXCEPT:
E. All of the above A. Bowman’s capsule
B. Endothelium
21. Which of the following doesnot consist C. Epidermis
of the intermediate cytoskeleton? D. Alveoli
A. Vimentin E. Mesothelium
B. Cytokeratin
C. Actin 27. Transitional epithelium is found in the :
D. Desmin A. Uterus
E. None of the above B. Vagina
C. Urinary bladder
22. What part of the cell synthesizes D. Trachea
proteins? E. None of the above
A. Ribosomes
B. Mitochondria 28. Connective tissue fibers include the
C. Lysosomes following EXCEPT:
D. Endoplasmic reticulum A. Collagen fibers
E. All of the above B. Keratin fibers
C. Elastic fibers
23. Which of the following is not found in a D. Reticular fibers
bacterial cell? E. None of the above
A. DNA
B. Nuclear membrane 29. Resident connective tissue cells
C. Cell membrane include all of these EXCEPT:
D. Cytoplasm A. Macrophages
E. None of the above B. Fibroblasts
C. Monocytes
24. The absorptive surface of the gut is D. Histiocytes
lined by: E. Mast cells
A. Simple cuboidal epithelium
B. Simple columnar epithelium 30. The foramen magnum is located in
C. Stratified columnar epithelium which bone?
D. Stratified squamous epithelium A. Sphenoid
E. All of the above B. Occipital
C. Frontal
25. The glial cells include all of the D. Temporal
following EXCEPT: E. Ethmoid
A. Microglia
B. Astrocytes
SECTION B: ANSWER QUESTIONS: Use the answer sheet provided

31.
a) Define the word tissue (2 points)
b) Name the four basic tissues (4 points)
c) List one function for each (4 points)

32. Giving at least one example each, briefly describe the following terms (3 points each)
a) Foramen
b) Tuberosity
c) Abduction
d) Short bone
e) Fossa

SECTION C: ESSAY QUESTIONS: Use the answer sheet provided

33. Outline the differences between the eukaryotic and prokaryotic cells. (15 points)
34. Name and briefly explain the main components of the thoracic cage (15 points)
35. Discuss the heart under the following (5 points each)
a. Location and wall composition
b. Chambers and valves
c. Blood circulation
MUHIMBILI UNIVERSITY OF HEALTH AND ALLIED SCIENCES
SCHOOL OF MEDICINE
DEPARTMENT OF ANATOMY AND HISTOLOGY
MD1 / DDS1 CAT -2 ANATOMY EXAMINATION
CANDIDATE’S NAME: ………………………………… TIME: 1HR 30 MIN
DECEMBER 2009

GENERAL INSTRUCTIONS
This paper consists of a total of 75MCQ questions; answer all of them. Choose the most correct answer and
write its corresponding letter alongside its question number on ANSWER SHEET provided. Quarter (1/4) a
mark will be deducted for every wrong answer

1. Part of the lung aerated by a respiratory E. A,B and C


bronchiole is known as a/an: 5. Regarding the thyroid gland:
A. Segment A. It is the second largest endocrine
B. Lobule gland after the pituitary
C. Pulmonary unit B. It’s secretory product is stored
D. Alveolus extracellulary
E. Lobe C. The follicle is made up of simple
2. Regarding the lymphoid tissue: cuboidal epithelial cells
A. Hassall’s corpuscles are found in D. A and B
the red pulp of the spleen E. B and C
B. Splenic cords of Billroth are
found in the white pulp of the 6. Which of the following statement(s)
spleen is/are TRUE, regarding the male
C. Splenic nodules do not possess reproductive system?
germinal centres A. Sertoli cells support and nourish
D. A and B spermatids
E. None of the above B. Tunica vaginalis refers to a tough
3. Regarding the liver: fibrous coat
A. The space of Disse lies between C. Testosterone is secreted by cells
sinusoidal walls and plates of of the epididymis
hepatocytes D. A and B
B. Kuppfer cells are present in the E. A and C
sinusoidal lining of the liver
C. The portal area comprise of 7. Regarding the female reproductive
branches of the portal vein, system:
hepatic artery and bile duct A. Primordial follicles consists of an
D. A and C oocyte surrounded by a single
E. A,B and C layer follicular cells
4. The mucous membrane of the trachea B. The ampulla of the fallopian tube
consists of: is the narrowest portion
A. Ciliated cells C. The uterus has prominent circular
B. Mucous goblet cells coat of smooth muscle
C. Brush cells D. A and C
D. A and B E. B and C
8. In reference to the thymus, pick the B. Parietal cells
CORRECT statement(s): C. Enteroendocrine cells
A. The epithelial reticular cells are less D. Paneth cells
numerous in the cortex than in E. All of the above
medulla 13. The submandibular gland:
B. The thymocytes are less numerous A. Is a mucus type salivary gland
in the medulla than in the cortex B. Lies inferomedial to the mandibular
C. Hassall’s corpuscles are found in the body
medulla C. Contributes the biggest portion of
D. A and C salivary volume
E. A,B and C D. A and B
E. B and C
9. Regarding the spleen; find the CORRECT
statement(s): 14. The uterine layer that peel off during
A. Splenic nodules aggregates of red menstruation is the:
blood cells A. Basal endometrium
B. It is important hemopoietic organ B. Functional endometrium
during fetal life C. Endometrial glands
C. Splenic sinuses collect lymph D. Myometrium
D. A and B E. All of the above
E. B and C
15. Regarding the juxtaglomerular apparatus,
10. Regarding the small intestines the following choose the CORRECT statement(s):
are true EXCEPT: A. JG cells produce rennin
A. Peyer’s patches are found in the B. Macula densa is a part of the distal
mucosal and submucosal band tubule
B. Auerbach’s plexus is in the C. Lacis cells are found between
submucosa afferent and efferent arteriole
C. Brunner’s glands are found in the D. A and B
duodenum E. A,B and C
D. Plicae riculares are more prominent 16. Hydrocele refers to the collection of serous
in the duodenum than in the ileum fluid in the:
E. None of the above A. Tunica vaginalis
B. Tunica albuginea
11. Villi are the longest in the: C. Tunica dartos
A. Duodenum D. Tunica vasculosa
B. Jejunum E. Mediastinum testis
C. Ileum 17. The classical liver lobule:
D. Iliocecal junction A. Is a triangular in outline
E. Colon B. Has the central vein at its centre
C. Has portal canals at its centre
12. The low gastric pH due to hydrochloric acid D. Its center has the highest blood
and it is secreted by: supply
A. Chief cells E. All of the above
18. The ureter is lined by the: 23. The pituitary gland has the following
A. Simple squamous features EXCEPT the:
B. Simple cuboidal A. Acidophilic cells
C. Transitional epithelium B. Basophilic cells
D. Pseudostratified epithelium C. Pituicytes
E. Stratified squamous non-keratinized D. Lymphocytes
E. Herring bodies
19. The bisection of the kidney may show the 24. Regarding the adrenal glands:
following structure(s): A. The zona glomerulosa produce
A. Renal capsule mineralocorticoids
B. Renal cortex B. The medulla is avascular
C. Renal medulla C. Ganglion cells in the cortex produce
D. Renal calyx adrenaline
E. All of the above D. A and B
E. A,B and C
20. The erectile tissue of the penis include:
A. Corpora carvenosa 25. Regarding the Dura matter:
B. Corpus spongiosum A. It has an endosteal and meningeal
C. Glans penis layer
D. A and B B. The endosteal layer doesnot extend
E. B and C through the foramen magnum
C. The meningeal layer is a strong
21. Regarding the pancreas: fibrous tissue
A. Intercalated ducts are lined by the D. A and B
cuboidal epithelium E. A,B and C
B. Centro-acinar cells secrete
protrypsin 26. Find the FALSE statement(s):
C. Alpha cells in the islets of A. The falxi cerebri houses the straight
langerhans produce insulin sinus at its junction with tentorium
D. A and B cerebelli
E. A and C B. The arachnoid matter closely invests
the brain
22. Regarding large intestines: C. The inferior sagittal sinus occupies
A. Mucosal glands are present but villi the attached margin of the falxi
are absent cerebelli
B. It is lined by the columnar cells with D. A and B
many goblet cells E. A and C
C. The inner circular and outer
longitudinal muscle layers for the 27. The following structures pass through the
teniae coli superior orbital fissure EXCEPT:
D. A and B A. Abducent nerve
E. A and C B. Maxillary nerve
C. Opthlamic nerve
D. Trochlear nerve
E. None of the above A. It communicates with carvenous
sinus
28. Regarding the venous drainage of thyroid: B. It empties into the external jugular
A. The inferior thyroid vein drains into vein
the brachiocephalic vein C. The supraorbital and supratrochlear
B. The middle thyroid veins drains into veins form its main tributaries
the internal jugular vein D. It also communicates with the deep
C. The superior thyroid vein drains into facial vein
the subclavian E. None of the above
D. A and B
E. A and C 33. Regarding the prostate gland find the
CORRECT statement (s):
29. Regarding the emissary veins find the A. It is a fibromuscular gland
correct statement(s): B. It surrounds the prostatic urethra
A. They are found in the loose C. Contributes about 60% of the
connective tissue layer ejaculate volume
B. They connect the superficial veins D. A and B
of the scalp with diploic veins E. A,B and C
C. They are valveless
D. B and C 34. The uterus has the following parts EXCEPT:
E. A,B and C A. Body
B. Cervix
30. The pharynx is divided into: C. Fundus
A. Nasopharynx D. Ampulla
B. Oropharynx E. None of the above
C. Laryngopharynx
D. A and B 35. The venous drainage of the liver is through:
E. A,B and C A. Hepatic veins
B. Hepatic portal vein
31. Regarding the dural venous sinuses find the C. Superior mesenteric vein
FALSE statement(s) : D. Inferior vena cava
A. These are venous channels with E. All of the above
strong semilunar valves
B. Theya are found in the dura matter 36. Regarding spermatogenesis find the
of the brain CORRECT statement:
C. They communicate with diploic A. It begins at puberty and stops at
veins around 45 years
D. They drain into the internal jugular B. It takes place in the interstitial
vein tissues of the testes
E. None of the above C. It give rise to diploid cells called
spermatozoa
32. Regarding facial vein find the INCORRECT D. It begins during the intrauterine life
statement(s): E. None of the above
D. The thyroid cartilage
37. During the oogenesis the mature follicle is E. All of the above
also called: 42. The glomerular filtration barrier comprises
A. Primary follicle of all of these EXCEPT:
B. Secondary follicle A. Endothelial cells
C. Graafian follicle B. Podocytes
D. Corpus luteum C. Visceral bowman’s capsule
E. None of the above epithelium
D. Capillary basement membrane
38. The following structures pass through the E. None of the above
foramen magnum:
A. Vertebral arteries 43. The longest part of the fallopian tube is the:
B. The spinal cord A. Fimbria
C. Internal jugular vein B. Infundibulum
D. A and B C. Ampulla
E. A,B and C D. Isthmus
E. Uterine part
39. The following arteries form the circle of
Willis EXCEPT? 44. The vagina:
A. Anterior cerebral artery A. Is a female copulatory organ
B. Anterior communicating artery B. Acts as a birth canal during
C. Posterior cerebral artery parturition
D. Posterior communicating artery C. Also conveys urine to the exterior
E. None of the above D. A and B
E. A,B and C
40. Which of the following orderly describes the
histological parts of the adrenal cortex from 45. A female baby is born with predetermined
the outside to inside? number of gametic cell at what stage of
A. Zona glomerulosa, zona fasculata, development:
zona reticularis A. Oogonium
B. Zona glomerulosa, zona reticularis B. Primary oocyte
zona fasculata C. Primordial germ cell
C. zona fasculata, zona glomerulosa, D. Secondary oocyte
zona reticularis E. Ootid
D. zona fasculata, zona reticularis, zona
glomerulosa 46. Which cell provides physical and nutritional
E. zona reticularis, zona glomerulosa, support to the growing sperm cells?
zona fasculata A. Sertoli cell
B. Leydig cell
41. Which structure prevents food from entering C. Spermatogenic cells
the trachea? D. Phagocytic cells
A. The tongue E. All of the above
B. The epiglottis
C. The uvula
47. Which of the following is not phagocytic 52. The greater occipital nerve:
cell? A. Its root value is the C4 posterior
A. Monocyte ramus
B. Neutrophil B. Also participates in the innervations
C. Macrophage of the trapezius muscle
D. Lymphocyte C. Innervates the posterior scalp after
E. Dendritic cells piercing the semispinalis capitis and
trapezius muscle
48. Langerhan’s cells are said to be responsible D. A and C
for the immune system of the skin and the E. B and C
originate from the:
A. Neural crest 53. Branches of the external carotid artery
B. Lymph nodes include the following EXCEPT:
C. Thymus A. Mandibular artery
D. Bone marrow B. Maxillary artery
E. Spleen C. Facial artery
D. Occipital artery
49. Find the correct statement (s): E. Lingual artery
A. Brunner’s glands are found in the
duodenum 54. Which of the following muscle does not
B. Brunner’s glands are mainly form the floor of the posterior triangle of the
mucosal glands neck:
C. Lacteal is a characteristic lymphatic A. Semispinalis capitis
vessel found in the core of villi B. Splenius capitis
D. A and B C. Levator scapulae
E. A and C D. Trapezius
E. None of the above
50. Which of these does not form part of
adenohypophysis? 55. Tributaries of the internal jugular vein
A. Pars distalis includes:
B. Pars intermedia A. Lingual veins
C. Pars nervosa B. Pharyngeal veins
D. Pars tuberalis C. Facial veins
E. None of the above D. A and B
E. A,B and C
51. The following hormones are secreted by the
anterior pituitary gland EXCEPT: 56. The foramen spinosum is located in which
A. Thyroid stimulating hormone bone?
B. Prolactin A. Sphenoid
C. Follicle stimulating hormone B. Occipital
D. Oxytocin C. Frontal
E. Adrecorticotropic hormone D. Temporal
E. Ethmoid
57. The foramen magnum is located in which 62. Which of the following accompanies optic
bone? nerve through the optic canal?
A. Sphenoid A. Cranial nerves III,IV,VI
B. Occipital B. Ophthalmic nerve
C. Frontal C. Meninges and ophthalmic artery
D. Temporal D. Ophthalmic veins
E. Ethmoid E. None of the above

58. The middle cerebral artery is a continuation 63. Which nerve innervates the muscle for tight
of: closure of the eyelids?
A. External carotid artery A. Facial
B. Internal carotid artery B. Oculomotor
C. Vertebral artery C. Sympathetic
D. Basilar artery D. Trigeminal
E. Interior choroids artery E. Vagus
64. The sella turcica is a part of which bone:
59. The posterior cranial fossa: A. Temporal
A. Its roof is formed by the tentorium B. Sphenoid
cerebri C. Ethmoid
B. The occipital bone forms its floor D. Occipital
C. The parietal bone forms its E. Lacrimal
anterolateral wall
D. Houses the posterior part of the 65. The thyroid gland can be examined in which
cerebral hemispheres of the following triangles of the neck?
E. All of the above A. Submental
B. Glandular
60. Which of the following structures extends in C. Carotid
the midline between the two cerebral D. Submandibular (or digastric)
hemispheres? E. Muscular (or visceral)
A. Falxi cerebri
B. Tentorium cerebella 66. The facial artery give rise to branches that
C. Falxi cerebella supply each of the region below EXCEPT
D. Diaphragm sellae for the:
E. Leptomeninges A. Medial angle of the orbit
B. Lateral nose
61. The superior sagittal sinus: C. Eyebrow region
A. Drains into the straight sinus D. Upper lip
B. Is attached to the petrous temporal E. Lower lip
bone 67. The female homologue of the penis is the:
C. Is continous with the right A. Hymen
transverse sinus B. Vagina
D. Communicates with the carvenous C. Cervix
sinus D. Clitoris
E. Receives the superior petrosal sinus E. Labia majora
68. Lymphatic capillaries: D. Lateral pterygoid
A. Are lined by squamous cells E. None of the above
B. Endothelial cells overlap to form a
one-way valve 73. Accessory genital glands included the
C. Are generally small then ordinary following:
blood capillaries A. Bulbourethral glands
D. A and B B. Prostate glands
E. A,B and C C. Seminal vesicles
D. B and C
69. Regarding the breast choose the CORRECT E. A,B and C
statement (s):
A. The glandular tissue extend deeper 74. Wrinkling of the scrotal skin is due to:
to the pectoral muscles A. Dartos muscles
B. Has about 150 lobes B. Cremaster muscle
C. The nipple is covered with non- C. Tunica albuginea
keratinized squamous epithelium D. Tunica vasculosa
D. Lactiferous duct traverses the nipple E. All of the above
to open to the exterior
E. None of the above 75. Spermatozoa are stored in the:
A. Seminal vesicles
70. Regarding the pancreas find the CORRECT B. Epididymis
statement (s): C. Ductus deferens
A. It has both exocrine and endocrine D. Spermatic cords
components E. Testes
B. Opens into the duodenum together
with the common hepatic duct
C. Opens into the stomach together
with the common hepatic duct
D. A and B
E. A and C

71. The coronoid process belongs to which bone


in the head?
A. Maxilla
B. Mandible
C. Sphenoid ----- SUCCESS----
D. Occipital
E. Temporal

72. The muscles of mastication include the


following EXCEPT:
A. Masseter
B. Temporalis
C. Medial pterygoid
CANDIDATE No: ………………………

UNIVERSITY OF DAR ES SALAAM


MUHIMBILI COLLEGE OF HEALTH SCIENCES
SCHOOL OF MEDICINE

END OF FIRST SEMESTER UNIVERSITY EXAMINATIONS FEBRUARY 2007


EXAMINATION FOR THE DEGREE OF DOCTOR OF MEDICINE
ANATOMY PAPER 1

DATE: 12TH FEBRUARY, 2007 TIME: 9:00 AM – 12:00 NOON


____________________________________________________________________________

INSTRUCTIONS: Answer all multiple choice questions. Choose one most appropriate answer
and CIRCLE the preceding letter (1 mark each)
NOTE: For each wrong attempt quarter of a mark will be deducted
____________________________________________________________________________

1. The following muscles belong to the ‘rotator cuff’ of the shoulder EXCEPT:
A. Subscapularis
B. Deltoid
C. Supraspinatus
D. Infraspinatus
E. Teres minor

2. The following muscle is inserted on the medial lip of the intertubecular groove:
A. Teres major
B. Teres minor
C. Pectoralis minor
D. Pectoralis major
E. Latissimus dorsi

3. In the hand, the median nerve supplies:


A. Abductor pollicis brevis
B. Adductor pollicis
C. First dorsal interossei
D. Abductor pollicis longus
E. Extensor indicis
4. The innervations of the lumbrical muscles is related to the innervations of:
A. Flexor digitorum superficialis
B. Flexor digitorum profundus
C. Extensor digitorum
D. The interossei
E. The two flexor carpi muscles
5. The following are the branches of the axillary artery EXCEPT:
A. Internal thoracic artery
B. Thoracoacromial artery
C. Lateral thoracic artery
D. Subscapular artery
E. Anterior circumflex humeral artery

6. The following statement (s) is/are TRUE:


A. The anterior axillary fold is formed by the lower margin of the pectoralis major
muscle
B. Posterior axillary fold is formed by the tendon of latissimus dorsi
C. The inferior angle of the scapula lies opposite the seventh thoracic spine
D. A and B
E. A,B and C

7. The ulnar nerve is closely related to the:


A. Medial epicondyle of the humerus
B. Shaft of the humerus
C. Lateral epicondyle of the humerus
D. Surgical neck of the humerus
E. None of the above

8. The floor of the ‘anatomical snuffbox’ is formed by the following structures:


A. Scaphoid
B. Trapezium
C. Base of the first metarcarpal bone
D. Styloid process of the radius
E. All of the above

9. Regarding the upper limb, the following statement (s) is/are TRUE:
A. The mammary gland is partly supplied by the perforating branches of the internal
thoracic artery
B. In the dorsum of the hand the basilica vein passes medially
C. In the cubital fossa the cephalic and basilica veins are united by the medial
cubital vein
D. A and B
E. A,B and C

10. Regarding the lymphatic drainage of the mammary gland (breast) the following is/are
TRUE?
A. The lateral parts of the gland is drained into the anterior axillary lymph nodes
B. The medial parts drain into the lymph nodes located along the internal thoracic
artery
C. Involvement of the lymph nodes with cancer can lead to disseminated disease
D. A and B
E. A,B and C

11. The anterior wall of the axilla is formed by the:


A. Pectoralis major
B. Subclavius
C. Pectoralis minor
D. A and B
E. A,B and C

12. Long thoracic nerve supplies the following muscles:


A. Subscapularis
B. Latissimus dorsi
C. Teres major
D. Serratus anterior
E. Trapezius

13. The branche(s) of the second part of axillary artery include:


A. Highest thoracic artery
B. Subscapular artery
C. Anterior circumflex humeral artyery
D. A and B
E. A,B and C

14. The root value of ulnar nerve is:


A. C8 and T1
B. T1
C. C5 and C6
D. C5
E. C5,6 and 7

15. The medial cord of brachial plexus gives rise to the following branche(s):
A. Ulnar nerve
B. Axillary nerve
C. Musculocutaneous nerve
D. Radial nerve
E. A and B

16. Arterial branches that take part in the anastomosis around the shoulder joint include:
A. Superficial cervical artery
B. Suprascapular artery
C. Subscapular artery
D. A and B
E. A,B and C
17. Lateral rotation of the shoulder joint is performed by the following muscles:
A. Infraspinatus
B. Teres minor
C. Posterior fibres of the deltoid muscle
D. A and B
E. A,B and C
18. The pectoralis minor muscle is inserted at the:
A. Coracoid process of the scapula
B. Clavicle
C. Lateral lip of the bicipital groove of the humerus
D. Third, fourth and fifth ribs
E. Lesser tuberosity of the humerus
19. Brachialis is innervated by the:
A. Radial nerve
B. Musculocutaneous nerve
C. Ulnar nerve
D. A and B
E. A and C
20. The triceps muscle is inserted at the:
A. Tuberosity of radius
B. Coracopid process of ulna
C. Olecranon process of the ulna
D. A and B
E. B and C
21. Regarding the internal thoracic artery, the following statement (s) is/are TRUE:
A. it gives the anterior intercostals arteries
B. it is a branch of the subclavian artery
C. it supplies the mammary gland
D. it gives the superior epigastric artery
E. all of the above
22. The following are the components of the middle mediastinum EXCEPT:
A. The pericardium
B. Roots of the lungs
C. Azygous veins
D. Phrenic nerve
E. None of the above

23. Regarding the intercostals space:


A. The posterior intercostals arteries are branches of the thoracic aorta
B. The anterior intercostals arteries are branches of the thoracic aorta
C. All the intercostals nerves are the dorsal rami of the thoracic nerve
D. The axillary artery supplies the upper five intercostals space
E. None of the above
24. Find the FALSE statement(s):
A. The cordae tendinae connects the tricuspid valves to the papillary muscles
B. The right recurrent branch of the vagus nerves turns around the arch of the aorta
C. The brachiocephalic veins are continuations of the superior vena cava
D. Coronary sinus is a continuation of the greater cardiac vein
E. None of the above
25. Regarding the pericardium, find the FALSE statement(s):
A. The fibrous pericardium is supplied by the coronary arteries
B. The fibrous pericardium lines the heart
C. Cardiac temponade may result from pericardial effusion
D. A and B
E. B and C
26. The following statement(s) is/are TRUE:
A. In pericardial effusion fluid accumulates between the visceral and parietal
pericardium
B. Pneumothorax results from accumulation of air in the pleural cavity
C. Normally the left lung has one major fissure
D. A and B
E. A,B and C

27. Regarding the vertebrae:


A. The thoracic vertebrae have bifid transverse process
B. Nucleus polposus is the central portion of the intervertebral disc
C. Lumbar vertebrae have thicker intervertebral discs compared to the thoracic
region
D. A and C
E. A,B and C

28. The angle of Louis:


A. Is also known as the sternal angle
B. Its palpation is crucial for the identification of the intercostals spaces during chest
examination
C. In many cases, it is located at the lower border of T2
D. A and B
E. A,B and C

29. Regarding the surface markings of the heart:


A. The right border is formed by the right atrium
B. The left border is formed by the left ventricle
C. The inferior border id formed by the right ventricle and apical part of the left
ventricle
D. A and C are true
E. A,B and C
30. The following statement (s) is/are TRUE:
A. The presence of cervical rib can lead to pain in the upper limb and pressure to
the subclavian artery
B. The sternum is the commonest site for bone marrow biopsy because it contains
the red bone marrow throughout life
C. Foramen transvesarium of the seventh cervical vertebra contains the vertebral
artery
D. A and B
E. A, B and C

31. The coronary arteries from the:


A. Arch of the aorta
B. Ascending aorta
C. Left atrium
D. Aortic valves
E. Descending aorta

32. The root value of phrenic nerve is:


A. C3 ,4 ,5
B. C1, 2, 3
C. C5, 6
D. C4, 5, 6,7
E. C3 only

33. The following muscles are supplied by the mandibular nerve EXCEPT:
A. Temporalis
B. Masseter
C. Buccinator
D. Anterior belly of the digastric muscle
E. Medial pterygoid

34. The three divisions of the trigeminal nerve are:


A. Ophthalmic
B. Maxillary
C. Mandibular
D. A and B
E. A, B and C
35. The sternocleidomastoid muscle is innervated by the :
A. Vagus nerve
B. Accessory nerve
C. Glossopharyngeal nerve
D. Hypoglossal nerve
E. Branches from the facial nerve
36. The following are muscles of facial expression EXCEPT:
A. Temporalis
B. Orbicularis oculi
C. Orbicularis oris
D. Buccinator
E. Occipito-frontalis muscle

37. The parotid duct opens into the oral cavity opposite the crown of the:
A. Upper first premolar tooth
B. Upper second molar
C. Lower first premolar
D. Upper second premolar
E. Lower second molar

38. Regarding the pterion, the following statement(s) is/are TRUE:


A. Is a point where the greater wing of the sphenoid bone meets the antero-inferior
angle of the parietal bone
B. Is related to the anterior branch of the middle meningeal artery
C. Injury can lead to subdural haemorrhage
D. A and B
E. A, B and C

39. Regarding the head and neck, the following statement (s) is/are TRUE:
A. Normally the thyroid gland is supplied by the two arteries and drained by three
veins
B. The inferior thyroid veins lie in front of the fifth, sixth and seventh tracheal rings
C. Platysma muscle is supplied by the facial nerve
D. Glossopharyngeal nerve has parasympathetic fibers
E. All of the above

40. Regarding the external jugular vein the following statement(s) is/are:
A. It begins by the union of the posterior auricular and the retromandibular veins
B. It opens into the subclavian vein
C. It also opens into the internal jugular vein
D. A and B
E. A, B and C

41. Regarding the SCALP the following statement (s) is/are TRUE:
A. Its infection can spread to the brain
B. It contains the emissary veins that communicates with the diploic veins
C. Its wounds may bleed profusely
D. A and B
E. A, B and C
42. The paranasal air sinuses include the following:
A. Sphenoid
B. Maxillary
C. Ethmoid
D. Frontal
E. All of the above
43. The 6th cervical vertebrae marks the:
A. Junction of the larynx with the trachea
B. Level at which inferior artery enters the thyroid gland
C. Junction of the pharynx with the oesophagus
D. A and B
E. A, B and C
44. The cartilages of the larynx include the:
A. Cricoid cartilage
B. Arytenoids cartilage
C. Conchae
D. A and B
E. A,B and C
45. The carotid sheath contain the following structures:
A. Vagus nerve
B. Internal jugular vein
C. Subclavian arteries
D. A and B
E. B and C
46. Platysma muscle is supplied by the:
A. Vagus nerve
B. Glossopharyngeal nerve
C. Facial nerve
D. Mandibular nerve
E. Cervical branches of the spinal nerve

47. The eyelids are supplied by the following nerves:


A. Lacrimal nerve
B. Supratrochlear
C. Infratrochlear
D. Supraorbital
E. All of the above

48. The foramen magnum is located in the:


A. Sphenoid bone
B. Occipital bone
C. Vomer
D. Temporal bone
E. Parietal bone
49. Regarding the head and neck, the internal ear is located in the:
A. Temporal bone
B. Occipital bone
C. Parietal bone
D. Zygomatic bone
E. Sphenoid bone
50. In a normal baby the anterior fontanelle closes at about:
A. 1 year
B. 18 months
C. 3 years
D. 4 years
E. Above 5 years
51. The optic canal transmits the:
A. Optic nerve
B. Ophthalmic artery
C. Maxillary nerve
D. A and B
E. B and C
52. The jugular foramen transmits the following structures:
A. Inferior petrosal sinus
B. Sigmoid sinus
C. Glossopharyngeal nerve
D. A and B
E. A, B and C
53. Regarding the cranial cavity, the sella turcica contains:
A. Pituitary gland
B. Otic ganglion
C. Temporal bones
D. Mesencephalic ganglion of the trigeminal nerve
E. All of the above are true

54. The cerebral spinal fluid circulates in the:


A. Subarachnoid space
B. Subdural space
C. Epidural space
D. Dural sinuses
E. A and C
55. Regarding the brain, the cerebral aqueduct is a cavity in the:
A. Pons
B. Midbrain
C. Medulla oblongata
D. Diencephalons
E. Cerebellum
56. Regarding the cranial cavity, the middle cranial fossa contains the:
A. Temporal lobes
B. Occipital lobes
C. Frontal lobes
D. Cerebellum
E. Lateral sulcus
57. Regarding the brain, pineal gland is a part of:
A. Occipital lobe
B. Diencephalons
C. Temporal lobes
D. Basal ganglia
E. None of the above
58. Regarding the meninges, the outer-most covering of the brain is:
A. Dura matter
B. Pia matter
C. Arachnoid matter
D. Pia-arachnoid matter
E. Ependymal cells

59. Reduced muscle tone and hyporeflexia are associated with:


A. Upper motor neurone lesion
B. Lower motor neurone lesion
C. Both upper and lower motor neurone lesion
D. Injury to parasympathetic motor neurons
E. All of the above
60. Regarding the skeletal muscles, the motor impulses are influenced by the:
A. Basal ganglia
B. Cerebellum
C. Hypothalamus mainly
D. A and B
E. B and C
61. Parkinson’s diseases may result from lesion to the:
A. Basal ganglia
B. Ventral horn of the spinal cord
C. Medulla oblongata
D. Pons
E. All of the above
62. Edinger-Westphal nucleus is located in the:
A. Pons
B. Midbrain
C. Medulla oblongata
D. Diencephalons
E. Cerebral cortex
63. The deep cerebellar nuclei include the following:
A. Dentate
B. Emboliform
C. Fastigial
D. Nucleus globosus
E. All of the above
64. The largest branch of the internal carotid artery in the cranial cavity is the:
A. Middle cerebral artery
B. Anterior cerebral artery
C. Ophthalmic artery
D. Choroidal artery
E. Posterior communicating artery
65. Glial cells that are responsible for depositing myelin sheath to the nerve axons are:
A. Schwann cells
B. Oligodendrocytes
C. Microglia
D. A and B
E. A, B and C
66. Membrane bound organelle include the following:
A. Endoplasmic reticulum
B. Mitochondria
C. Golgi apparatus
D. Lysosome
E. All of the above
67. Calcium ions are stored in the following organelles:
A. Endoplasmic reticulum
B. Mitochondria
C. Peroxisomes
D. A and B
E. B and C

68. Large molecules are transported into or outside the cell cytoplasm via;
A. Pinocytosis
B. Exocytosis
C. Phagocytosis
D. A and B
E. A, B and C

69. Regarding the intercellular junctions the following statement (s) is/are TRUE:
A. Anchoring junctions are associated with the cytoskeleton
B. Desmosomes have a function of holding cells cells together
C. Hemi-desmosomes occurs between the cell and extracellular elements
D. Communications junctions include gap junctions and chemical synapse
E. All of the above
70. The following are functions of the smooth endoplasmic reticulum:
A. Lipid synthesis
B. Detoxification of harmful substances in the liver cell
C. Release of platelets into the circulation
D. A and B
E. A, B and C
71. Regarding the Golgi apparatus, the following statement(s) is/are TRUE:
A. The Cis-compartment receive materials from the endoplasmic reticulum
B. The trans-compartment is associated with secretory vesicles
C. Transfer of materials to the Golgi is done by transport vesicles
D. A and C
E. A, B and C
72. During mitosis the nuclear envelope disappears at:
A. Prophase
B. Interphase
C. Metaphase
D. Anaphase
E. It does not disappear
73. Mitotic spindle includes:
A. Polar microtubules
B. Astral microtubules
C. Kinetochore microtubules
D. A and C
E. A, B and C
74. Examples of proteins that form intermediate dilaments include:
A. Keratin
B. Vimentin
C. Desmin
D. Neurofilaments
E. All of the above
75. Regarding the bone, find the CORRECT statement(s);
A. It is surrounded by fibrous tissue known as periosteum
B. Calcium salts are deposited in the intercellular matrix
C. Red bone marrow contain erythroblasts,myeloblasts and megakaryoblasts
D. A and B
E. A,B and C

76. The following cells are found in the bone EXCEPT:


A. Osteocytes
B. Chondrocytes
C. Osteoclasts
D. Osteoblasts
E. Osteoprogenitor cells
77. Which of the following is NOT a long bone:
A. Radius
B. Ulna
C. Scaphoid
D. Metatarsal
E. Middle phalanx
78. Cells that perform bone destruction are:
A. Osteoclasts
B. Osteoblasts
C. Osteocytes
D. A and C
E. A, B and C
79. The cell that synthesize connective tissue fibers are:
A. Fibroblasts
B. Neuronal cells
C. Mast cells
D. Eosinophils
E. None of the above
80. Connective tissue macrophages arises from:
A. Blood monocytes
B. Mast cells
C. Neutrophils
D. Adipocytes
E. None of the above
81. The functions of macrophages include:
A. Phagocytosis
B. Antigen presentation
C. Production of some trophic (growth) factors
D. A and B
E. A, B and C

82. Regarding the cardiac muscle cells, the following is/are TRUE:
A. Have centrally placed nuclei
B. Are connected by the intercalated discs
C. Desmosomes are the commonest intercellular junctions
D. Presence of gap junction allows electrical coupling among cells
E. All of the above

83. Smooth muscles cells are supplied by:


A. Somatic nerves
B. Visceral nerves
C. Both viscera and somatic nerves
D. A and B
E. A, B and C
84. Nissl granules are seen in:
A. Neuronal cells
B. Motor neurons only
C. Sympathetic neurons only
D. Sensory neurons
E. Dendrites
85. Regarding endothelium, find the CORRECT statement(s):
A. It forms the inner lining of blood vessels
B. It contains simple squamous epithelium
C. It is the main component of body capillaries
D. A and B
E. A, B and C
86. Regarding the epithelial tissue, the following statement(s) is/are TRUE:
A. It forms the glandular tissue
B. It is associated with the basement membrane
C. Normally the cells do not have a direct contact with blood vessels
D. In simple epithelium the cells have apical and basolateral surfaces
E. All of the above
87. The anal canal is lined by:
A. Stratified squamous epithelium
B. Simple columnar epithelium
C. Keratinized epithelium
D. A and B
E. A, B and C
88. Testosterone is secreted by:
A. Leydig cells
B. Cells of the adrenal cortex in small amount
C. Sertoli cells
D. A and B
E. B and C

89. Regarding fertilization, the following statement(s) is/are TRUE:


A. It normally occur in the fallopian tubes
B. It involve enzymatic activities
C. One oocyte is normally penetrated by a single spermatozoon
D. A and B
E. A, B and C

90. The morulla consists of:


A. Blastomeres
B. Cytotrophoblasts
C. Blastocoele
D. Corona radiata
E. All of the above
91. The following are features are associated with implantation:
A. Loss of zona pellucida
B. Erosion of endometrium by the trophoblasts
C. Formation of deciduas basalis, parietalis and capsularis
D. A and B
E. A, B and C

92. Regarding twinning, the following is/are TRUE:


A. Identical twins are normally of the same sex
B. Under normal circumstances each twin has its own umbilical cord
C. Fertilization of two oocytes is likely to result into identical twins
D. A and B
E. A, B and C

93. During the gonadal formation germ cells arise from the:
A. Wall of the yolk sac
B. Embryonic mesoderm
C. Ectoderm
D. Neural crest cells
E. None of the above

94. The following feature(s) is/are associated with folding of the embryo:
A. The endoderm is incorporated to form the primitive gut
B. The heart come to lie caudal to the septum transversum
C. The developing brain assume the most cranial position
D. A and B
E. A, B and C

95. Find the CORRECT statement(s):


A. The neural tube develop from embryonic ectoderm
B. Adrenal medulla arise from migrated neural crest cells
C. The intermediate mesoderm is also called nephrogenic mesoderm
D. Splanchopleuric mesoderm give rise to smooth muscle of the gut
E. All of the above

96. The diaphragm:


A. Arises from the xiphoid process, lower six costal cartilages and three lumbar
vertebrae
B. Is supplied by the fibers that come from C3, 4 and 5
C. Its origin is from the central tendon
D. A and B
E. A and C
97. Presence of air in the thoracic cavity is clinically reffered to as:
A. Pneumothorax
B. Haemothorax
C. Pleural effusion
D. B and C
E. None of the above
98. Congential heart defects include the following conditions:
A. Atrial septal defect
B. Ventricular septal defect
C. Patent ductus arteriosus
D. Tetralogy of Fallot
E. All of the above
99. The cervical vertebrae:
A. Are 7 in number
B. Vertebral artery ascends in the transverse foramina
C. One of the typical characteristic is a bifid spinous process
D. A and B
E. A, B and C
100. The following structure(s) stabilize(s) the temporomandibular joint:
A. The lateral temporomandibular ligament
B. Sphenomandibular ligament
C. Stylomandibular ligament
D. Lateral pterygoid muscle
E. All of the above
101. The digastric muscle is supplied by the:
A. Trochlear nerve
B. Facial nerve
C. Hypoglossal nerve
D. Glossopharyngeal nerve
E. None of the above
102. The salivary glands include the following:
A. The parotid glands
B. The submandibular glands
C. Smaller glands located beneath the oral mucosa
D. A and B
E. A, B and C
103. The following vessel(s) supply blood to the tongue
A. Lingual branch of the external carotid artery
B. Tonsilar branch of the facial nerve
C. Ascending palatine branch of the facial artery
D. Ascending pharyngeal arteries
E. All of the above
104. The Waldeyer’s ring consists of the following:
A. Tubal tonsils
B. Naso-pharygeal tonsils (adenoid)
C. Lingual tonsils
D. Palatine tonsils
E. All of the above

105. Muscles of facial expression include the following:


A. Platysma
B. Occipito-frontalis
C. Buccinator
D. Orbicularis oculi
E. All of the above

106. The foramen magnum transmits the following structures:


A. The medulla oblongata and its surrounding meninges
B. Vertebral arteries
C. Ascending spinal part of the accessory nerve
D. A and B
E. A, B and C

107. Purkinje fibers are found in the:


A. Cerebral cortex
B. Cerebellar cortex
C. Spinal cord
D. Medulla oblongata
E. None of the above

108. Pre-ganglionic neurons of the autonomic nerves (cell bodies) are located in the
following parts of the central nervous system EXCEPT:
A. Thoracic and lumbar segments of the spinal cord
B. Sacral segments of the spinal cord
C. Midbrain
D. Cerebral cortex
E. None of the above

109. Large intestines:


A. Haustrations
B. Taenia coli
C. Appendices epiploicae
D. A and B
E. A, B and C
110. The following structure if present can cause symptoms similar to those that occur in
acute appendicitis:
A. Meckel’s diverticulum
B. Remnants of the primitive streak
C. Epoophoron
D. Thyro- glossal cyst
E. None of the above

111. Find the TRUE statement(s):


A. Lymphatic drainage of the testes is likely to go to the superficial inguinal nodes
B. Hydrocoele refers to the accumulation of fluid in the tunica vaginalis
C. Traumatic injury causing bleeding into the testes can cause destruction of the
seminiferous tubules
D. A and C
E. B and C

112. Retroperitoneal structures includes the following:


A. Kidneys
B. Part of the duodenum
C. The ascending colon
D. Adrenal glands
E. all of the above

113. The smooth muscles of the stomach are supplied by the:


A. Vagus nerve
B. Splanchnic nerve
C. Somatic nerves
D. A and B
E. B and C

114. The ischiorectal fossa contains the:


A. Pudendal nerve
B. Internal pudendal artery
C. Internal pudendal vein
D. A and C
E. A, B and C

115. The commonest site of location of the vermiform appendix is:


A. Retrocaecal
B. Paracaecal
C. Paraileal
D. Pelvic
E. Ileal
116. Find the CORRECT statement(s):
A. The proximal 2/3 of the transverse colon is supplied by the left colic artery
B. Descending colon is innervated by the vagus nerve
C. Ascending colon is supplied by the middle colic artery
D. A and B
E. None of the above

117. The following statement(s) is/are true EXCEPT:


A. In abnormal situation, the kidneys may receive two renal arteries
B. Animals that conserve water have long loop of henle
C. The left renal vein receive the left gonadal vein
D. B and C
E. None of the above

118. Regarding the spleen, find the TRUE statement(s):


A. Gastrosplenic ligamentis attached to the greater curvature
B. The lienorenal ligament is associated with the splenic vessels
C. Accessory spleen exists in the peritoneal cavity in about 10% of subjects
D. Splenomegaly is commonly associated with chronic malaria infection
E. All of the above

119. Regarding the pancreas, the following statement(s) is/are TRUE:


A. Its neck is closely related to the portal vein and the beginning of superior
mesenteric artery
B. The tail is closely related to the hilum of the spleen
C. The head is related to the second part of the duodenum
D. A and B
E. A, B and C

120. The renal arteries arises from the:


A. Ovarian artery
B. Abdominal aorta
C. Inferior mesenteric artery
D. Celiac trunk
E. None of the above

121. The lumbar plexus is formed from the anterior rami of the following spinal cord
segments:
A. L1,2,3 and 4
B. T12, L1,2,3 and 4
C. T12,L1,2 and 3
D. L1,2,3,4 and 5
E. L1,2,3,4,5 and S1
122. The portal-systemic anastosomosis:
A. Occurs if there is obstruction to blood flow in the liver
B. May cause engorgement of vessels in the anterior abdominal wall
C. May lead to appearance of blood in the stool
D. A and C
E. A, B and C

123. Regarding the peritoneum, the following are TRUE:


A. It plays an important role in limiting the spread of infection
B. The visceral peritoneum is supplied by autonomic nerves
C. Parietal peritoneum is supplied by spinal nerves
D. A and B
E. A, B and C

124. Rectus abdominis muscle:


A. Is inserted at the symphisis pubis and iliac crest
B. Has its origin at the 5th, 6th, and 7th costal cartilage
C. One of its action is flexion of the vertebral column
D. Its origin includes also the xiphoid process
E. All of the above

125. The conjoint tendon is formed by the following muscles:


A. Transverse abdominis and external oblique
B. Transverse abdominis and internal oblique
C. External oblique and internal oblique
D. Internal oblique and rectus abdominis
E. Internal oblique, transverse abdominis and external oblique

126. The vulva includes the following:


A. Labia majora
B. Labia minora
C. Vagina
D. A and B
E. B and C

127. Find the CORRECT statement(s):


A. Enlarged prostate can cause urinary obstruction
B. Ejaculatory duct opens at the penile urethra
C. Vasectomy interferes with erection and ejaculation
D. A and B
E. A, B and C
128. Find the CORRECT statement(s):
A. Normally the uterus in most women is in anteverted, anteflexed position
B. The uterus is related anteriorly to the urinary bladder and posteriorly to the
rectum and anal canal
C. Fallopian tubes cannot support the pregnancy to term
D. A and B
E. A,B and C

129. Accessory genital glands include the following:


A. Bulbourethral glands
B. Prostate gland
C. Seminal vesicles
D. B and C only
E. A, B and C

130. The vagina is lined by:


A. Stratified squamous non-keratinized epithelium
B. Stratified squamous keratinized epithelium
C. Transitional epithelium
D. Simple columnar epithelium
E. Columnar epithelium with mucous secreting cells

131. External anal sphincter contains:


A. Skeletal muscles
B. Smooth muscles
C. Skeletal and smooth muscles
D. Mainly connective tissue fibers
E. None of the above

132. The symphisis pubis is:


A. Synovial joint and allows slight movements
B. Fibrous joint
C. Gliding type of joint
D. Cartilaginous joint
E. None of the above

133. The most mobile part of the large intestine is the:


A. Ascending colon
B. Descending colon
C. Caecum
D. Transverse colon
E. None of the above
134. The uterus is partly supported by the following:
A. Transverse cervical ligaments
B. Pubocervical ligaments
C. Sacrocervical ligaments
D. Round ligament of the uterus
E. All of the above

135. The following are the features of the female pelvis


A. The sacrum is shorter and wider
B. The subpubic angle is more rounded and wider
C. The pelvic cavity is shorter and wider
D. A and C
E. A,B and C

136. Regarding the ureters, the following is/are TRUE:


A. It has three constrictions along its course which can be clinically significant
B. It receive blood from testicular (ovarian), renal and superior vesical arteries
C. It is lined by transitional epithelium
D. In its course, it is crossed by the uterine artery
E. All of the above

137. Regarding the urinary bladder, the following statement(s) is/are TRUE:
A. The neck of the bladder is continous with the prostatic urethra
B. In obstructed labor it may sustain injuries that may give rise to vesico-vaginal
and/or rectovesical fistulae
C. The neck is closely associated with prostate gland and seminal vesicles
D. It is supplied by the branches of the internal iliac artery
E. All of the above

138. Regarding the liver Ito cells are found in the:


A. Perisinusoidal space of Disse
B. Wall of sinusoid
C. Around the central vein
D. At the portal triad only
E. None of the above

139. The following statement (s) is/are TRUE:


A. The placenta is expelled during the third stage of labor
B. The umbilical cord normally contains two arteries and one vein
C. The maternal surface of the placenta presents with cotyledons
D. Amnion is the innermost fetal membrane that encloses the amniotic fluid and
fetus
E. All of the above
140. Peroneus tertius is supplied by the:
A. Superficial peroneal nerve
B. Deep peroneal nerve
C. Tibial nerve
D. Sural nerve
E. None of the above

141. The following muscles extend the foot at the ankle joint:
A. Tibialis anterior
B. Extensor hallucis longus
C. Extensor digitorum longus
D. A and C
E. A, B and C

142. The root value of femoral nerve is:


A. L2, L3 and L4
B. L4, L5, S1 and S2
C. L2, L5, S1, S2 and S2
D. L2, L2, L4, L5 and S1
E. L1, L2, L3 and L4

143. Regarding the eosophagus, the following statement(s) is/are TRUE:


A. Is continous above with the pharynx opposite the sixth cervical vertebra
B. It is lined mainly by stratified squamous epithelium
C. Its muscular wall contain both skeletal and smooth muscles
D. It has submucosal glands
E. All of the above

144. Branches of the anterior division of the internal iliac artery are:
A. Inferior gluteal artery
B. Superior gluteal artery
C. Obturator artery
D. A and B
E. A, B and C

145. The spermatic cord contains the following EXCEPT:


A. Testicular artery
B. Vas deferens
C. Pampiniform plexus
D. Artery to the vas deferens
E. None of the above

146. Payer’s patches are associated with the following organ:


A. Duodenum
B. Jejunum
C. Stomach
D. Ileum
E. Duodenum and ileum

147. Quadriceps femoris muscle consists of the following parts:


A. Rectus femoris
B. Vastus lateralis
C. Vastus medialis
D. Vastus intermedius
E. All of the above

148. Sartorius is supplied by:


A. Femoral nerve
B. Sciatic nerve
C. Branches from lumbar plexus
D. Obturator nerve
E. None of the above

149. Gluteus medius is supplied by the:


A. Inferior gluteal nerve
B. Superior gluteal nerve
C. Inferior and superior gluteal nerve
D. First and second sacral nerves
E. Obturator nerve

150. Regarding the knee joint, find the CORRECT statement(s):


A. The anterior cruciate ligament is attached to the posterior intercondylar area of
tibia
B. The posterior cruciate ligament is attached to the posterior intercondylar area
of tibia
C. The semilunar cartilages are attached both to the posterior and anterior
intercondylar areas
D. A and B
E. B and C
MUHIMBILI UNIVERSITY OF HEALTH AND
ALLIED SCIENCES
SCHOOL OF DENTISTRY
FEBRUARY 2008 UNIVERSITY EXAMINATIONS

SEMESTER ONE EXAMINATIONS FOR THE DEGREE OF


DOCTOR OF DENTAL SURGERY (DDS)
ANATOMY PAPER I (AN 100)
DATE: 11TH FEBRUARY, 2008 TIME: 9:00 – 12:00HRS

CANDIDATE NO:………………………………………………………………………………

MULTIPLE CHOICE QUESTIONS

INSTRUCTIONS: ANSWER ALL QUESTIONS

For each multiple choice question choose the one the most appropriate answer and circle the preceding
letter (1 mark each).

NOTE: ¼ a mark will be deducted for each wrong attempt

1. Organelle(s) involved with intracellular calcium ion storage is/are:


A. Endoplasmic reticulum
B. Mitochondria
C. Golgi apparatus
D. A and B
E. A, B and C
2. The following statement(s) is/are TRUE:
A. Most cell receptors are made up of transmembrane proteins
B. Membrane carbohydrates are located on the external surface of the plasma membrane
C. Membrane cholesterol increases flexibility and mechanical strength of the plasma
membrane
D. A and C
E. A, B and C
3. Growth of the microtubules occurs from the following organelle:
A. Endoplasmic reticulum
B. Mitochondria
C. Peroxisomes
D. Centrioles
E. Peroxisomes
4. The cell cortex is made up mainly of the following type of the cytoskeleton:
A. Intermediate filaments
B. Actin filaments
C. Microtubules
D. Lamins proteins
E. None of the above
5. Intracellular cell transport system is mainly forced by the following:
A. Microtubules
B. Intermediate filaments
C. Actin filaments
D. Multipass transmembrane proteins
E. None of the above
6. Oxidative detoxification of organic compounds such as alcohol and phenols occurs in the
following organelle(s):
A. Centrioles
B. Basal bodies
C. Peroxisomes
D. A and B
E. A, B and C
7. Regarding the Golgi apparatus, the following statement(s) is/are TRUE:
A. The Cis-compartment receive materials from the endoplasmic reticulum
B. The trans-compartment is associated with secretory vesicles
C. Transfer of materials to the Golgi is done by transport vesicles
D. A and C
E. A, B and C
8. During mitosis the nuclear envelope disappears at:
A. Prophase
B. Interphase
C. Metaphase
D. Anaphase
E. It does not disappear
9. Mitotic spindle includes:
A. Polar microtubules
B. Astral microtubules
C. Kinetochore microtubules
D. A and C
E. A, B and C
10. Examples of proteins that form intermediate filaments include:
A. Keratin
B. Vimentin
C. Desmin
D. Neurofilaments
E. All of the above
11. Regarding the bone, find the CORRECT statement(s);
A. It is surrounded by fibrous tissue known as periosteum
B. Bone growth occurs at the epyphyseal cartilage
C. Red bone marrow contain erythroblasts,myeloblasts and megakaryoblasts
D. A and B
E. A,B and C
12. The following cells are found in the bone EXCEPT:
A. Osteocytes
B. Chondrocytes
C. Osteoclasts
D. Osteoblasts
E. Osteoprogenitor cells
13. The root value of ulnar nerve is:
A. C8,T1
B. C5,6 and 7
C. C7,8 and T1
D. C4,5 and 6
E. None of the above
14. Bones of the shoulder girdle are:
A. Clavicle
B. Scapula
C. Humerus
D. A and B
E. A, B and C
15. The commonly occurring cells in connective tissue are the:
A. Fibroblasts
B. Neuronal cells
C. Mast cells
D. Eosinophils
E. None of the above

16. Connective tissue macrophages arises from:


A. Blood monocytes
B. Mast cells
C. Neutrophils
D. Adipocytes
E. None of the above

17. The function of the macrophages include:


A. Phagocytosis
B. Antigen presentation
C. Production of some trophic (growth) factors
D. A and B
E. A, B and C
18. Regarding the cardiac muscle cells, the following is/are TRUE:
A. Have centrally placed nuclei
B. Are connected by the intercalated discs
C. Desmosomes are the commonest intercellular junctions
D. Presence of gap junction allows electrical coupling among cells
E. All of the above

19. Smooth muscles cells are supplied by:


A. Somatic nerves
B. Visceral nerves
C. Both viscera and somatic nerves
D. A and B
E. A, B and C
20. Skeletal muscles cells:
A. Are multinucleated
B. Shows striations the cytoplasm
C. Are supplied by the somatic nerves
D. A and B
E. A, B and C
21. Nissl granules are seen in:
A. Neuronal cells
B. Motor neurons only
C. Sympathetic neurons only
D. Sensory neurons
E. Dendrites
22. Neurons of brain and spinal cord:
A. Differentiates from the neuroblasts of the neural tube
B. Contain neurofilament in their cytoplasm
C. Communicate via synapse
D. A and B
E. A, B and C
23. The vascular endothelium is made up of the:
A. Pseudostratified cells
B. Simple squamous cells
C. Columnar cells
D. Cuboidal cells
E. None of the above

24. Regarding the epithelial tissue, the following statement(s) is/are TRUE:
A. It forms the glandular tissue
B. It is associated with the basement membrane
C. Normally the cells do not have a direct contact with blood vessels
D. In simple epithelium the cells have apical and basolateral surfaces
E. All of the above
25. The gingiva and hard palate are lined by:
A. Stratified squamous keratinized epithelium
B. Simple columnar epithelium
C. Keratinized epithelium
D. A and B
E. A, B and C
26. Ovarian cycle is associated with:
A. Development of follicles
B. Secretion of female sex hormones
C. The luteal phase
D. A and B
E. A, B and C
27. Regarding the fertilization, the following statement(s) is/are TRUE:
A. It normally occur in the fallopian tubes
B. It involve enzymes present in the acrosome
C. One oocyte is normally penetrated by a single spermatozoon
D. A and B
E. A, B and C
28. Regarding the cleavage division, the following statement (s) is/are TRUE:
A. It begins immediately after fertilization
B. The resulting cells are known as blastomeres
C. Is associated with the synthesis of cytoplasm
D. A and B
E. A, B and C
29. The following features are associated with the implantation:
A. Loss of zona pellucida
B. Completion of the first meiotic division
C. Completion of second meiotic division
D. A and B
E. A, B and C

30. Spermatogenic cells of the seminiferous tubules include the following:


A. Spermatogonia
B. Primary and secondary spermatocytes
C. Myoid cells
D. A and B
E. A, B and C

31. Abnormal implantation can result into:


A. Tubal pregnancy
B. Abdominal pregnancy
C. Placenta praeviae
D. A and B
E. A, B and C
32. Find the TRUE statement(s):
A. The zona pellucida is specie specific; it limits fertilization by spermatozoon from other
species
B. Final maturation of the spermatozoa occurs in the epididymis
C. Genetic sex is determined soon after implantation
D. A and B
E. None of the above

33. During embryonic development, the germ cells of the gonads arise from the:
A. Wall of the yolk sac
B. Embryonic mesoderm
C. Ectoderm
D. Neural crest cells
E. None of the above
34. The following feature(s) is/are associated with folding of the embryo:
A. The yolk sac is incorporated to form the primitive gut
B. The heart come to lie caudal to the septum transversum
C. The developing brain assume the most cranial position
D. A and B
E. A, B and C
35. Find the CORRECT statement(s)
A. The neural tube develop embryonic ectoderm
B. Adrenal medulla arise from migrated neural crest cells
C. The intermediate mesoderm is also called nephrogenic mesoderm
D. Splanchopleuric mesoderm give rise to smooth muscle of the gut
E. All of the above
36. The parts of the primitive heart tube include:
A. Sinus venosus
B. Bulbus cordis
C. Truncus arteriosus
D. B and C
E. A, B and C
37. During the development truncus arteriosus of the primitive heart tube will give rise to the:
A. Trabeculated part of the right atrium
B. Smooth part of the left ventricle
C. Aorta and pulmonary trunk
D. Coronary sinus
E. None of the above
38. Regarding the development of the heart:
A. Septum primum is associated with the development of the ventricular septum
B. Atrial septum has membranous and muscular portions
C. Foramen ovale is associated with the inter- atrial septum
D. A and B
E. None of the above
39. Regarding the congenital anomalies of the heart patent ductus arteriosus (PDA) occurs:
A. Between the right and left atria
B. Between the pulmonary trunk and the arch of aorta
C. Between the right and left ventricles
D. Between the arch of aorta and subclavian artery
E. None of the above
40. The following statement(s) is/are TRUE:
A. Polyhydramnios may be associated with esophageal atresia
B. Spinal bifida occulta is an example of neural tube defects
C. Elevated levels of α-fetoprotein is commonly associated with the presence of neural tube
defects
D. A and B
E. A, B and C
41. During embryonic development the structures derived from the foregut include:
A. Liver
B. Proximal duodenum
C. Trachea and lungs
D. Oesophagus
E. All of the above
42. The cartilage of the first arch is:
A. Hyoid cartilage
B. Meckel’s cartilage
C. Reichet’s cartilage
D. Maxillary
E. None of the above
43. The following conditions are associated with the maldevelopment of the facial processes:
A. Bilateral cleft lip
B. Cleft palate
C. Oblique facial cleft
D. A and B
E. A, B and C
44. The hard palate develop from the:
A. Horizontal palatine shelves
B. Pre-maxilla
C. Meckel’s cartilage
D. A and B
E. A, B and C

45. The tongue develop from the following embryonic structures:


A. Lingual swellings
B. Tuberculum impar
C. Hypobranchial eminence
D. A and B
E. A and C
46. Factors that are secreted during the development of the reproductive system include:
A. Mullerian- inhibiting factor
B. Testes determining factor
C. Sry gene
D. A and B
E. None of the above
47. During the embryonic development the thyroid gland, it originates:
A. From the foramen cecum as the tongue develops
B. The diverticulum called thyroglossal duct
C. Hypobranchial eminence
D. A and B
E. A, B and C
48. The root of the tooth develop from the :
A. Hertwig’s epithelial toor sheath
B. Dental papilla
C. Dental sac
D. A and B
E. A, B and C
49. Physiological umbilical hernia occurs as a result of:
A. Development of the heart
B. Development of the lungs
C. Development of the midgut loop
D. Rotation of the embryo
E. None of the above
50. During the embryonic development, the ureteric bud will from the:
A. Proximal convoluted tubule
B. Major and minor calyces
C. Loop of henle
D. Renal glomerulus
E. All of the above
51. Cells of the outer cell mass that show invasive properties during the implantation are:
A. Cytotrophoblasts
B. Syncytiotrophoblasts
C. Epiblasts
D. Hypoblasts
E. Decidual cells

52. During development the notochord will give rise to the following adult structure:
A. Nucleus polposus
B. Somites
C. Myotome
D. Ribs
E. Muscles of the back
53. The following conditions may be associated with male infertility
A. Oligospermia
B. Azoospermia
C. Immotile and abnormal spermatozoa
D. A and B
E. A, B and C
54. Biceps brachii muscle is supplied by the :
A. Radial nerve
B. Axillary nerve
C. Musculocutaneous nerve
D. Ulnar nerve
E. Median nerve
55. Regarding the shoulder joint, the following statement(s) is/are TRUE:
A. The tendon of the long head of biceps is intracapsular
B. Articulating bones include the head of humerus and glenoid cavity of the scapula
C. It shows a wide range of movement
D. A and B
E. A, B and C

56. Regarding the joints of the upper limb, the following statement(s) is/are TRUE:
A. Sternoclavicular joint has an articular disc
B. Acromioclavicular joint is a plane synovial type of joint
C. The shoulder joint is least supported at its inferior surface
D. A and B
E. A, B and C

57. Regarding the axillary lymph node, the following statement(s) is/are TRUE:
A. They drain lymph from the lateral part of the breast
B. They drain lymph from the thoracic wall
C. They receive lymph from the upper limb
D. A and B
E. A, B and C
58. Trapezius muscle is innervated by:
A. Ulnar nerve
B. Accessory
C. Radial
D. Axillary
E. Brachial plexus
59. The veins of the upper limb include:
A. Basilica vein
B. Cephalic vein
C. Median cubital vein
D. A and B
E. A, B and C
60. Find the CORRECT statement (s):
A. The apex beat is normally palpated at the fifth intercostals space in the midclavicular line
B. Coronary arteries arise from the ascending aorta
C. Coronary sinus opens into the left atrium
D. A and B
E. A, B and C
61. The terminal branches of internal thoracic artery are:
A. Superior epigastric artery
B. Musculophrenic artery
C. Pericardiacophrenic artery
D. A and B
E. B and C
62. The diaphragm:
A. Arises from the xiphoid process, lower six costal cartilages and three lumbar vertebrae
B. Is supplied by the fibers that come from C3, 4 and 5
C. Its fibers is inserted into the central tendon
D. A and B
E. A, B and C

63. Presence of air in the thoracic cavity is clinically referred to as:


A. Pneumothorax
B. Haemothorax
C. Pleural effusion
D. B and C
E. None of the above
64. The right atrioventricular orifice is guarded by :
A. A valve with three cusps
B. A valve with two cusps
C. A valve with no cusps
D. Endocardial folds
E. None of the above

65. Anomalies of the lung may include the following:


A. Presence of incomplete major fissures
B. Presence of minor fissures
C. Lobation anomalies
D. A and B
E. A, B and C
66. In normal anatomy, the left lung has:
A. Three secondary bronchi
B. Two secondary bronchi
C. One secondary bronchus
D. Four bronchopulmonary segments
E. None of the above
67. The thoracic duct terminated by opening into the:
A. Right atrium directly
B. Left brachiocephalic vein
C. Left atrium directly
D. Coronary sinus
E. None of the above
68. The following statement(s) is/are TRUE:
A. The oesophagus has three anatomical constrictions that may be of clinical importance
B. The right main bronchus is shorter and wider than the left main bronchus
C. The lungs lies in the mediastinum
D. A and B
E. A, B and C
69. Regarding the openings in the diaphragm:
A. The aortic opening lies at the level of the 8th thoracic vertebra
B. The oesophageal opening is at the level of the 12th thoracic vertebra
C. The aortic opening transmits the aorta, thoracic duct and the azygous vein
D. A and B
E. B and C
70. Regarding the Pterion , find the CORRECT statement(s):
A. Is a point where the greater wing of the sphenoid bone meet with the antero-inferior
corner of the parietal bone
B. Beneath it, lies the anterior branch of middle meningeal artery
C. It is the thinnest part of the lateral wall of the skull
D. A and B
E. A, B and C
71. The following statement(s) is/are TRUE:
A. Optic nerve contains motor fibers to the extraocular muscles of the eye
B. The anterior fontanelle closes approximately at the age of 6 months
C. The parotid duct opens into the mouth opposite the upper second molar tooth
D. A and B
E. A, B and C
72. The cervical vertebrae:
A. Are 8 in number
B. Have foramina in the transverse processes
C. Have broader, short and non-bifid spinous process
D. A and B
E. B and C
73. Regarding the temporomandibular joint, the mandibular head articulates with the:
A. Mandibular fossa of the sphenoid bone
B. Mandibular fossa of the temporal bone
C. Mandibular fossa of the sphenoid bone
D. Greater wing of the sphenoid bone
E. None of the above
74. The muscles of mastication are innervated by the following nerve:
A. Facial
B. Mandibular
C. Maxillary
D. Vagus
E. Hypoglossal
75. Regarding the tongue, the lingual papillae are located on the:
A. Ventral surface
B. Posterior one third only
C. Dorsal surface
D. On the dorsal and ventral surface of the anterior two third
E. None of the above
76. The salivary glands include the following:
A. The parotid glands
B. The submandibular glands
C. Smaller glands located beneath the oral mucosa
D. A and B
E. A, B and C
77. Buccinator muscle is supplied by the:
A. Hypoglossal nerve
B. Glossopharyngeal nerve
C. Facial nerve
D. Lingual nerve
E. None of the above

78. The Waldeyer’s ring consists of the following:


A. Tubal tonsils
B. Naso-pharyngeal tonsils (adenoid)
C. Lingual tonsils
D. Palatine tonsils
E. All of the above

79. Arterial pulsation can be easily felt by palpating the following vessel:
A. Facial artery
B. Superficial temporal artery
C. Maxillary artery
D. A and B
E. None of the above
80. Muscles of facial expression include the following:
A. Platysma
B. Occipito-frontalis
C. Buccinator
D. Orbicularis oculi
E. All of the above
81. Lingual tonsils are located in the:
A. Anterior two third of the tongue
B. Floor of the mouth
C. Posterior one third of the tongue
D. Lower lips
E. None of the above
82. The posterior cranial fossa contains:
A. Medulla oblongata
B. Cerebellum
C. Pons
D. A and B
E. A, B and C
83. The dental sac give rise to all of the following except:
A. Cementum
B. Periodontal ligament
C. Dentin
D. Alveolar process
E. Gingival

84. The following are facts during shedding:


A. Deciduous tooth undergo root resorption
B. Tooth buds for successors move apically and lingually
C. Deciduous teeth are known as predecessors
D. A and B
E. A, B and C

85. The congenital anomalies of the kidney include the following:


A. Presence of polar vessels
B. Supernumerary renal vessels
C. Horseshoe kidneys
D. A and C
E. A, B and C
86. The common maxillary sinus:
A. Middle meatus
B. Inferior meatus
C. Superior meatus
D. Oral cavity
E. None of the above
87. The tongue is innervated mainly by the:
A. Trigeminal nerve
B. Glossopharyngeal nerve
C. Facial nerve
D. A and B
E. A, B and C
88. This/these is/are among the structures contained in the infratemporal fossa
A. Mandibular nerve
B. Maxillary artery pudendal artery
C. Pterygoid venous plexus
D. Pterygoid muscle
E. All of the above
89. The stylomandibular ligament originates from the:
A. Styloid process
B. Mastoid process
C. Lingual
D. Medial pterygoid plate
E. None of the above
90. Find the CORRECT statement(s):
A. The oral vestibule lies lateral to the teeth
B. The parotid duct opens opposite to the upper second molar tooth
C. The submandibular salivary gland receive parasympathetic fibers from the facial nerve
D. A and B
E. A, B and C

91. The following statement(s) is/are TRUE:


A. The corda tympany nerve communicates with the lingual nerve
B. The corda tympany nerve is a branch of facial nerve
C. The corda tympany nerve carries parasympathetic fibers
D. A and B
E. A,B and C
92. The foramen spinosum allow passage of:
A. The middle meningeal artery
B. The mandibular nerve
C. The maxillary artery
D. The deep temporal artery
E. None of the above
93. The reduced enamel epithelium:
A. Is composed of 4 cell layers
B. Cover the enamel before the tooth emerges into the oral cavity
C. Participate in the formation of the junctional epithelium
D. A and B
E. A, B and C

94. The ophthalmic artery normally arises from the:


A. External carotid artery
B. Internal carotid artery
C. Subclavian artery
D. Common carotid artery
E. All of the above
95. The tooth socket:
A. Gomphosis type of joint
B. Condylar joint
C. Synovial joint
D. Ball and socket joint
E. None of the above
96. The odontoblasts are:
A. Dentin forming cells
B. Cementum forming cells
C. Have odontoblastic process that lies in dentinal tubules
D. A and B
E. A, B and C

97. Regarding the liver sinusoids opens into the:


A. Perisinusoidal space of Disse
B. Central veins
C. Inferior vena cava
D. Portal vein
E. None of the above
98. The periodontal ligament is including the following fibers:
A. Alveolar crest fibers
B. Inerradicula fibers
C. Periapical fibers
D. A and B
E. A, B and C
99. The following vessel(s) is/are related with the pterion:
A. Middle meningeal artery
B. Middle meningeal vein
C. Superficial temporal artery
D. A and B
E. A, B and C
100. Branches of the maxillary artery include the following EXCEPT:
A. Inferior alveolar artery
B. Middle meningeal artery
C. Superficial anterior alveolar artery
D. Ascending pharyngeal artery
E. None of the above

101. Payer’s patches forms the distinguishing features of the:


A. Jejunum
B. Duodenum
C. Ileum
D. Stomach
E. Colon
102. The root value of phrenical nerve is:
A. C2, C3 and C4
B. C3, C4 and C5
C. C2, C3,C4 and C5
D. C1 and C2
E. None of the above
103. Regarding the tooth germ, the following statement(s) is/are TRUE:
A. Dentin develops from the dental papilla
B. Dental pulp develops from the dental papilla
C. Cementum develops from the dental sac
D. A and B
E. A, B and C
104. The soft palate contains:
A. Skeletal muscles
B. Smooth muscles
C. Tensor veli palatine
D. A and C
E. None of the above
105. The costochondral joint is a:
A. Synovial joint and it allows slight movements
B. Fibrous joint
C. Gliding type of joint
D. Cartilagenous joint
E. None of the above

106. The ligament providing stability to the temporomandibular joint is the:


A. Stylomandibular ligament
B. Sphenomandibular ligament
C. Lateral temporomandibular ligament
D. Articular disc
E. None of the above
107. The first permanent mandibular molar has the following features:
A. Five cusps
B. Four cusps
C. Two roots
D. A and C
E. None of the above
108. The following are the features of maxillary permanent molars:
A. Four cusps
B. Five cusps
C. Three roots
D. A and C
E. B and C
109. Regarding the muscles of mastication the following (s) is/are TRUE:
A. Develop from the first arch
B. Supplied by trigeminal nerve
C. Buccinator is an example
D. A and B
E. B and C
110. Regarding the tongue, the following statement(s) is/are TRUE:
A. The dorsum is lined by specialized epithelium
B. The anterior two thirds contain lingual papilla
C. The posterior one third contain the lingual tonsil
D. A, B and C
E. B and C
111. The three functions of the dental organ include:
A. Formation of enamel
B. Determine the rown form
C. Induce formation of dentin
D. A and B
E. A, B and C
112. The cerebrospinal fluid enters the subarachnoid space via:
A. Three foramina located in the fourth ventricle
B. Openings in the lateral ventricle
C. Cerebral aqueduct
D. Central canal of the spinal cord
E. Third ventricle
113. The foramen magnum transmits the following EXCEPT:
A. Vertebral artery
B. Spinal cord
C. Occipital artery
D. Spinal accessory nerve
E. None of the above

114. Brunner’s glands are associated with the following organ:


A. Duodenum
B. Jejunum
C. Stomach
D. Ileum
E. Duodenum and ileum
115. The following statements is/are TRUE:
A. In fracture of the root of the tooth cementum is important in healing
B. Sclerotic dentin forms the pulpal surface
C. There is enamel formation at the cervical region
D. A and B
E. A, B and C
116. Regarding Gingiva, find the FALSE statement(s):
A. The free gingival forms the interdental papilla
B. The attached gingival is stippled
C. The gingival sulcus is about 3mm
D. It is covered by keratinizing stratified squamous epithelium
E. None of the above
117. Temporalis muscle is supplied by the:
A. Mandibular nerve
B. Maxillary nerve
C. Inferior alveolar nerve
D. Facial nerve
E. None of the above
118. The Mandibular teeth are supplied by:
A. Maxillary nerve
B. Inferior alveolar nerve
C. Mental nerve
D. Buccal nerve
E. None of the above
119. Occipito-frontalis is supplied by the:
A. Facial nerve
B. Superficial temporal nerve
C. Deep temporal nerves
D. Supraorbital nerves
E. None of the above

120. The laryngeal cartilage include:


A. Thyroid cartilage
B. Cricoid cartilage
C. Arytenoids cartilage
D. Corniculate cartilage
E. All of the above
121. Cells of the skills are located in the:
A. Dermis
B. Dermal papillae
C. Epidermis
D. Superficial fascia
E. None of the above
122. The jugular foramen transmits the following structures:
A. Sigmoid sinus
B. Glossopharyngeal nerve
C. Vagus nerve
D. Inferior petrosal sinus
E. All of the above
123. The vocal folds are covered by:
A. Ciliated columnar cells
B. Stratified squamous cells
C. Simple squamous cells
D. Pseudostratified ciliated columnar cells
E. Transitional epithelium
124. Find the CORRECT statement (s):
A. In the colon longitudinal muscle layer form the taeniae coli
B. The alveolar wall contains types I and II pneumocytes
C. Dust cells are examples of alveolar macrophages
D. A and B
E. A, B and C
125. Regarding the ovarian follicles, find the CORRECT statement(s):
A. The consists of an oocyte and the surrounding follicular cells
B. During the ovarian cycle about 20 of them are recruited to develop
C. At ovulation the surrounding follicular cells are converted into glandular tissue
D. A and B
E. A, B and C
126. Regarding the breast, the following is/are TRUE:
A. The glandular cells secrete via apocrine secretion
B. Basal surface of the glandular cells is associated with the myoepithelial cells
C. It is influenced by the hormone prolactin
D. A and B
E. A, B and C
127. The following hormone act on smooth muscles:
A. Antidiuretic hormone
B. Oxytocin
C. Erythropoietin
D. Follicle stimulating hormone
E. Luteinizing hormone
128. Regarding the brown adipose tissue, the following is/are TRUE:
A. the cells have numerous mitochondria
B. is well supplied with blood
C. metabolic activities of cells causes release of large amount of heat
D. is present in newborn babies
E. A , B and C

GOOD LUCK
CANDIDATE NO:…………………………

MUHIMBILI UNIVERSITY OF HEALTH AND ALLIED SCIENCES

SCHOOL OF MEDICINE
UNIVERSITY EXAMINATIONS - FEBRUARY 2009

FIRST YEAR END OF SEMESTER ONE EXAMINATIONS FOR THE DEGREE


OF DOCTOR OF MEDICINE

ANATOMY PAPER I (AN 100)

DATE: 16TH FEBRUARY, 2009 TIME: 9:00AM – 12:00AM

MULTIPLE CHOICE QUESTIONS

INSTRUCTIONS: ANSWER ALL QUESTIONS

For each multiple choice question choose the one the most appropriate answer and circle the preceding
letter (1 mark each).

NOTE: ¼ a mark will be deducted for each wrong attempt

1. Calcium ions are required during: 4. Tight junctions are numerous in the:
A. Cell divisions A. Mucosa of the gut
B. Secretory process B. Mucosa of the urinary bladder
C. Cell locomotion C. Endocrine glands
D. Signaling process D. A and B
E. All of the above E. A, B and C

2. The following is/are TRUE; 5. Intracellular cell transport system is


A. Cell cortex is located below the mainly formed by the following:
plasma membrane A. Microtubules
B. Membrane carbohydrates are B. Intermediate filaments
located on the external surface C. Actin filaments
of the plasma membrane D. Multipass transmembrane
C. Plasma membrane of the red proteins
blood cells is rich in cholesterol E. None of the above
D. A and C
E. A, B and C 6. Protein synthesis occurs in the following
organelles:
3. Cilia grow from the: A. Lysosomes
A. Basal bodies B. Rough endoplasmic reticulum
B. Plasma membrane C. Peroxisomes
C. Intermediate filaments D. Centrioles
D. Actin filaments E. Smooth endoplasmic reticulum
E. Peroxisomes
7. Regarding the Golgi apparatus, the 12. Cells of the cartilage are:
following statement(s) is/are TRUE : A. Osteocytes
A. The cis- compartment receive B. Chondrocytes
materials from the endoplasmic C. Osteoclasts
reticulum D. Osteoblasts
B. The trans – compartment is E. Osteoprogenitor cells
associated with secretory
vesicles 13. The following is NOT a long bone:
C. Transfer of material to the Golgi A. Radius
is done by the transport vesicles B. Ulna
D. A and C C. Scaphoid
E. A, B and C D. Metatarsal
E. Middle phalanx
8. The nuclear envelope disappears at: 14. Cells that perform bone destruction:
A. Prophase A. Osteoclasts
B. Interphase B. Osteoblasts
C. Metaphase C. Osteocytes
D. Anaphase D. A and C
E. It does not disappaear E. A, B and C

9. Mitotic spindle includes: 15. The cells that synthesize connective


A. Polar microtubules tissue fibers are:
B. Astral microtubules A. Fibroblasts
C. Kinetochore microtubules B. Neuronal cells
D. A and C C. Mast cells
E. A, B and C D. Eosinophils
E. None of the above
10. Human diploid cells have:
A. 23 chromosomes 16. The intervertebral disc contains the
B. 46 chromosomes following types of cartilage:
C. 48 chromosomes A. Fibrocartilage
D. 22 chromosomes B. Hyaline cartilage
E. None of the above C. Elastic cartilage
D. Elastic and hyaline cartilage
11. Find the CORRECT statement(s); E. None of the above
A. It is surrounded by fibrous
tissue known as periosteum 17. Macrophages of the liver are:
B. Calcium salts are deposited in A. Ito cells
the intercellulat matrix B. Dust cells
C. Red bone marrow contain C. Kupffer cells
erythroblasts,myeloblasts and D. Meckels cells
megakaryoblasts E. None of the above
D. A and B
E. A,B and C
18. Regarding the cardiac muscle cells, the A. It forms the inner lining of
following is/are TRUE: blood vessels
A. Have centrally placed nuclei B. It contains simple squamous
B. Are connected by the epithelium
intercalated discs C. It is the main component of
C. Desmosomes are the body capillaries
commonest intercellular D. A and B
junctions E. A, B and C
D. Presence of gap junction allows
electrical coupling among cells 24. The mesothelium contains the following
E. All of the above types of cells:
A. Simple columnar
19. Smooth muscles cells are supplied by: B. Simple cuboidal
A. Somatic nerves C. Simple squamous
B. Visceral nerves D. Stratified squamous
C. Both viscera and somatic nerves E. None of the above
D. A and B
E. A, B and C 25. The anal canal is lined by the:
A. Stratified squamous epithelium
20. Pseudounipolar neurons: B. Simple columnar epithelium
A. Are located in the spinal root C. Keratinized epithelium
ganglia D. A and B
B. Have central and peripheral E. A,B and C
nerve processes
C. Can also form a motor neurone 26. Testosterone is secreted by:
D. A and B A. Leydig cells
E. A, B and C B. Cells of the adrenal cortex in
small amount
21. Nissl granules are seen in: C. Sertoli cells
A. Neuronal cells D. A and B
B. Motor neurons only E. B and C
C. Sympathetic neurons only
D. Sensory neurons 27. Regarding fertilization, the following
E. Dendrites statement(s) is/are TRUE:
A. It normally occur in the
22. Neuronal cell bodies are located in: fallopian tubes
A. Cortex B. It involve enzymatic activities
B. Ganglia C. One oocyte is normally
C. Nuclei of the brain penetrated by a single
D. A and B spermatozoon
E. A, B and C D. A and B
E. A, B and C
23. Regarding the endothelium, find the
CORRECT atatement(s): 28. Cleavage division will give rise to:
A. Blastomeres C. The developing brain assume
B. Cytotrophoblasts the most cranial position
C. Blastocoele D. A and B
D. Corona radiata E. A, B and C
E. None of the above
33. Neural crest cells develop to form:
29. The following features are associated A. Adrenal cortex
with implantation: B. Adrenal medulla
A. Loss of zona pellucida C. Cerebral cortex
B. Erosion of endometrium by the D. Spinal cord
trophoblasts E. None of the above
C. Formation of deciduas basalis,
parietalis and capsularis 34. Biceps brachii muscle is supplied by the
D. A and B A. Radial nerve
E. A, B and C B. Axillary nerve
C. Musculocutaneous nerve
30. Regarding twinning, the following is/are D. Ulnar nerve
TRUE: E. Median nerve
A. Identical twins are normally of
the same sex 35. Injury to the radial nerve will cause the
B. Under normal circumstances following conditions:
each twin has its own umbilical A. Waiters-tip hands
cord B. Claw hand
C. Fertilization of two oocytes is C. Wrist drop
likely to result into identical D. Ape-hand
twins E. Carpel tunnel syndrome
D. A and B 36. The commonest dislocation position of
E. A, B and C the shoulder joint is:
A. Posterior
31. The germ cells of the embryonic gonads B. Antero-inferior
arises from the: C. Lateral
A. Wall of the yolk sac D. Postero-superior
B. Embryonic mesoderm E. Medial
C. Ectoderm 37. Regarding the axillary lymph node, the
D. Neural crest cells following statement is/are TRUE:
E. None of the above A. They drain lymph from the
lateral part of the breast
32. The following feature(s) is/are B. They drain lymph from the
associated with folding of the embryo: thoracic wall
A. The endoderm is incorporated to C. They receive lymph from the
form the primitive gut upper limb
B. The heart come to lie caudal to D. A and B
the septum transversum E. A, B and C
38. The third part of the axillary artery give B. Is supplied by the fibers that
off the following branches(s): come from C3, 4 and 5
A. Subscapular artery C. Its fibers is inserted into the
B. Anterior circumflex humeral central tendon
artery D. A and B
C. Posterior circumflex humeral E. A, B and C
artery
D. A and B 43. Presence of air in the thoracic cavity is
E. A, B and C clinically referred to as:
A. Pneumothorax
39. Which of the following muscles belong B. Haemothorax
to the group of rotator cuff muscles: C. Pleural effusion
A. Pectoralis major D. B and C
B. Teres major E. None of the above
C. Teres minor
D. Pectoralis minor 44. Regarding the Pterion , find the
E. Rhomboid major CORRECT statement(s):
A. Is a point where the greater
40. Find the CORRECT statement (s): wing of the sphenoid bone meet
A. The apex beat is normally with the antero-inferior corner
palpated at the fifth intercostals of the parietal bone
space in the mid-clavicular line B. Beneath it, lies the anterior
B. The arch of the aorta lies behind branch of middle meningeal
the manubrium sterni artery
C. The rami communicates connect C. It is the thinnest part of the
the intercostals nerve to a lateral wall of the skull
ganglion of the sympathetic D. A and B
chain E. A, B and C
D. A and B
E. A, B and C 45. The following statement (s) is/are
TRUE:
41. The branches of internal thoracic artery A. The maxillary air sinuses lie
is/are: below the infraorbital foramen
A. Pericardiacophrenic artery on each side
B. Superior epigastric artery B. The parotid duct opens into the
C. Musculophrenic artery mouth opposite the upper
D. Anterior intercostal artery second molar tooth
E. All of the above C. The anterior frontanelle closes
approximately at the age of 18
42. The diaphragm: months
A. Arises from the xiphoid process, D. A and B
lower six costal cartilages and E. A, B and C
three lumbar vertebrae
46. The cervical vertebrae:
A. Are 8 in number 52. The Waldeyer’s ring consists of the
B. The 8 cervical spinal nerves following:
corresponds to each vertebra A. Tubal tonsils
C. Vertebral artery ascends in the B. Naso-pharyngeal tonsils
transverse foramina (adenoid)
D. One of the typical characteristic C. Lingual tonsils
is long spinous process D. Palatine tonsils
E. All of the above E. All of the above

47. Extrinsic muscles of the tongue are: 53. The floor of the oral cavity is formed by
A. Genioglossus the following:
B. Hyoglossus A. Mylohyoid
C. Styloglossus B. Anterior belly of the digastric
D. A and B C. Genioglossus
E. A, B and C D. Styloglossus
48. The impression unique to the left lung is E. All of the above
the: 54. Muscles of facial expression include the
A. Cardiac notch following:
B. Diaphragmatic notch A. Platysma
C. Phrenic notch B. Occipito-frontalis
D. Vagus notch C. Buccinator
E. None of the above D. Orbicularis oculi
49. The digasric muscles is supplied by:
A. Trochlear nerve 55. The infratemporal fossa contains the
B. Facial nerve following structures EXCEPT:
C. Hypoglossal nerve A. Deep temporal nerve
D. Glossopharyngeal nerve B. Deep temporal artery and vein
E. None pf the above C. Superficial temporal artery
50. The salivary glands include the D. Auriculotemporal nerve
following: E. None of the above
A. The parotid glands
B. The submandibular glands 56. The following statement(s) is/are
C. Smaller glands located beneath CORRECT:
the oral mucosa A. Fracture of the palate can cause
D. A and B airway obstruction
E. A, B and C B. Scalp infection can spread to the
51. The sternocleidomastoid muscles: brain
A. Demarcates the anterior and C. Injury to the scalp can cause
posterior triangles of the neck profuse bleeding
B. Is supplied by the vagus nerve D. Infection in the nasal cavity can
C. Is supplied by the accessory spread to the paranasal air
nerve sinuses
D. A and B E. All of the above
E. A and C
57. Branches of the mandibular division of 63. Regarding the glial cells, the phagocytic
the trigeminal nerve include: cells are:
A. A meningeal brand A. Astrocytes
B. Nerve to the medial pterygoid B. Oligodendrocytes
C. Masseteric nerve C. Microglia
D. Auriculotemporal nerve D. Ependymal cells
E. All of the above E. Tanycytes

58. The foramen magnum transmits the 64. Large intestines have:
following structures: A. sacculations
A. The medulla oblongata and its B. Taenia coli
surrounding meninges C. Appendices epiploicae
B. Vertebral arteries D. A and B
C. Ascending spinal part of the E. A, B and C
accessory nerve 65. Caput medusa is the name given to the
D. A and B enlargement of the veins at the
E. A, B and C following site:
A. Around umbilicus
59. Purkinje fibers are found in the: B. In the scrotum
A. Cerebral cortex C. Deep part of the neck
B. Cerebellar cortex D. Lower oesophagus
C. Spinal cord E. None of the above
D. Medulla oblongata 66. Find the TRUE statement(s):
E. None of the above A. Lymphatic drainage of the testes
60. The outermost meningeal layer is known is likely to go to the superficial
as: inguinal nodes
A. Dura matter B. Hydrocoele refers to the
B. Arachnoid matter accumulation of fluid in the
C. Pia matter tunica vaginalis
D. Pia-arachnoid C. Traumatic injury causing
E. None of the above bleeding into the testes can
61. The lateral ventricles are located in the: cause destruction of the
A. Pons seminiferous tubules
B. Medulla oblongata D. A and C
C. Midbrain E. B and C
D. Cerebral hemisphere
E. Cerebellum 67. Retroperitoneal structures includes the
62. Peripheral nerves are myelinated by the: following:
A. Oligodendrocytes A. Kidneys
B. Schwann cells B. Part of the duodenum
C. Microglia C. The ascending colon
D. Oligodendrocytes and Schwann D. Adrenal glands
cells E. all of the above
E. Tanycytes
68. The smooth muscles of the stomach are 73. Regarding the spleen, find the TRUE
supplied by the: statement(s):
A. Vagus nerve A. Gastrosplenic ligamentis
B. Splanchnic nerve attached to the greater curvature
C. Somatic nerves B. The lienorenal ligament is
D. A and B associated with the splenic
E. B and C vessels
69. The ischiorectal fossa contains the: C. Accessory spleen exists in the
A. Pudendal nerve peritoneal cavity in about 10%
B. Internal pudendal artery of subjects
C. Internal pudendal vein D. Splenomegaly is commonly
D. A and C associated with chronic malaria
E. A, B and C infection
70. The pelvic diaphragm is formed by the E. All of the above
following muscles/structures:
A. Perineal membrane 74. Regarding the pancreas, the following
B. Levator ani statement(s) is/are TRUE:
C. Obturator internus A. Its neck is closely related to the
D. Superficial and deep transverse portal vein and the beginning of
perineal membrane superior mesenteric artery
E. Sphincter urethrae and deep B. The tail is closely related to the
transverse perineal muscles hilum of the spleen
C. The head is related to the
71. Find the CORRECT statement(s): second part of the duodenum
A. The proximal 2/3 of the D. A and B
transverse colon is supplied by E. A, B and C
the left colic artery
B. Descending colon is innervated 75. The anterior branch (es) of the
by the vagus nerve abdominal aorta is/are:
C. Ascending colon is supplied by A. Ovarian artery
the middle colic artery B. Testicular artery
D. A and B C. Inferior mesenteric artery
E. None of the above D. Renal artery
72. The following statement (s) is/are true E. All of the above
EXCEPT:
A. The kidneys occupy 76. The lumbar plexus is formed from the
approximately three lumbar anterior rami of the following spinal
vertebrae cord segments:
B. Animals that conserve water A. L1,2,3 and 4
have long loop of henle B. T12, L1,2,3 and 4
C. Diseases of the kidney can lead C. T12,L1,2 and 3
to anaemia D. L1,2,3,4 and 5
D. B and C E. L1,2,3,4,5 and S1
E. None of the above
77. The portal-systemic anastosomoses: E. Internal oblique, transverse
A. Occurs if there is obstruction to abdominis and external oblique
blood flow in the liver
B. May cause engorgement of 81. The vulva includes the following:
vessels in the anterior A. Labia majora
abdominal wall B. Labia minora
C. May lead to anastomosis C. Vagina
between the superior and middle D. A and B
rectal veins E. B and C
D. A and C
E. A, B and C 82. Find the CORRECT statement(s):
A. Enlarged prostate can cause
78. Regarding the peritoneum, the following urinary obstruction
are TRUE: B. Ejaculatory duct opens at the
A. It plays an important role in penile urethra
limiting the spread of infection C. Vasectomy interferes with
B. The visceral peritoneum is erection and ejaculation
supplied by autonomic nerves D. A and B
C. Parietal peritoneum is supplied E. A, B and C
by spinal nerves
D. A and B 83. Find the CORRECT statement(s):
E. A, B and C A. Normally the uterus in most
women is in anteverted,
79. Rectus abdominis muscle: anteflexed position
A. Is inserted at the symphisis B. The uterus is related anteriorly
pubis and iliac crest to the urinary bladder and
B. Has its origin at the 5th, 6th, and posteriorly to the rectum and
7th costal cartilage anal canal
C. One of its action is flexion of C. Fallopian tubes cannot support
the vertebral column the pregnancy to term
D. Its origin includes also the D. A and B
xiphoid process E. A,B and C
E. All of the above
84. Accessory genital glands include the
80. The conjoint tendon is formed by the following:
following muscles: A. Bulbourethral glands
A. Transverse abdominis and B. Prostate gland
external oblique C. Seminal vesicles
B. Transverse abdominis and D. B and C only
internal oblique E. A, B and C
C. External oblique and internal
oblique 85. The vagina is lined by:
D. Internal oblique and rectus A. Stratified squamous non-
abdominis keratinized epithelium
B. Stratified squamous keratinized 91. The uterine cervix is lined by the
epithelium following:
C. Transitional epithelium A. Stratified squamous cells
D. Simple columnar epithelium B. Simple columnar cells
E. Columnar epithelium with C. Simple cuboidal cells
mucous secreting cells D. A and B
E. A, B and C
86. External anal sphincter contains: 92. Regarding the urinary bladder, the
A. Skeletal muscles following statement(s) is/are TRUE:
B. Smooth muscles A. The neck of the bladder is
C. Skeletal and smooth muscles continuous with the prostatic
D. Mainly connective tissue fibers urethra
E. None of the above B. In obstructed labor it may
87. Which of the following structure is part sustain injuries that may give
of or is formed by the internal oblique rise to vesico-vaginal and/or
muscle: rectovesical fistulae
A. Lacunar ligament C. The neck is closely associated
B. Inguinal ligament with the prostate gland and
C. Cremasteric muscle seminal vesicles
D. External spermatic fascia D. It is supplied by the branches of
E. Internal spermatic fascia the internal iliac artery
88. The most mobile part of the large E. All of the above
intestine is the: 93. Pregnancy that occur outside the uterine
A. Ascending colon cavity is clinically known as:
B. Descending colon A. Placenta praevia
C. Caecum B. Ectopic pregnancy
D. Sigmoid (pelvic) colon C. Multiple pregnancy
E. None of the above D. Corpus luteum of pregnancy
89. Foot drop may be caused by damage to E. None of the above
the: 94. Regarding the liver Ito cells are found in
A. Common peroneal nerve the:
B. Superficial peroneal nerve A. Perisinusoidal space of Disse
C. Deep peroneal nerve B. Wall of sinusoid
D. Tibial nerve C. Around the central vein
E. None of the above D. At the portal triad only
90. The following are the features of the E. None of the above
female pelvis: 95. Functional lobes of the cerebellum
A. The sacrum is shorter and wider include the following EXCEPT:
B. The subpubic angle is more A. Spinocerebellum
rounded and wider B. Cerebrocerebellum
C. The pelvic cavity is shorter and C. Posterior (middle) lobe
wider D. A and C
D. A and C E. B and C
E. A, B and C
96. The following statement (s) is/are C. Its muscular wall contain both
TRUE: skeletal and smooth muscles
A. The placenta is expelled during D. It has submucosal glands
the third stage of labor E. All of the above
B. The umbilical cord normally
contains two arteries and one 101. Branches of the anterior division of
vein the internal iliac artery are:
C. The maternal surface of the A. Inferior gluteal artery
placenta presents with B. Superior gluteal artery
cotyledons C. Obturator artery
D. Amnion is the innermost fetal D. A and B
membrane that encloses the E. A, B and C
amniotic fluid and fetus
E. All of the above 102. The cerebrospinal fluid enters the
subarachnoid space via the:
97. Peroneus tertius is supplied by the: A. Foramina located in the
A. Superficial peroneal nerve fourth ventricle
B. Deep peroneal nerve B. Openings in the lateral
C. Tibial nerve ventricle
D. Sural nerve C. Cerebral aqueduct
E. None of the above D. Central canal of the spinal
cord
98. The following muscles extend the foot E. Third ventricle
at the ankle joint:
A. Tibialis anterior 103. The spermatic cord contains the
B. Extensor hallucis longus following EXCEPT:
C. Extensor digitorum longus A. Testicular artery
D. A and C B. Vas deferens
E. A, B and C C. Pampiniform plexus
D. Artery to the vas deferens
99. The root value of femoral nerve is: E. None of the above
A. L2, L3 and L4
B. L4, L5, S1 and S2 104. The head of the pancreas is mainly
C. L2, L5, S1, S2 and S2 associated with the:
D. L2, L2, L4, L5 and S1 A. First part of the duodenum
E. L1, L2, L3 and L4 B. Second part of the
duodenum
100. Regarding the eosophagus, the C. Third part of the duodenum
following statement(s) is/are TRUE: D. Right kidney
A. Is continous above with the E. Fundus of the stomach
pharynx opposite the sixth
cervical vertebra 105. Payer’s patches are associated with
B. It is lined mainly by stratified the following organ:
squamous epithelium A. Duodenum
B. Jejunum 110. Gluteus medius is supplied by the:
C. Stomach A. Inferior gluteal nerve
D. Ileum B. Superior gluteal nerve
E. Duodenum and ileum C. Inferior and superior gluteal
nerve
106. The following is/are TRUE: D. First and second sacral nerves
A. Fracture of the last three ribs is E. Obturator nerve
likely to cause injury to the
spleen 111. Regarding the knee joint, find the
B. The hilum of the spleen is CORRECT statement(s):
associated with lienorenal A. The anterior cruciate ligament is
ligament attached to the posterior
C. Splenic notch are located on the intercondylar area of tibia
anterior border B. The posterior cruciate ligament
D. A and C is attached to the posterior
E. A, B and C intercondylar area of tibia
C. The semilunar cartilages are
107. The portal vein is formed: attached both to the posterior
A. As a continuation of the and anterior intercondylar areas
superior mesenteric vein D. A and B
B. By the union of inferior E. B and C
mesenteric vein and splenic vein
C. By the union of inferior 112. The following cranial nerve(s)
mesenteric vein and splenic vein contains both motor and sensory fibers
at the neck of pancreas EXCEPT:
D. As a continuation of splenic A. Vagus nerve
vein B. Glossopharyngeal nerve
E. None of the above C. Hypoglossal nerve
D. Trigeminal nerve
108. Quadriceps femoris muscle consists E. Facial nerve
of the following parts:
A. Rectus femoris 113. Regarding the otic ganglia the
B. Vastus lateralis following is/are true EXCEPT:
C. Vastus medialis A. It is parasympathetic ganglia
D. Vastus intermedius B. Its post-ganglionic fibers supply
E. All of the above the parotid gland
C. The pre-ganglionic fibers travel
109. Sartorius is supplied by: via the glossopharyngeal nerve
A. Femoral nerve D. The post-ganglionic fibers travel
B. Sciatic nerve via the vagus nerve
C. Branches from lumbar plexus E. None of the above
D. Obturator nerve
E. None of the above 114. Elevation of the mandible is
performed by the following:
A. Temporalis 119. Regarding the skin, the sensory
B. Masseter receptors are located in the:
C. Medial pterygoid A. Dermis
D. A and B B. Dermal papillae
E. A, B and C C. Epidermis
D. Superficial fascia
115. Find the INCORRECT statement(s): E. Hair follicles
A. The inferior sagittal sinus
occupies the free lower margin 120. The jugular foramen transmits the
of the falxi cerebri following structures:
B. The inferior sagittal sinus joins A. Sigmoid sinus
the great cerebral vein to form B. Glossopharyngeal nerve
the transverse sinus C. Vagus nerve
C. Sigmoid sinuses are direct D. Inferior petrosal sinus
continuation of the transverse E. All of the above
sinus
D. The cavernous sinuses are 121. The vocal folds are covered by:
situated in the middle cranial A. Ciliated columnar cells
fossa B. Stratified squamous cells
E. All of the above C. Simple squamous cells
D. Pseudo-stratified ciliated
116. Levator palpebrae superioris is columnar cells
supplied by: E. Transitional epithelium
A. Occulomotor nerve
B. Trochlear nerve 122. Find the CORRECT statement(s):
C. Abducent nerve A. The cricoid cartilage is
D. Parasympathetic nerve continuous with trachea
E. Supratrochlear nerve B. The larynx is supplied by the
superior and inferior thyroid
117. Branches of ophthalmic nerve include: arteries
A. Ciliary arteries C. Elevation of the larynx is caused
B. Lacrimal artery by the stylohyoid, mylohyoid
C. Supratrochlear artery and the genlohyoid muscles
D. Infratrochlear artery D. A and B
E. All of the above E. A, B and C
118. The intrinsic muscles of the larynx are 123. In the ovary, the ovarian follicles are
supplied by: located in the:
A. Recurrent laryngeal nerve A. Ovarian medulla
B. External laryngeal nerve B. Ovarian cortex
C. Parasympathetic fibers from C. Both the ovarian cortex and
vagus nerve medulla
D. A and B D. At the hilum
E. A, B and C E. None of the above
124. Regarding the breast, the following 128. The cubital fossa contains the
is/are TRUE: following structures:
A. The glandular cells secrete via A. Median nerve
apocrine secretion B. Radial nerve
B. Basal surface glandular cells is C. Tendon of biceps brachii muscle
associated with myoepithelial D. Brachial artery
cells E. All of the above
C. It is influenced by the hormone 129. Pronator teres is supplied by the:
prolactin A. Median nerve
D. A and B B. Radial nerve
E. A, B and C C. Ulnar nerve
D. Axillary nerve
125. The following hormone act on E. Musculocutaneous nerve
smooth muscles: 130. Structures that pass posterior to the
A. Antidiuretic hormone (ADH) extensor retinaculum are:
B. Oxytocin A. Cephalic vein
C. Erythropoietin B. Basilic vein
D. Follicle stimulating hormone C. Superficial branch of radial
(FSH) nerve
E. Luteinizing hormone (LH) D. Posterior cutaneous branch of
the ulnar nerve
126. Regarding the brown adipose tissue, E. All of the above
the following is/are TRUE:
A. The cells have numerous 131. Pivot types of joints include:
mitochondria A. Elbow
B. Is well supplied with blood B. Knee
C. Metabolic activities of cells C. Ankle
causes release of large amount D. Atlanto-axial
of heat E. Wrist
D. Is present in newborn babies 132. Mesothelium forms the:
E. A, B and C A. Pericardium
127. Regarding the hand, the following B. Peritoneum
is/are TRUE: C. Pleural membrane
A. The dorsal interossei are D. A and B
supplied by the deep branch of E. A, B and C
ulnar nerve
B. Adductor pollicis is supplied by 133. Glial cells include:
the median nerve A. Ependymal cells
C. opponens pollicis is supplied by B. Microglia cells
the deep branch of ulnar nerve C. Astrocytes
D. flexor digiti minimi is D. Oligodendrocytes
innervated by the median nerve E. All of the above
E. none of the above
134. The following cell (s) myelinate 140. The left gastro-epiploic artery is a
peripheral axons: branch of the:
A. Oligodendrocytes A. Splenic artery
B. Schwann cells B. Left gastric artery
C. Astrocytes C. Common hepatic artery
D. Tanycytes D. Gastroduodenal artery
E. A and B E. Short gastric artery
135. The loose connective tissue that
surround a entire nerve is called the: 141. The greater vestibular gland is
A. Epineureum located in the:
B. Perineureum A. Vestibular bulb
C. Myoneureum B. Crus of the clitoris
D. None of the above C. Ischiorectal fossa
136. The olecranon process of the ulna D. Deep perineal pouch
receive the: E. Superficial perineal pouch
A. Articularis cubit
B. Anconeus 142. Injury to femoral nerve may lead to:
C. Brachioradialis A. Inability to abduct the thigh at
D. Brachialis the hip joint
E. Pronator teres B. Inability to extend the leg at the
knee joint
137. The largest branch of branchial C. Inability to extend the thigh at
artery: the hip joint
A. Main nutrient artery D. Loss of eversion of the foot
B. Profunda brachii artery E. Weakened adduction of the
C. Inferior ulnar collateral artery thighs
D. Superior ulnar collateral artery
E. Muscular artery to the triceps 143. The nerve associated with the lateral
brachii side of the neck of the fibula is:
138. The permanent kidney develop A. Common peroneal nerve
from: B. Deep peroneal membrane
A. Pronephros C. Saphenous nerve
B. Mesonephros D. Tibial nerve
C. Metenephros E. Superficial peroneal nerve
D. Urogenital sinus
E. Cloaca 144. The following are muscles of the
anterior compartment of the leg
139. Which of the following glands has both EXCEPT:
endocrine and exocrine functions: A. Tibialis anterior
A. Suprarenal B. Extensor hallucis longus
B. Thyroid C. Extensor digitorum longus
C. Pituitary D. Peroneous brevis
D. Salivary E. Peroneous tertius
E. Pancreas
145. The semitendineous is supplied by:
A. Femoral nerve
B. Obturator nerve
C. Tibial nerve
D. Common peroneal nerve
E. Sciatic nerve
146. Ovarian follicles contains:
A. An oocyte
B. Zona pellucida
C. Surrounding follicular cells
D. A and B
E. A , B and C
147. Meissner’s plexus is located in the:
A. Tunica mucosa
B. Tunica submucosa
C. Tunica muscularis
D. Tunica adventitia
E. Between B and C
148. Submucosal glands are found in the
following region(s):
A. Duodenum
B. Esophagus
C. Stomach
D. Ileum
E. A and B
149. The following cells show
spermiogenesis:
A. Primary spermatocytes
B. Secondary spermatocytes
C. Spermatids
D. Spermatogonia
E. None of the above
150. Gap junctions are well developed in
the:
A. Endocrine glands
B. Skin
C. Brain
D. Scalp
E. None of the above
Candidate No:………………………..

UNIVERSITY OF DAR ES SALAAM

MUHIMBILI COLLEGE OF HEALTH SCIENCES


SCHOOL OF MEDICINE

UNIVERSITY SUPPLIMENTARY EXAMINATIONS SEPTEMBER 2006


FIRST YEAR EXAMINATION FOR THE DEGREE OF

DOCTOR OF MEDICINE

ANATOMY PAPER
DATE: 18/09/2006 TIME: 9:00 – 12:00

INSTRUCTIONS

ATTEMPT ALL QUESTIONS

SECTION A: MULTIPLE CHOICE QUESTIONS

IN EACH MULTIPLE CHOICE QUESTION CHOOSE ONE THE MOST APPROPRIATE


ANSWER AND CIRCLE THE PRECEDING LETTER (1 MARK EACH).

NOTE: ¼ A MARK WILL BE DEDUCTED FOR EACH WRONG ATTEMPT

1. The glial cells include the following:


A. Oligodendrocytes
B. Microglia
C. Astrocytes
D. Tanycytes
E. A, B and C
2. Neurons may be identified by the presence of the following:
A. Nissl granules
B. Golgi bodies
C. Ribosomes
D. Nucleolus
E. Peroxisomes

3. Find the CORRECT statement (s):


A. Sensory ganglia are associated with all spinal and cranial nerves
B. Autonomic ganglia contains pre and post-ganglionic neurons
C. The neurotransmitters in most autonomic ganglia is acetycholine
D. A and B
E. B and C
4. Regarding the spinal cord, the following is/are TRUE:
A. The white matter sorrounds the grey matter
B. Give rise to spinal nerves
C. Has the central canal that communicates with the fourth ventricle
D. The gray matter is divided into sensory and motor parts
E. A, B and C
5. The following cranial nerves supplies the intrinsic muscle of the tongue:
A. Vagus
B. Hypoglossal
C. Accessory
D. Glossophharyngeal
E. Facial

6. Ventricles of the brain include the following:


A. Lateral ventricles
B. Third ventricle
C. Fourth ventricle
D. Cerebral aqueduct
E. All of the above

7. The thymus is identified by the presence of:


A. Hassals’ corpuscles
B. Large number of macrophages
C. Numerous lymphocytes
D. Presence of sinusoids
E. Lymphatic nodules
8. The tonsils include:
A. Palatine
B. Lingual
C. Tubal
D. Pharyngeal
E. All of the above
9. The following cells shows phagocytosis:
A. Macrophages
B. Neutrophils
C. Monocytes
D. A and C
E. A, B and C
10. The salivary gland include the following:
A. Parotid glands
B. Sublingual
C. Submandibular
D. A and C
E. A, B and C
11. Regarding the bone, find the CORRECT statement(s);
A. It is surrounded by fibrous tissue known as periosteum
B. Calcium salts are deposited in the intercellulat matrix
C. Red bone marrow contain erythroblasts,myeloblasts and megakaryoblasts
D. A and B
E. A,B and C

12. The following cells are found in the bone EXCEPT:


A. Osteocytes
B. Chondrocytes
C. Osteoclasts
D. Osteoblasts
E. Osteoprogenitor cells

13. Which of the following is NOT a long bone:


A. Radius
B. Ulna
C. Sacphoid
D. Metatarsal
E. Middle phalanx

14. Rigidity of the bone is mainly due to:


A. Abundance of collagen and elastic fibers
B. Calicification of the ground substances
C. Presence of harvesian canals
D. A and B
E. A and C
15. Pneumatic bone is characterized by:
A. Presence of an air sinus
B. Development in tendons
C. Cuboidal in shape
D. A and C
E. A, B and C
16. At the end of growth period, the epiphyseal plate ossifies by:
A. Synchondrosis
B. Synostosis
C. Epiphysis
D. A and C
E. A, B and C
17. Which of the following is NOT a variety of synovial joint:
A. Saddle
B. Ellipsoid
C. Pivot
D. Syndesmosis
E. Condyloid
18. The following statement is/are true EXCEPT:
A. Hinge joint allows flexion, extension and small degree of abduction
B. Muscle tone plays some roles in joint stability
C. Fibrocartilage is seen in primary cartilaginous joint
D. A and B
E. B and C
19. The following muscle are named according to their shape EXCEPT:
A. Pryamidalis
B. Trapezius
C. Deltoid
D. Rhomboid
E. None of the above
20. Sinusoids have the following features:
A. Discontinuous endothelial lining
B. Phagocytic cells in their walls
C. Discontinuous basal lamina
D. A and B
E. A, B and C
21. Regarding the urinary system:
A. The nephron is the functional units of the kidney
B. The renal corpuscles are supplied by different aretioles
C. The filtration takes place at the glomerulus
D. A, B and C
E. A and C

22. Simple squamous epithelium lines the:


A. Bowman’s capsule
B. Stomach
C. Body cavities
D. A and B
E. A and C

23. The esophagus is lined by the:


A. Stratified squamous epithelium
B. Simple squamous epithelium
C. Pseudostartified epithelium
D. Simple cuboidal epithelium
E. None of the above

24. Regarding the cell division:


A. Meiosis may be defined as reduction division
B. Chromosomes duplication takes place during the S-phase of prophase
C. A multinucleated cell may result when cytokinesis does not take place
D. A, B and C
E. A and C
25. The following nerves arises from the brachial plexus:
A. Median nerve
B. Axillary nerve
C. Radial nerve
D. Ulnar nerve
E. All of the above

26. The clavipectoral fascia above the pectoralis minor is pierced by the following structures:
A. Cephalic vein
B. Thoracoacromial artery
C. Lateral pectoral nerve
D. A, B and C
E. B and C

27. The following are contents of the axillary sheath EXCEPT the:
A. Axillary vein
B. Axillary artery
C. Axillary nerve
D. Long thoracic nerve
E. None of the above
28. The following bones form the proximal row of the carpus
A. Trapezium,trapezoid and scaphoid
B. Trapezium, lunate and hamate
C. Capitates, trapezoid and triquetral
D. Scaphoid, lunate, triquetral and pisiform
E. Triquetral, pisiform and scaphoid
29. The following muscle has double innervations:
A. Pronator teres
B. Branchialis
C. Brachioradialis
D. Flexor digitorum superficialis
E. Flexor digitorum profunda
30. The suprascapular nerve supplies the following muscles:
A. Levator scapulae
B. Supraspinatus
C. Infraspinatus
D. A and B
E. B and C
31. The trachea:
A. Is continous above with the cricoid cartilage
B. Is lined by respiratory epithelium
C. Has C-shaped cartilage
D. Is related posteriorly to the oesophagus
E. All of the above
32. Regarding the heart:
A. Coronary sinus opens into the right ventricle
B. The left atrium receives the superior and inferior vena cava
C. Atrioventricular openings have two cusps
D. The pulmonary and aortic openings are guarded by the mitral valves
E. None of the above
33. The membrane that encloses the heart is called:
A. Endocardium
B. Mediastinum
C. Peritoneum
D. Pericardium
E. None of the above
34. Regarding the respiratory system, find the CORRECT statement (s):
A. Primary bronchi are normally seen beyond the angle of Louis
B. Tertiary bronchi have also C-shaped cartilages
C. In each lung there are three secondary bronchi
D. Tertiary bronchi supply the bronchopulmonary segments
E. All of the above
35. The root of the lung contains the following structures EXCEPT:
A. Bronchus
B. Vagus nerve
C. Pulmonary ligament
D. Pulmonary artery
E. Pulmonary vein

36. The following veins opens into the right atrium:


A. Inferior vena cava
B. Coronary sinus
C. Superior vena cava
D. A and B
E. A, B and C
37. Presence of fluid in the pleural cavity is called:
A. Pneumothorax
B. Pleural effusion
C. Ascites
D. Pericardial effusion
E. None of the above
38. The diaphragm is mainly supplied by the following nerve:
A. Subcostal nerve
B. Vagus
C. Phrenic
D. Sympathetic
E. Parasympathetic
39. The cartilages of the larynx include the following:
A. Thyroid cartilages
B. Arytenoids cartilages
C. Cricoid cartilage
D. Corniculate and cuneiform cartilages
E. All of the above
40. The brachiocephalic veins are formed from the union of the :
A. Pulmonary vein and inferior vena cava
B. Internal jugular and inferior vena cava
C. External jugular and inferior vena cava
D. Internal jugular and subclavian veins
E. Azygous and axillary veins

41. Regarding the triangles of the neck:


A. The sternocleidomastoid muscle subdivides the anterior triangle of the neck
B. The common carotid artery divides within the submandibular triangle
C. The digastrics contains the submandibular salivary gland
D. A and B
E. B and C

42. Find the CORRECT statement(s):


A. The cervical plexus is formed by the posterior rami of spinal cervical nerves
B. The phjrenic nerve is a branch from the cervical plexus
C. The lesser occipital nerve is branch from the external carotid artery
D. The inferior thyroid artery is a branch of from the external carotid artery
E. All of the above
43. The thyroid gland is drained by the following vein (s):
A. Superior thyroid vein
B. Middle thyroid vein
C. Inferior thyroid vein
D. A and C
E. A, B and C
44. The following are branches from the facial nerve EXCEPT:
A. Buccal nerve
B. Chorda tympani nerve
C. Nerve to stapedius
D. Nerve to the anterior belly of the digastrics
E. None of the above

45. Regarding the venous drainage, find the FALSE statement (s):
A. The internal jugular vein receives the common facial vein
B. The ophthalmic veins drain into the cavernous sinus
C. The superficial middle cerebral vein drains into the cavernous sinus
D. The straight sinus receive the greater cerebral vein
E. None of the above

46. Regarding the larynx:


A. The superior laryngeal nerves supplies the mucous membrane of the larynx
below the vocal folds
B. The left recurrent nerve arches around the arch of the aorta
C. The recurrent laryngeal nerve is both motor and sensory to the larynx
D. A, B and C
E. B and C
47. The cervical vertebrae:
A. Are 8 in number
B. The 8 cervical spinal nerves corresponds to each vertebra
C. Vertebral artery ascends in the transverse foramina
D. One of the typical characteristic is a long spinous process
E. All of the above

48. The following structure(s) stabilize(s) the temporomandibular joint:


A. The lateral temporomandibular ligament
B. Sphenomandibular ligament
C. Stylomandibular ligament
D. Lateral pterygoid muscle
E. All of the above

49. Find the CORRECT statement(s):


A. Occipital sinus is enclosed in the attached margin of the falx cerebella
B. Inferior sagittal sinus is enclosed in the lower free margin of the falx
C. Tentorium cerebella roofs the posterior cranial fossa
D. A and B
E. A, B and C

50. Regarding the orbit:


A. Trochlear nerve supplies the inferior oblique muscles
B. Superior rectus muscle elevates and medially rotates the cornea
C. Frontal nerve contains both motor and sensory fibers
D. The eyeball is suspended by the check ligament
E. All of the above
51. The sciatic nerve supplies the following muscles:
A. Gracilis
B. Adductor longus
C. Gluteus maximus
D. Adductor brevis
E. None of the above
52. Injury to the superficial peroneal nerve may lead to:
A. Inversion
B. Dorsiflexion
C. Plantar flexion
D. Eversion
E. Extension
53. Muscles of the lateral compartment of the leg include:
A. Peroneus longus muscles
B. Peroneus brevis
C. Peroneus tertius
D. A and B only
E. A, B and C
MUHIMBILI UNIVERSITY OF HEALTH AND ALLIED SCIENCES
SCHOOL OF MEDICINE

DEPARTMENT OF ANATOMY AND HISTOLOGY


TEST NO.3 FIRST YEAR MD & DDS

NAME AND COURSE:………………………………………………………………………...

DATE: 11TH JANUARY, 2008 TIME: 10:30AM – 12:30PM


______________________________________________________________________________
INSTRUCTIONS

In each multiple choice question choose one the most appropriate answer and CIRCLE the preceding
letter (1 mark each).

NOTE: Quarter Mark Will Be Deducted For Each Wrong Answer

1. Regarding the triangles of the neck: 3. The following are branches from the
A. The sternocleidomastoid muscle facial nerve EXCEPT:
subdivides the anterior triangle A. Buccal nerve
of the neck B. Chorda tympani nerve
B. The common carotid artery C. Nerve to stapedius
divides within the D. Nerve to the anterior belly of
submandibular triangle the digastrics
C. The digastric contains the E. None of the above
submandibular salivary gland
D. A and B 4. Regarding the venous drainage, find the
E. B and C FALSE statement (s):
A. The internal jugular vein
2. Find the false statement: receives the common facial vein
A. The superior thyroid vein drains B. The ophthalmic veins drain into
into the internal jugular vein the cavernous sinus
B. The superior thyroid artery is a C. The superior sagittal sinus
branch from the subclavian drains into the right transverse
artery sinus
C. The superior thyroid artery is a D. The straight sinus receive the
branch from the subclavian greater cerebral vein
artery E. None of the above
D. The inferior thyroid vein drains
into the internal jugular vein
E. A and C
5. Regarding the larynx: 9. Regarding the orbit:
A. The superior laryngeal nerves A. Trochlear nerve supplies the
supplies the mucous membrane inferior oblique muscles
of the larynx below the vocal B. Superior rectus muscle elevates
folds and medially rotates the eyeball
B. The left recurrent nerve arches C. Frontal nerve contains both
around the arch of the aorta motor and sensory fibers
C. The recurrent laryngeal nerve is D. The eyeball is suspended by the
both motor and sensory to the check ligament
larynx E. All of the above
D. A, B and C
E. B and C 10. The salivary gland include:
A. Parotid gland
6. The cervical vertebrae: B. Sublingual gland
A. They are 8 in number C. Submandibular gland
B. Each vertebra corresponds with D. A and B
a cervical nerve E. A, B and C
C. Vertebral artery ascends in the
transverse foramina 11. The SCALP is associated with:
D. One of the typical characteristic A. Diploic veins
is a long spinous process B. Emissary veins
E. All of the above C. Occipito-frontalis muscles
D. Loose connective tissue
7. The following structure(s) stabilize(s) E. All of the above
the temporomandibular joint:
A. The lateral temporomandibular 12. The parotid gland receives its
ligament parasympathetic secretomotor fibers
B. Sphenomandibular ligament from:
C. Stylomandibular ligament A. The facial nerve
D. Lateral pterygoid muscle B. The mandibular nerve
E. All of the above C. The glossopharyngeal nerve
D. The chorda tympany nerve
8. Find the CORRECT statement(s): E. None of the above
A. Occipital sinus is enclosed in
the attached margin of the falx 13. Muscles of mastication include the
cerebella following:
B. Inferior sagittal sinus is A. Temporalis
enclosed in the lower free B. Masseter
margin of the falx C. Lateral and medial pterygoid
C. Tentorium cerebella roofs the D. A and B
posterior cranial fossa E. A,B and C
D. A and B
E. A, B and C 14. The following structures pass through
the carvenous sinus:
A. Mandibular nerve 19. Ptosis or dropping of the upper eyelid is
B. Middle meningeal nerve a clinical condition which may result
C. Internal carotid artery from:
D. Middle meningeal artery A. A lesion in the cervical
E. All of the above sympathetic trunk
B. Paralysis of the smooth muscle
15. Regarding the dura matter: and fibers of the levator
A. It has an endosteal and a palpebrae superioris
meningeal layer C. A lesion in the thoracic
B. The endosteal layer is an sympathetic trunk
ordinary periostium D. A and B
C. The meningeal layer is a strong E. B and C
fibrous tissue
D. A and B are true 20. The olic ganglion is a parasympathetic
E. A, B and C are true ganglion functionally associated with:
A. The chorda tympany nerve
16. Find the correct statement(s): B. The facial nerve
A. The falx cerebri houses the C. The glossopharyngeal nerve
straight sinus at its junction with D. The lingual nerve
tentorium cerebella E. None of the above
B. The pia matter closely invests
the brain 21. Which of the following muscles acts
C. The inferior sagittal sinus together with the superior rectus in
occupies the attached margin of moving the eyeball directly up:
the falx cerebri A. levator palpebral superioris
D. A and B B. inferior oblique
E. A, B and C C. superior oblique
D. inferior rectus
17. The following muscles are supplied by E. none of the above
the facial nerve except: 22. The following nerves supply the skin of
A. Temporalis muscle the face except:
B. Buccinator muscle A. Ophthalmic nerve
C. Occipito-frontalis muscle B. Facial nerve
D. Platysma muscle C. Maxillary nerve
E. Orbicularis oris muscle D. Mandibular nerve
E. None of the above
18. Ophthalmic artery is a branch of the:
A. Internal carotid artery 23. The extrinsic muscles of the tongue are
B. External carotid artery supplied by:
C. Facial artery A. Lingual nerve
D. Middle meningeal artery B. Hypoglossal nerve
E. None of the above C. Glossopharyngeal nerve
D. Facial nerve
E. None of the above
24. Find the true statement(s): C. Temporal bone
A. The accessory nerve contributes D. Parietal bone
to the pharyngeal plexus E. None of the above
B. The frontal air sinus open in the
inferior meatus 29. The following are the branches of the
C. The ethmoidal air sinuses are external carotid artery except:
contained in the frontal bone A. The vertebral artery
D. The olfactory nerve passes B. The occipital artery
through the foramen rotundum C. The ascending pharyngeal artery
E. None of the above D. The lingual artery
E. None of the above
25. Regarding the thyroid gland:
A. Superior thyroid artery is one of 30. Regarding the emissary veins, find the
its arterial supply correct statement(s):
B. It is drained by three veins A. They are found in the loose
C. It has only two lobes connective tissue layer
D. It is located between the fifth B. They connect the superficial
and sixth tracheal rings veins of the scalp and diploic
E. All of the above veins
C. They are valveless
26. The following are the branches of the D. B and C
internal carotid artery: E. A, B and C
A. The ophthalmic artery
B. The middle meningeal artery 31. Find the false statement, regarding the
C. The anterior cerebral artery scalp:
D. The middle cerebral artery A. It is highly vascularized
E. None of the above B. The temporalis muscle form one
of its layers
27. Find the correct statement (s): C. The periostium covering the
A. A typical cervical vertebra has skull forms one of its
bifid transverse process component layers
B. Typical cervical vertebrae have D. The occipito-frontalis muscle is
foramina transversarium supplied by the facial nerve
C. A structure known as dens is E. None of the above
found on the first cervical
vertebra 32. Regarding the dural venous sinuses, find
D. The nodding movement occurs the false statement(s):
between the atlas and the axis A. These are venous channels with
E. All of the above valves
B. They are found in the dura
28. Foramen magnum is located in the matter of the brain
following cranial bone(s): C. They communicate with diploic
A. Sphenoid bone veins
B. Occipital bone
D. They drain into the internal C. They depress the mandible
jugular veins when the hyoid bone is fixed
E. None of the above D. They fix the mandible when the
infrahyoid muscles depress the
33. Arterial supply to the scalp is from all of hyoid bone
the following except : E. All of the above
A. Branches of the ophthalmic
veins 37. Regarding the lymphoid organs around
B. Occipital artery the pharynx:
C. Vertebral artery A. Pharyngeal tonsils are found on
D. Superficial temporal artery the posterior wall of the
E. None of the above nasopharynx
B. Inflammation of the palatine
34. Regarding the venous drainage of the tonsils is commonly referred to
head and neck, find the incorrect as adenitis
statement(s): C. The lingual tonsils are paired
A. The facial vein communicates D. Children with tonsillitis are
with the pterygouid venous characterized by mouth
plexus breathing
B. The pterygoid venous plexus E. All of the above
communicates with the
cavernous sinuses via diploic 38. Find the correct statement(s):
veins A. The hypoglossal nerve gives
C. The ophthalmic veins drain into motor supply to the intrinsic
the pterygoid venous plexus muscles of the tongue
D. The ophthalmic veins are B. The facial nerve receive special
tributaries of the cavernous sensory fibers from the anterior
sinuses two-third of the tongue
E. All of the above C. Taste from the posterior one-
third is via the vagus nerve
35. The following structures are found in D. A and B
the substance of the parotid gland: E. A, B and C
A. The retromandibular vein
B. The facial nerve 39. Regarding the bones of the head and
C. The superficial temporal artery face:
D. External carotid artery A. The frontal bone forms the roof
E. All of the above of the orbit
B. The greater wing of the
36. Regarding the suprahyoid muscles: sphenoid forms part of the
A. Geniohyoid is included in this lateral wall of the orbit
group C. The maxilla forms part of the
B. The digastric muscles is floor of the orbit
included in this group D. A, B and C
E. B and C
40. Regarding the temporomandibular joint: B. It gives a meningeal branch to
A. It is bicondylar synovial joint the dura matter
B. It has 2 joint cavities C. It receives fibers from the
C. Movements occur only in the cranial part of the accessory
upper joint cavity nerve to the pharyngeal plexus
D. Infrahyoid muscles act directly D. It has both motor and sensory
on the joint components
E. A and B E. None of the above

41. Regarding the larynx: 45. The following are tributaries of the
A. Its skeleton is made up of internal jugular vein except
mainly hyaline cartilage A. The inferior thyroid vein
B. The epiglottis is a flap of B. The facial vein
hyaline cartilage that gurads the C. The retromandibular vein
inlet of the larynx D. The middle thyroid vein
C. The vocal folds are lined by E. None of the above
stratified squamous epithelium
D. A and B 46. The following are the branches from the
E. A, B and C subclavian artery:
A. The occipital artery
42. The superior oblique muscle of the eye B. The thyrocervical trunk
is supplied by the: C. The internal thoracic artery
A. Occulomotor nerve D. A and B
B. Trochlear nerve E. B and C
C. Abducent nerve
D. Vagus nerve 47. Find the correct statement(s):
E. None of the above A. The temporalis muscle
originates from the
43. Regarding the sternocleidomastoid infratemporal fossa
muscle, find the correct statement(s): B. The temporalis muscle is
A. Has motor supply from the innervated by the facial nerve
spinal part of the accessory C. The temporalis muscle
nerve protrudes the mandible
B. Receives proprioreceptive fibers D. The temporalis muscle is
from C2 and C3 covered by the temporal fascia
C. Extendes the head at the atlanto- E. All of the above
occipital joint when both
muscles act together 48. The masseter muscle:
D. The two muscles can act as A. Depresses the mandible during
accessory muscles of inspiration opening of the mouth
E. All of the above B. Is attached to the zygomatic
arch superiorly
44. Regarding the vagus nerve: C. Is one of the muscles of facial
A. It is twelfth cranial nerve expression
D. Is supplied by the maxillary 53. Find the correct statement(s):
nerve 1. Tonsillitis is a clinical condition
E. All of the above due to inflammation of the
49. Regarding the trigeminal nerve: pharyngeal tonsils
A. The ophthalmic nerve is mixed 2. The tubal tonsils are located on
nerve that supplies the skin of the lateral wall of the
the face and muscles in the orbit oropharynx
B. The mandibular nerve exits the 3. The palatine tonsils are located
skull through the foramen ovale between the palatoglossal and
C. The maxillary nerve exits the palatopharyngeal arches
skull through the foramen 4. A and B
spinosum 5. A,B and C
D. The maxillary nerve is mixed 54. The glial cells include the following:
nerve that supplies the skin of A. Oligodendrocytes
the face and palatine muscles B. Astrocytes
E. All of the above C. Microglia
D. Ependymal cells
50. Regarding the glossopharyngeal nerve: E. All of the above
A. It is cranial nerve number
twelve 55. Regarding the nervous tissue. All are
B. It is functionally related to the true except:
otic ganglion A. Ependymal cells line the
C. It gives sensory innervations to cavities of the brain and spinal
the anterior two-third of the cord
tongue B. Pseudounipolar neurons are
D. It gives motor innervations to found in the spinal root ganglia
the muscles of the tongue C. Bipolar neurons are found in the
E. All of the above olfactory mucosa
D. The axons brings impulse
51. The following are contents of the towards the cell body of the
infratemporal fossa except: neuron
A. The temporalis muscle E. None of the above
B. The lateral pterygoid muscle
C. The medial pterygoid muscle 56. Regarding the spinal cord:
D. The mandibula nerve A. The central canal is a
E. None of the above continuation of the ventricles of
52. The following are the features of the the brains
lower motor neurone lesion: B. The caudal end forms the conus
A. Flaccid paralysis medullaris
B. Decreased tone C. It end at the level of the lower
C. Marked wasting of muscles border of LL
D. Decreased tendon reflexes D. The dura matter is attached at
(hyporeflexia) the coccygeal vertebra
E. All of the above E. All of the above
57. Regarding the spinal cord: C. General sensory decussation
A. The cauda equine is formed by occurs caudal to the motor
nerve fibers traversing the decussation
subarachnoid space below LL D. A and B
B. The dorsal column contains E. A, B and C
general sensory fibers ascending
to higher centres 61. Find the correct statement(s):
C. The ventral contains cell bodies A. Spastic paralysis is a
of general sensory neurons characteristic feature of upper
D. A and B motor neurone lesion
E. A, B and C B. Lesion of the cervical part of the
spinal cord and medulla
58. Regarding the central nervous system, oblongata presents with Honer’s
find the false statement (s): syndrome
A. Cerebrospinal fluid is produced C. Flaccid paralysis occurs in
in the ventricles lower motor neurone lesion
B. Cerebrospinal fluid if absorbed D. Upper motor neurone damage
via the arachnoid granulation affects muscle groups
C. Choroid plexus synthesize the E. All of the above
cerebrospinal fluid
D. The subarachnoid space 62. The following cranial nerve nuclei are
contains cerebrospinal fluid located in pons except:
E. None of the above A. Main sensory nucleus of the
trigeminal nerve
59. Regarding the spinal cord: B. Motor nucleus of the trigeminal
A. Substantia gelatinosa and nerve
nucleus proprius run the whole C. Hypoglossal nucleus
length D. Nucleus of the abducent nerve
B. Dorsal nucleus of Clarke is a E. Motor nucleus of the facial
general sensory nucleus located nerve
in the thoracic region
C. Ventral horn contain motor 63. Regarding midbrain, find the false
neurons statement(s):
D. The intermediate (lateral) horn A. The tectum contains the
contains autonomic neurons superior and inferior colliculi
E. All of the above B. The superior are relay centres
for visual reflexes
60. Regarding the brain: C. The oculomotor nucleus is
A. Brain stem is comprised of located at the level of superior
medulla oblongata, pons and colliculus
midbrain D. The cerebral aqueduct connects
B. Diencephalon is part of the the two lateral ventricles
forebrain E. None of the above
64. Regarding the cerebral cortex: E. A, B and C
A. The pyramidal cells form
projection fibers 68. Regarding the Hutington’s disease:
B. The corpus callosum contain A. It is a hereditary autosomal
commissural fibers dominant disease
C. Projection fibers form a B. It present with dementia
component of the internal C. It present with chorea
capsule D. It commonly occurs around the
D. Other neurons form 4th -5th decade
interneurons within cerebral E. All of the above
cortex
E. All of the above 69. Find the false statement:
A. The middle cerebella peducle
65. Regarding the cerebral cortex, find the connects the pons and
true statement(s): cerebellum
A. The Wenecke’s area is located B. The red nucleus is located in the
in the lateral sulcus of the right midbrain
temporal lobe C. The pyramid contains
B. The Wenecke’s area is a descending motor fibers
secondary auditory area D. Dentate nucleus is one of the
C. Fibers from the Wenecke’s area basal ganglia nuclei
project to Broca’s motor speech E. None of the above
area
D. A and B 70. Regarding the cerebellum, find the false
E. B and C statement:
A. The purkinje cell is the sole
66. Regarding the cerebral cortex, find the output
false statement(s): B. The lateral hemisphere is the
A. The primary motor is located on cerebro-cerebellum
the pre-central gyrus C. The pendular knee jerk is one of
B. The frontal lobe is a motor lobe the symptoms of cerebellar
C. The temporal lobe is the visual lesion
lobe D. Ataxia presents with loss of fine
D. Areas 17 and 18 are visual areas movements
E. None of the above E. None of the above

67. Find the correct statement(s):


A. Lesion of thalamus may lead to
reproductive disorders
B. Diabetes insipidus may result
from lesion of thalamus
C. Parkinson’s disease is one of the
diseases of the basal ganglia
D. A and B
MUHIMBILI UNIVERSITY OF HEALTH AND ALLIED SCIENCES
SCHOOL OF MEDICINE

DEPARTMENT OF ANATOMY
CONTINOUS EXAMINATION FOR THE MD, BSCN, BSCRTT,
DDS, BPHARM, AND BMLS DEGREE PROGRAMS
NOVEMBER 2011
TIME: 1hr 30 min (90 min)

INSTRUCTIONS
1. This paper consists of 60 multiple choice questions; answer all of them
2. For each question choose the most correct answer and write its corresponding letter
alongside its question number on a different ANSWER SHEET provided.
3. Do not use the question paper for answering this examination

1. Often regarded as the father of A. Circumduction


modern anatomy, which early B. Anticlockwise rotation
anatomist first used human cadavers C. Supination
for dissection? D. Clockwise rotation
A. William Harvey E. Pronation
B. Hippocrates
C. Alexander Fleming 5. Which of the following veins of the
D. Andreas Vesalius upper limb is NOT routinely used
E. Galen foe venipuncture?
2. A plane that divides the body into A. Axillary
anterior and posterior halves is: B. Cephalic
A. Coronal C. Median cubital
B. Sagittal D. Basilica
C. Anteroposterior E. Median antebrachial
D. Transverse
E. Contralateral 6. In her first bedside learning session,
a second year nursing student
3. Pick the “odd man” out: attempted to measure the blood
A. Medial pressure of a patient by placing the
B. Superior sphygmomanometer cuff round the
C. Lateral arm region. From which artery was
D. Frontal the blood pressure detected?
E. Superficial A. Axillary
B. Basilic
4. Which movement is employed at the C. Brachial
forearm when tightening a bolt into a D. Subclavian
nut with the right upper limb? E. Cephalic
7. A typical vertebra contains the Which is the most likely nerve to be
following EXCEPT: injured that the surgeon is supposed to
A. Body be extremely be careful of during the
B. Transverse foramen operation?
C. Vertebral foramen A. Phrenic nerve
D. Spinous process B. Recurrent laryngeal nerve
E. Transverse process C. Left vagus nerve
D. Right vagus nerve
8. During his internship training a E. Sympathetic chain
radiographer noted a bone-like structure
in the lower neck region of which on 12. Muscles of the hypothenar eminence are
close examination it was found to innervated by which nerve(s)?
articulate with vertebral column. What A. Median
structure is this likely to be? B. Ulnar
A. Clavicle C. Radial
B. Cervical rib D. Median and radial
C. Atlas E. Median and ulnar
D. Axis
E. Acromion 13. The aortic arch is located in which
subdivision of the mediastinum?
9. A patient with congestive heart failure A. Superior
(CHF) was noted to have an overtly B. Middle
swollen and pulsating vein in the C. Posterior
anterior triangle of the neck below the D. Anterior
mandible. What blood vessel is this E. None of the above
likely to be:
A. Common carotid artery 14. Which of the following wrist bones is
B. Internal carotid artery found in the proximal row?
C. Internal jugular vein A. Trapezium
D. Superior vena cava B. Trapezoid
E. Brachiocephalic vein C. Capitate
D. Hamate
10. All of the following are parts of the E. Scaphoid
parietal pleura EXCEPT:
A. Cervical 15. The medial wall of the axilla is formed
B. Costal by which of the following muscles?
C. Pulmonary A. Teres major
D. Diaphragmatic B. Serratus anterior
E. Mediastinal C. Pectoralis major
D. Subscapularis
11. A neonate presenting with cardiac signs E. Pectoralis minor
and cyanosis (blue discoloration) was
recommended for surgery to obliterate a 16. The ulnar vein arises from which of the
patent ductus arteriosus (PDA). following parts of the brachial plexus?
A. Medial cord A. 1st rib
B. Lateral cord B. 3rd rib
C. Posterior cord C. 5th rib
D. Superior branch D. 7th rib
E. None of the above E. 9th rib

17. A young girl laid with her arm pressing 22. The heart is located in which anatomical
against the edge of the bed. The division of the mediastinum?
following morning the child complained A. Anterior
of the “pins and needles” in the skin on B. Middle
the medial half of the palm of the hand. C. Posterior
Which nerve was most likely affected? D. Lateral
A. Ulnar E. Superior
B. Radial
C. Median 23. How many lobes does the left lung
D. Superficial radial contains?
E. Musculocutaneous A. 1
B. 2
18. Which of the following muscles is not C. 3
named based on their shapes? D. 4
A. Pyramidalis E. 5
B. Trapezius 24. Increased resistance to pulmonary blood
C. Infraspinatus flow in the lungs would cause a direct
D. Rhomboid strain on which chamber of the heart?
E. Deltoid A. Right ventricle
B. Left atrium
19. The nerve most likely to be injured in C. Right atrium
fractures of the lateral epicondyle is the: D. Left ventricle
A. Radial E. None of the above
B. Axillary
C. Ulnar 25. Which of the following accompanies the
D. Median anterior interventricular artery?
E. Musculocutaneous A. Middle cardiac vein
B. Coronary sinus
20. Which artery supplies the posterior C. Small cardiac vein
compartment of the arm? D. Great cardiac vein
A. Axillary E. Anterior cardiac vein
B. Brachial 26. The sinuatrial node is located in the:
C. Ulnar A. Right atrial wall
D. Profunda brachii B. Left atrial wall
E. Radial C. Right ventricular wall
D. Left ventricular wall
21. Which of the following is not a TRUE E. Interventricular septum
rib?
27. The azygous vein drains into the: D. Relaxes and shrinks
A. Superior vena cava E. Recoils
B. Inferor vena cava
C. Right atrium 33. The heart is supplied by the:
D. Left brachiocephalic vein A. Coronary artery
E. Right brachiocephalic vein B. Aorta
C. Left pulmonary artery
28. The base of the heart is mainly formed D. Right pulmonary artery
by the: E. Internal thoracic artery
A. Left atrium and partly right
atrium 34. On an ultrasound scan, a radiographer
B. Right atrium and right ventricle noted a fleshly lobated organ filling the
C. Left ventricle right hypochondriac region. What did he
D. Left ventricle and left atrium observe?
E. Left ventricle and right atrium A. Liver
B. Spleen
29. The following branches arise from the C. Pancreas
aortic arch EXCEPT: D. Left kidney
A. Brachiocephalic trunk E. Right kidney
B. Left common carotid artery 35. Clinically the abdomen is usually
C. Left subclavian artery divided into how many regions:
D. Right subclavian artery A. 3
E. None of the above B. 6
C. 9
30. If a child inhales a foreign body, it is D. 10
more likely to be found at the: E. 12
A. Left main bronchus 36. A 30 years old female reported the
B. Right main bronchus gastroenterology unit with a distended
C. At the carina abdomen which on ultrasonography, the
D. Oesophagus peritoneal cavity was filled with a clear
E. none of the above fluid. What is the name of the
condition?
31. The trachea bifurcates into right and left A. Hypertension
primary bronchi at the level of the: B. Obesity
A. Plane of the sternal angle C. Ascites
B. Suprasternal (or jugular) notch D. Pregnancy
C. First rib E. Oedema
D. Seventh cervical vertebra 37. Which of the following organ does NOT
E. Xiphoid process receive blood from the celiac trunk?
A. Stomach
32. During inhalation, the diaphragm: B. Pancreas
A. Remains static C. Duodenum
B. Moves upward and forward D. Spleen
C. Moves downward and backward E. Appendix
38. An elderly woman with constipation 43. A man was admitted to the ENT clinic
(difficulty in passing stool) was found to with headache (migraine), fever and loss
have massive fecal impaction in the of appetite. A diagnosis of sinusitis
lower left abdominal region. What part (infection of paranasal air sinuses) was
of the colon was the problem localized? made. Which of these does NOT
A. Descending colon constitute the sinuses?
B. Ascending colon A. Occipital
C. Vermiform appendix B. Maxillary
D. Sigmoid C. Sphenoid
E. Transverse colon D. Ethmoid
E. Frontal
39. On CT scan, a radiographer noted a 44. The blood from most of the abdominal
fleshly blood-filled organ in the left viscera (stomach, mesentery, intestines
hypochondriac region. What did he and spleen) drains into the liver through
observe? which vein?
A. Liver A. Gastric
B. Spleen B. Splenic
C. Pancreas C. Inferior mesenteric
D. Left kidney D. Superior mesenteric
E. Right kidney E. Portal
45. Which of the following is NOT true
40. The following are bones of the skull regarding the pancreas?
EXCEPT: A. It is a glandular organ
A. Frontal B. It is finger- shaped
B. Temporal C. It secretes into the stomach
C. Parietal lumen
D. Sagittal D. Its endocrine parts secretes
E. Occipital insulin
E. Its head fills the duodenal
41. The first cervical vertebra is: concavity
A. Mastoid
B. Atlas 46. A 39 years old woman with a history of
C. Sacrum a breast cancer which was managed by
D. Axis mastectomy (surgical removal of breast
E. Acromion tissue) later developed painful swollen
masses in the axilla of the same side.
42. The mandible articulates with the skull What are these swellings likely to be?
at which bone? A. Lymph nodes
A. Frontal B. Tonsils
B. Temporal C. Pus pockets
C. Parietal D. Bursae
D. Sphenoid E. Cysts
E. Occipital
47. Which of the following lobes of the liver 52. If the inter-ventricular septum has a
is normally the most massive? congenital anomaly that allows the
A. Left communication between the two
B. Right chambers. What is the most likely
C. Caudate movement of the blood during systole?
D. Quadrate (LV = Left ventricle, RV = Right
E. Middle ventricle)
48. A young boy prone to repeated A. From LV to RV
constipation later developed very B. From RV to LV
painful bloody swellings in the anal C. Bidirectional movement
mucosal. What are these structures D. No movement at all
likely to be? E. Turbulent movement
A. Hemorrhoids 53. The inferior vena cava pours its contents
B. Swollen lymph nodes into:
C. Tonsils A. Right ventricle
D. Cancer B. Right atrium
E. Bursae C. Left ventricle
49. Which one of the following viscera is D. Left atrium
immediately continous with the stomach E. Coronary artery
distally?
A. Rectum 54. Pulmonary TB is usually diagnosed with
B. Ileum a chest x-ray (CXR). Seen from the
C. Duodenum anterior, all of the following structures
D. Jejunum are likely to be visible in the picture
E. Vermiform appendix EXCEPT:
50. A young doctor wanted to palpate A. Esophagus
(detection using fingers) a suspected B. Heart
swollen kidney. In which abdominal C. Trachea
region did she approach the organ? D. Bronchial tree
A. Hypochondriac E. Lungs
B. Epigastrium 55. The root of the lungs contains the
C. Umbilical following EXCEPT:
D. Pubic A. Bronchus
E. Lumbar B. Vagus nerve
C. Pulmonary ligament
51. Blood returning from the lungs back to D. Pulmonary artery
the heart first fills: E. Pulmonary vein
A. Right ventricle 56. A collection of serous fluid inside the
B. Right atrium pleural cavity is clinically termed as:
C. Left ventricle A. Ascites
D. Left atrium B. Pneumothorax
E. Coronary artery C. Pleurisy
D. Hydrothorax
E. Hydrocele
57. A branch of axillary artery is the:
A. Costocervical
B. Vertebral
C. Dorsal scapular
D. Suprascapular
E. Subscapular

58. Brachialis muscle is innervated by


which nerve?
A. Radial
B. Musculocutaneous
C. Ulnar
D. Radial
E. Brachial

59. The long head of the biceps brachii


muscle arises from the:
A. Infraglenoid tubercle
B. Acromion process
C. Coracoid process
D. Clavicle
E. Supraglenoid tubercle

60. Which of the following muscle does


NOT rotate the arm medially?
A. Subscapularis
B. Supraspinatus
C. Latissimus dorsi
D. Pectoralis major
E. Anterior fibers of the deltoid
MUHIMBILI UNIVERSITY OF HEALTH AND ALLIED SCIENCES
SCHOOL OF MEDICINE

DEPARTMENT OF ANATOMY
CONTINUOUS EXAMINATION FOR MD AND DDS DEGREE PROGRAMS

MAY, 9TH 2014 TIME: 1HR 30 MIN

INSTRUCTIONS
1. This paper consists of 41 questions; answer all of them.
2. For section A, choose the most correct answer and use the table provided below.
3. For section B, the ESSAY question No. 41: Organize your answers orderly and logically.
Ensure neatness of your work and legibility of words while illustrating your answers
where necessary.

YOUR REG. NO………………………………………………………..

YOUR DEGREE PROGRAM:…………………………………………….


1. The fissure that divide the two cerebral hemisphere receives a dural membrane called:
A. Dura mater
B. Falxi cerebri
C. Falxi cerebelli
D. Tentorium cerebri
E. Tentorium cerebella

2. Find the CORRECT statement:


A. The falxi cerebri houses the straight sinus
B. The pia mater closely invests the brain
C. The inferior sagittal sinus occupies falxi cerebelli
D. The arachnoid follows brain gyri and sulci contours
E. Crista galli transmit the ophthalmic nerve

3. The following are branches of the external carotid artery EXCEPT:


A. Lingual artery
B. Facial artery
C. Ophthalmic artery
D. Posterior auricular artery
E. Superior thyroid

4. The aortic valvular flaps are:


A. Anterior and posterior
B. Anterior, posterior and septal
C. Left, right and anterior
D. Left, right and posterior
E. Left and right

5. A normal and non-enlarged heart occupies the middle mediastinal position reaching the
following boundaries on the thoracic surfaces EXCEPT:
A. 2 cm rightof the sternum
B. Right axillary line
C. 5th intercostal space
D. Sternal angle of Louis
E. None of the above

6. An adolescent girl gradually developed signs of fatigue, shortness of breath and chest
pain. A cardiologist diagnosed a condition known as mitral valve prolapse (MVP). What
are the structure(s) likely to have been abnormally loose consequently leading to the
prolapse (slip out of position)?
A. Trabecular carneae
B. Chorda tendinae
C. Pectinate muscle
D. Crista terninalis
E. Papillary muscles
7. Which of the following statements is true regarding the lungs?
A. The left lung is bigger than the right
B. Adult lungs are darker than those of infants
C. Lungs receive parasympathetic supply via vagus
D. Lungs receive sympathetic supply via thoracic nerves
E. Outside the thoracic cavity lungs shrink to 1/3 of original size
8. All of the following are parts of the parietal pleura EXCEPT:
A. Cervical
B. Costal
C. Pulmonary
D. Diaphragmatic
E. Mediastinal

9. Increased resistance to pulmonary blood flow in the lungs would cause a direct strain on
which chamber of the heart?
A. Right ventricle
B. Left atrium
C. Right atrium
D. Left ventricle
E. None of the above
10. Which of the following accompanies the anterior interventricular artery?
A. Middle cardiac vein
B. Coronary sinus
C. Small cardiac vein
D. Great cardiac vein
E. Anterior cardiac vein

11. The vagus nerve accompanies which of the following as it exits the thorax downwards?
A. Sympathetic chain
B. Inferior vena cava
C. Phrenic nerve
D. Aorta
E. Esophagus
12. The sinuatrial node is located in the:
A. Right atrial wall
B. Left atrial wall
C. Right ventricular wall
D. Left ventricular wall
E. Interventricular septum

13. The azygos vein drains into the:


A. Superior vena cava
B. Inferior vena cava
C. Right atrium
D. Left brachiocephalic vein
E. Right brachiocephalic vein
14. The fracture of the medial epicondyle of the humerus is likely to injure which nerve?
A. Median
B. Radial
C. Ulnar
D. Axillary
E. Musculocutaneous
15. While playing volleyball a college student fracture his humeral shaft in the lower distal
third. From the anatomical view point which nerve is most likely to be injured?
A. Median
B. Radial
C. Ulnar
D. Axillary
E. Musculocutaneous
16. Muscular attachment to the medial border of the scapular includes the following
EXCEPT:
A. Trapezius
B. Rhomboid minor
C. Rhomboid major
D. Levator scapulae
E. Serratus anterior
17. The long head of the biceps brachii muscle arises from the:
A. Infraglenoid tubercle
B. Acromial process
C. Coracoid process
D. Clavicle
E. Supraglenoid tubercle
18. The carpal tunnel contains the following important structures except which?
A. Flexor pollicis longus tendon
B. Flexor digitorum profundus tendons
C. Median nerve
D. Flexor carpi radialis tendon
E. Flexor digitorum superficialis tendons

19. An inability to oppose the thumb to the little finger can result from damage to which
nerve.
A. Anterior interosseous
B. Posterior interosseous
C. Radial
D. Ulnar
E. Median
20. Damage to the long thoracic nerve will result into:
A. Waiter’s tip
B. Winging of the shoulder
C. Paralysis of the latissimus dorsi muscle
D. Dark discoloration of the shoulder region
E. Tingling sensation of the lower back
21. Wrist drop is a clinical condition resulting from damage of which nerve?
A. Axillary nerve
B. Musculocutanous nerve
C. Lateral pectoral nerve
D. Radial nerve
E. Median nerve

22. A surgeon wants to operate on the submandibular salivary gland. In order to reach it the
surgeon should look for the gland in the:
A. Carotid triangle
B. Digastric triangle
C. Muscular triangle
D. Submental triangle
E. Supraclavicular triangle

23. A patients walks into the doctor’s office and the doctor observes the following: one side
of the face sags and facial expression is distorted. On the same side of the face the
inferior eyelid averts (it falls away from the surface of the eyeball). On the same side
saliva is dripping from the corner of the mouth. This situation is most likely the result of
injury of which nerve?
A. Maxillary nerve
B. Mandibular nerve
C. Facial nerve
D. Trigeminal nerve
E. Hypoglossal nerve

24. The lateral pterygoid muscle:


A. Has one massive head as its origin
B. Is inserted into the front of the neck of the mandible
C. Gets its nerve supply from the maxillary nerve
D. It main action is to pull the neck of the mandible backward
E. Is inferiorly located in relation to the medial pterygoid muscle

25. The medial pterygoid muscle:


A. Depresses the mandible
B. Is supplied by the inferior alveolar nerve
C. Is inserted on the lateral surface of the mandibular angle
D. Assists in elevation of the mandible
E. Originates from the infratemporal surface of the greater wing of the sphenoid

26. The scalp has the following layers EXCEPT :


A. Loose areolar tissue
B. Pericranium
C. Bone
D. Skin
E. Subcutaneous tissue
27. The larynx gets its nerve supply from the:
A. Vagus nerve
B. Hypoglossal nerve
C. Accessory nerve
D. Glossopharygeal nerve
E. Facial nerve

28. The following are functions of the muscles of the face EXCEPT:
A. Opening and closing the lips
B. Elevation of the mandible
C. Facial expression
D. Opening and closing the eyelids
E. Speech
29. The following are branches of the mandibular nerve EXCEPT the:
A. Masseteric nerve
B. Deep temporal nerves
C. Posterior superior alveolar nerve
D. Nerve to the medial pterygoid
E. Inferior alveolar nerve

Vignette for Qn 30-34:


A middle aged man with persistent headache, facial anesthesia (loss of general sensation) and
loss of visual claws provisionally diagnosed with a hemorrhagic stroke and a CT scan of the base
of the cranium and face was ordered as confirmatory. In the scan several findings were evident
including a rupture of the internal carotid artery (ICA) leading to hemorrhage which filled space
around the neural cortical surface including gyri and sulci. Finally neurosurgical operation was
performed.
30. A normal anatomical bilateral blood filled cavity traversed by the ICA and over the bony
base of the skull was observed. What structure is this?
A. Tentorium cerebella
B. Sella turcica
C. Carvenous sinus
D. Middle cranial fossa
E. Orbital fissure
31. The bony base referred to in Qn. 30 is chiefly made of which bone?
A. Sphenoid
B. Pterygoid
C. Occipital
D. Temporal
E. Zygoma
32. Which cranial fossa lies laterally immediately adjacent to this blood-filled structure?
A. Anterior
B. Middle
C. Posterior
D. Superior
E. Inferior
33. As the artery above pierced the dura to open onto the brain surface inferiorly it was
accompanied with a big nerve on its medial aspect. Which nerve is that?
A. Optic
B. Trochlear
C. Oculomotor
D. Ophthalmic
E. Mandibular
34. Specifically, the hemmorhage reffered to is:
A. Subdural
B. Epidural
C. Subarachnoid
D. Cerebral
E. Emmissary

Vignette for Qn 35-40: A pediatric (child) at 5yrs presented with persistent nasal congestion
(filling with mucus), nasal apnea (difficulty breathing) and snoring at night. Inspection of the
nasal cavity showed whitish structures (nasal polynasal) mucosa which were then managed by
surgical resection
35. The lateral wall of the nasal cavity is normally thrown into three elevated horizotally
displaced structures called:
A. Meatus
B. Concha
C. Vomer
D. Choena
E. Sinus
36. The air sinuses associated with the nasal cavity include all of these EXCEPT:
A. Maxillary
B. Nasal
C. Sphenoid
D. Frontal
E. Ethmoid
37. The spaces underlying these elevated structures (Qn 35) are called:
A. Meatus
B. Concha
C. Vomer
D. Choana
E. Sinus
38. The general sensory supply of the nasal mucosa in the vegnette above is mainly via
which nerve?
A. Corda tympani
B. Ophthalmic and Maxillary
C. Olfactory
D. Ascending mandibular
E. Facial branches
39. The nasal mucosa in the vegnette above has a rich arterial supply which is via multiple
arteries except:
A. Sphenopalatine
B. Anterior ethmoid
C. Greater petrosal
D. Facial artery branches
E. Greater palatine
40. The air sinus assocted with the nasal cavity includes all of these following except:
A. Maxillary
B. Nasal
C. Spheoid
D. Ethmoid
E. Frontal

Essay Question:
41. In a logical and clear order, discuss the shoulder girdle highlighting the major bones,
muscles, joints, blood vessels and nerves that specifically supply it. Add one clinical note
each in regard to the bones, joints and muscles (20 marks)

You might also like